Vous êtes sur la page 1sur 132

Head Office : B-32, Shivalik Main Road, Malviya Nagar, New Delhi-110017

• Sales Office : B-48, Shivalik Main Road, Malviya Nagar, New Delhi-110017
Tel. : 011-26691021 / 26691713

Page Layout : Prakash Chandra Sahoo

Typeset by Disha DTP Team

DISHA PUBLICATION
ALL RIGHTS RESERVED

© Copyright Author

No part of this publication may be reproduced in any form without prior permission of the publisher. The author and the
publisher do not take any legal responsibility for any errors or misrepresentations that might have crept in. We have
tried and made our best efforts to provide accurate up-to-date information in this book.

For further information about the books from DISHA,


Log on to www.dishapublication.com or email to info@dishapublication.com
STUDY PACKAGE IN PHYSICS FOR JEE MAIN & ADVANCED
Booklet No. Title Chapter Nos. Page Nos.
Ch 0. Mathematics Used in Physics
Ch 1. Units and Measurements
Units, Measurements &
1 Ch 2. Vectors 1-202
Motion
Ch 3. Motion in a Straight Line
Ch 4. Motion in a Plane
Laws of Motion and Ch 5. Laws of Motion and Equilibrium
2 203-318
Circular Motion Ch 6. Circular Motion
Ch 7. Work, Energy and Power
Work Energy, Power &
3 Ch 8. Collisions and Centre of Mass 319-480
Gravitation
Ch 9. Gravitation
4 Rotational Motion Ch 1. Rotational Mechanics 1-120
Ch 2. Properties of Matter
Properties of Matter &
5 Ch 3. Fluid Mechanics 121-364
SHM
Ch 4. Simple Harmonic Motion
Ch 5. Thermometry, Expansion &
Calorimetry
6 Heat & Thermodynamics Ch 6. Kinetic Theory of Gases 365-570
Ch 7. Laws of Thermodynamics
Ch 8. Heat Transfer
Ch 9. Wave – I
7 Waves 571-698
Ch 10. Wave –II
Ch 0. Mathematics Used in Physics
8 Electrostatics Ch 1. Electrostatics 1-216
Ch 2. Capacitance & Capacitors
Ch 3. DC and DC circuits
9 Current Electricity Ch 4. Thermal and Chemical effects of 217-338
Current"
Ch 5. Magnetic Force on Moving
Charges & Conductor
Ch 6. Magnetic Effects of Current
10 Magnetism, EMI & AC Ch 7. Permanent Magnet & Magnetic 339-618
Properties of Substance
Ch 8. Electromagnetic Induction
Ch 9. AC and EM Waves
Ch 1. Reflection of Light
Ch 2. Refraction and Dispersion
11 Ray & Wave Optics Ch 3. Refraction at Spherical Surface, 1-244
Lenses and Photometry
Ch 4. Wave optics
Ch 5. Electron, Photon, Atoms,
Photoelectric Effect and X-rays
12 Modern Physics 245-384
Ch 6. Nuclear Physics
Ch 7. Electronics & Communication
Contents
Contents
Study Package Booklet 7 - Waves
9. Wave - I 571-632 9.27 Reflection of sound 607
9.28 Reverberation 607
9.1 Introduction 572
9.29 Range of hearing 608
9.2 Pulse and wave 572
9.3 Graphical representation of 9.30 Ultrasound 608

simple harmonic wave 573 9.31 Sonar 609

9.4 Sound 575 9.32 Shock waves 609


9.5 Wave front 575 Review of formulae & important points 611
9.6 Equation of a travelling wave 576 Exercise 9.1 - Exercise 9.6 612-622
9.7 Plane progressive harmonic wave Hints & solutions 623-632
or sinusoidal wave: y = a sin(wt – kx) 577
10. Wave -II 633-698
9.8 Initial phase 577
9.9 Phase and phase difference 578 10.1 Reflection and refraction of
9.10 Particle velocity and acceleration 579 sound waves 634
9.11 Wave equation , y =a sin (kx –wt) 580 10.2 Reflection and transmission of
9.12 The speed of a travelling wave 581
transverse wave in stretched string 635
9.13 Non-sinusoidal waves 581
10.3 Superposition of waves 637
9.14 Speed of transverse wave 585
10.4 Interference 639
9.15 Sound waves 588
10.5 Interference of sound waves:
9.16 The speed of longitudinal wave 589
quinke’s tube 641
9.17 Speed of sound : newton’s formula 590
9.18 Laplace’s correction 591 10.6 Interference in time : beats 644

9.19 Factors affecting speed of sound in gas 592 10.7 Stationary waves 649

9.20 Energy of a progressive wave 594 10.8 Stationary waves in stretched 654
9.21 Power transmission 595 string fixed at the ends
9.22 Intensity of sound 597 10.9 Stationary longitudinal waves
9.23 Doppler effect 599 in organ pipes 657
9.24 Doppler effect in light 601
Review of formulae & important points 668
9.25 Some important cases of doppler effect 602
Exercise 10.1 - Exercise 10.6 670-684
9.26 Characteristics of sound 606
Hints & solutions 685-698
572 MECHANICS, HEAT, THERMODYNAMICS & WAVES
9.1 INTRODUCTION
When a stone is dropped into water, its surface gets disturbed. A disturbance is produced
at the point where
the stone enters into water. This disturbance does not confine to that place along where
stone is dropped but it spreads out. The disturbance spreads in the form of the concentric
circles of ever-increasing radii eventually reach all the parts and strike the boundary of
the pond.
Now let us put a wooden block on the disturbed water surface. The wooden block
moves up and down as the ripples pass but does not move outward along with the
Fig. 9.1 ripples. This clearly indicates that the particles of water do not move outward with the
disturbance. Up and down motion of wooden block as the wave disturbance passes on
the water surface.
Thus a disturbance which communicated from one to another place without transfer of
medium is called wave motion. Such a wave which require material medium for its
propagation is called mechanical wave. The wave which requires no material medium for
its propagation is called electromagnetic wave. Remember that mechanical wave can
travel only in a medium which has elasticity.
Properties of medium require for wave motion
If a wave is to travel through a medium such as water, air, string, it must cause the
particles of medium to oscillate as it passes through the medium. For this to happen, the
medium must have inertia so that kinetic energy can be stored. Then, the particles can
overshoot their mean position. After being displaced, the particles tend to regain the
original position. For this the medium must store potential energy, which require elasticity
to do this. Thus for the propagation of wave, medium must have inertia and elasticity .
These two properties of medium decides the speed of the wave.

9.2 PULSE AND WAVE


Take a long string and attach it to a wall at its one end. Give a quick jerk to the other end
of the string. A hump is produced in the string, such a disturbance that is sudden and
lasts for the short duration is called a pulse. If jerks are continuously produced, a wave
move along the string is called progressive wave.
There are two type of waves.
1. Mechanical waves : These waves require material medium for their propagation.
Sound waves, waves in stretched string are the examples of mechanical waves.
2. Non-mechanical or electromagnetic waves: These waves require no medium for
their propagation. Light waves are electromagnetic waves.
Fig. 9.2
There are two types of mechanical waves
(i) Transverse waves :
A transverse wave is the one in which the particles of medium execute oscillations
in a direction perpendicular to the direction of propagation of wave. Transverse
waves can travel only in solids and surface of liquids.
In case of waves in stretched string, the points like C where upward displacement
is maximum are called crests. Similarly the points like T, having maximum downward
Fig. 9.3 Transverse wave in stretched
displacement are called troughs.
string.
WAVE - I 573
(ii) Longitudinal waves :
Take a slinky and hold its free end on his hand (see Fig. 9.4). Give a sudden jerk
to the free end to the right, the slinky is suddenly compressed. A compression
pulse travel along the slinky. Apply push and pull periodically on the free end of
the slinky, compressions (C) and rarefactions (R) start travelling towards the fixed
end of the slinky. Such a wave is called longitudinal waves.
Thus in a longitudinal wave particles of medium execute oscillations in the direction
of propagation of wave. Longitudinal waves can travel in solids, liquids and
gases. Sound waves in air are longitudinal.

9.3 GRAPHICAL REPRESENTATION OF SIMPLE HARMONIC WAVE Fig. 9.4. Longitudinal wave in slinky.
The particles of the medium in transverse and longitudinal waves also oscillate about
their mean positions. They execute simple harmonic motion. When the oscillation of the
particles of the medium is simple harmonic, the waves that they produced are called
simple harmonic waves.
Displacement - time graph of harmonic wave
The simple harmonic motion of oscillating particles is represented by the equation,
2
y = A sin t
T
where A is the amplitude of oscillating particle which here is called amplitude of wave, T
is the time period of oscillation.
The value of y at different time is given in the following table.

Time, t Displacement, y
0 0
T
A
4
T
0
2
3T
A
4
T 0

Displacement - distance graph of harmonic wave


You have studied that in transverse waves, the displacement of the oscillating particles
are perpendicular to the direction of propagation of wave. On the other hand, in longitudinal
waves, the particles of medium oscillate along the direction of propagation of wave.
Thus, both transverse and longitudinal waves are simple harmonic waves and can be
represented graphically as follows.
Characteristics of harmonic waves
(a) Amplitude : The amplitude of a wave is the magnitude of maximum displacement Fig. 9.5 (a)
of the oscillating particles of the medium on either side of their mean position. It
is usually represented by letter A. Its SI unit is metre (m).
(b) Wavelength : The distance between two consecutive crests or two consecutive
troughs is called wavelength. Or the distance between two consecutive
compressions or two consecutive rarefactions is called wavelength. It is usually
represented by Greek letter lambda, . Its SI unit is metre (m).
(c) Time period : The time taken by crest or trough to move a distance equal to one
wavelength is called time period. In case of longitudinal wave, the time taken by
compression or rarefaction to travel a distance equal to one wavelength is called
the time period. This time period of the wave is the same as the period of the
particle which is take to complete an oscillation. Time period is usually represented
Fig. 9.5 (b)
by T. Its SI unit is second (s).
574 MECHANICS, HEAT, THERMODYNAMICS & WAVES
(d) Frequency : Frequency of the wave is equal to the frequency of oscillation of the
particles of the medium. The frequency of an oscillating particle is the number of
oscillations completed by the particle of the medium in one complete wavelength.
Thus the frequency of a wave may be regarded as the number of complete
wavelength traversed in one second. The SI unit of frequency is hertz (Hz).
(e) Wave velocity:
It is the distance travelled by the wave in one second. The wave velocity is
usually represented by v. Its SI unit is metre/ second (m/s).
Relationship between wave velocity, frequency and wavelength
We have defined that wave velocity is the distance moved by wave in one
second. Also we know that the distance travelled by wave in time T is equal to
the wavelength of the wave, so
Distance travelled
Wave velocity = Time taken

or v =
T
1
As = f
T
v = f
Thus wave velocity is the product of frequency and wavelength of wave.

Ex. 1 Sol.
(a) A wave transfer momentum. Can it transfer angular (a) A narrow pulse does not have a definite wavelength or frequency.
momentum? But being a sound, it has a definite speed.
(b) Frequency is the most fundamental property of a wave. Why? (b) The frequency of the note produced by the whistle is not equal to
(c) Which of the following is not a wave characteristic: Reflection, 1/20 or 0.05 Hz, it is only the frequency of pulse repetation.
refraction, interference, diffraction, polarisation, rectilinear
propagation?
(d) What is a non-dispersive medium? Give an example. Ex. 3
(e) We always see lightening before we hear thundering. Why?
(f) What is the difference between wave velocity and particle
velocity?
Sol.
(a) As the particles of the medium have translational motion, no
rotatory motion when wave travels, so there is no transfer of
Fig. 9.6
angular momentum.
A wave is travelling along the x -axis, whose displacement-time
(b) When wave travels from one medium to other , its speed and
graph is shown in Fig. 9.6. Find period and frequency of wave.
wavelength change. But frequency does not change. This is the
reason that frequency is the fundamental property of a wave. Sol.
(c) Rectilinear propagation is not a wave characteristic. The time period of wave
(d) If speed of a wave in any medium is independent of frequency, T = 0.50 s
then mediums called non-dispersive. For example, air is a non- The frequency of oscillation
dispersive medium for sound waves.
1 1
(e) The speed of light (3 × 108m/s) is much greater than the speed of f = = = 2 Hz Ans.
sound (340m/s). So, the flash of light reaches us much earlier than T 0.50
the sound of thunder . Ex. 4 A source of wave produces 40 crests and 40 troughs in 0.4
(f) The wave velocity is constant v = f in a given medium while the second. Find the frequency of the wave.
particle velocity changes harmonically with time. The particle
velocity is zero at the extreme position.
Sol.
The total number of waves produced in 0.4 s is 40.
Ex. 2 A narrow pulse (for example, a short pip by a whistle) is
sent across a medium. n 40
The frequency of wave, f ==
(a) Does the pulse have a definite t 0.4
(i) frequency, (ii) wavelength, (iii) speed of propagation? = 100 Hz Ans.
(b) If the pulse rate is 1 after every 20 s, (that is the whistle is Ex. 5 A boat at anchor is rocked by waves whose consecutive
blown for a split of second after every 20s), is the frequency of crests are 100 m apart. The wave velocity of the moving crests is 20
the note produced by the whistle equal to 1/20 or 0.05 Hz. m/s. What is the frequency of rocking of the boat?
WAVE - I 575
Sol. Ex. 7 Earthquakes generates sound waves inside the earth.
Given, wavelength of wave, = 100 m Unlike a gas, the earth can experience both transverse (S) and
longitudinal (P) sound waves. Typically the speed of S wave is about
and wave velocity, v = 20 m/s 4.0 km/s, and that of P wave is 8.0 km/s. A seismograph records P and
Frequency of rocking of boat = frequency of wave S waves from an earthquake. The first P wave arrives 4 min before
v 20 the first S wave. Assuming the waves travel in straight line, how far
or f = = = 0.20 Hz Ans. away does the earthquake occur?
100
Sol.
Ex. 6 A longitudinal wave is produced on a toy slinky. The wave
Suppose the earthquake occurs at a distance x km from the seismograph.
travels at a speed of 30 cm/s and the frequency of the wave is 20 Hz. Time taken by the S wave to reach the seismograph
What is the minimum separation between the consecutive
compressions of the slinky? x
t1 = s
Sol. 4
Time taken by the P wave to reach the seismograph
Given, speed of wave, v = 30 cm/s
and frequency of wave, f = 20 Hz x
t2 = s
8
v 30
Wavelength of wave, = As t1 – t1 = 4 × 60s
f = 20 = 1.5 cm
x x
Thus the separation between the consecutive compressions = 4 × 60
4 8
= 1.5 cm. Ans.
or x = 1920 km Ans.

9.4 SOUND
Sound is a form of energy which creates the sensation of hearing in human ears. Sound
can be produced by vibrating turning fork, by colliding bodies, by mouth etc. In
production of sound one form of energy changes into sound energy e.g. when we clap,
a sound is produced. In this process muscular energy changes into sound energy.
Sound waves are longitudinal waves
When sound waves propagate the density as well as the pressure of the medium at a
given time varies with the distance, above and below their average values. The figure
represents variation of density and pressure of a medium when sound wave propagates.
At some regions of medium the density as well as pressure is high. These regions are Fig. 9.7
called compressions. At some regions of medium the density as well as pressure is law.
These regions are called rarefactions. In the figure peak represents the regions of maximum
compression (pressure amplitude) and trough represents the rarefaction. The distance
between two consecutive compressions (C) or two consecutive rarefactions (R) is called
the wavelength, .

Note: Waves may be one-dimensional, two dimensional or three dimensional


according to the propagation of energy in one, two or three dimensions. Thus, transverse
waves in stretched string or longitudinal waves along a rod are one dimensional, surface
waves or ripples on water are two dimensional and sound waves produced by horn are
three dimensional.

9.5 WAVE FRONT


A plane or surface on which particles of the medium are in an identical state of motion at
a given instant, i.e., in the same phase, is called a wave front.
In an isotropic medium, the wave front is always perpendicular to the direction of the
wave motion and the position of a given wave front shifts outwards with the wave
speed. A line normal to the wave front thus gives the direction of propagation of the
wave is called a ray. The wave front may have different shapes. One dimensional waves
like waves in stretched string produces plane wave front and such waves are called
plane waves . A point source produces spherical wave front. All wave front at a large
distance from the source become plane wave front. Fig. 9.8
576 MECHANICS, HEAT, THERMODYNAMICS & WAVES
9.6 EQUATION OF A TRAVELLING WAVE
Consider a disturbance travels along positive direction of x with a constant speed v. It
x
travels a distance x in time . Let the displacement of the particle at a distance x from the
v
origin at any time t is y. The same amount of displacement of the particle situated at the
x
origin was at t . Therefore we can write
v
x
y ( x, t ) = y x 0, t .
Fig. 9.9 v
The displacement of the particle at x at time t i.e., y (x, t) can simply be written as y, and
so wave equation can be written as
x
y = f t … (1)
v
Equation (1) represents a wave travelling in the positive x - direction. Such a wave is
called a travelling or progressive wave. In such a wave time t and x must appear in the
t x/v
x x T
wave equation in the combination t . For example y = t , y = Ae ,
v v

t x/v x2 v2t 2
y = A sin . The equations y = A sin(ax 2 bt ) , y = A sin do not
T L
represent travelling wave. The equation of wave travelling in negative x -direction can
be written as
x
y = f t . …(2)
v
The wave equation can also be written as
vt x
y = f
v
As v is constant, so we can write
y = f (vt – x)
or y = g (x – vt) …(3)

Note:
1. The equation y = f (vt –x) represents the displacement of the particle at x = 0 as time
passes:

Fig. 9.10
2. The equation y = g (x – vt) represents the displacements of different particles of the
medium at any time.
WAVE - I 577
9.7 PLANE PROGRESSIVE HARMONIC WAVE OR SINUSOIDAL
WAVE : y = Asin( t – kx)
In the process of travelling of wave if particles of the medium vibrate simple harmonically
about their mean positions, then the wave is called plane progressive harmonic wave or
sinusoidal wave. In harmonic wave of given frequency, all particles have same amplitude
but phase of oscillation changes from one particle to the next. The displacement of a
oscillating particle at x = 0 and at any instant of time can be written as :
y = A sin t ...(1)
or y = A cos t ...(2)
where A is the amplitude of the particle. The displacement of any particle at x at time t will
be
x
y = f t
v

x
The required equation can be obtained by replacing t by t in the equation y = A sin
v
t. Thus we have
x
y = A sin t ...(3)
v

= A sin t x
v

2 f 2
But = = =k
v f
k is called propagation constant or angular wave number.
Hence
y = A sin( t kx ) …(4)
Equation (3) can be written in the following forms:

y = A sin (vt x)
v
2
or y = A sin (vt x) …(5)

v x
Also y = A sin 2 t

v 1
As = f=
T
t x
y = A sin 2 …(6)
T

9.8 INITIAL PHASE


If the particle of the medium does not start from mean position at t = 0, then there needs
an information regarding with its initial displacement. This can be possible by introducing
a quantity 0, is called initial phase. Thus if 0 is the initial phase, then the wave
equation can be written as
y = A sin( t kx 0)
578 MECHANICS, HEAT, THERMODYNAMICS & WAVES
Positive and negative phase:
If at x = 0, t = 0 ; y = A , then
A = A sin ( ×0–0+ 0)

or 0 = (Positive phase)
2
If at x = 0, t = 0 ; y = –A, then
–A = A sin ( ×0–k×0+ 0)

or 0 = – (Negative phase)
2

The equation of wave corresponding to 0 = becomes


2

y = A sin t kx
2
Fig. 9.11
= A cos( t kx )

and for 0 = –
2

y = A sin t kx
2
or y = –A cos ( t – kx)

9.9 PHASE AND PHASE DIFFERENCE


Phase of a wave: The phase of a harmonic wave is a quantity that gives complete
information regarding with wave at any time and at any position. It is equal to the
argument of the sine or cosine function representing the wave. In wave equation
y = A sin( t kx 0) , the phase of the wave = ( t kx 0)
Clearly, the phase of a wave is periodic both in time and space. At any position x, it
changes with time and at any time t , it changes with position.
Phase change with position:
We have = t kx 0

At any time t, the phase change can be obtained by differentiating above equation
k x =
Thus the phase difference between two particles separated by distance xis given by
2
= k x = x

The negative sign indicates that farther the particle is located from the origin in the
positive x -direction, the more it lags behind in phase.
Phase change with time:
We have = t kx 0

At any position x, the phase change with time can be obtained by differentiating
above equation.
2
= t = t
Fig. 9.12 T
WAVE - I 579

x t
0 0 0 0
T
4 2 4 2
T
2 2
2 T 2

Ex. 8 What is the phase difference between the particles 1 and Sol.
2 located as shown in Fig. 9.13.
v 360
Sol. (a) Given = 60° =
3
rad and = f =
500
= 0.72 m
The distance between the particles
We know that,

x = 2
2 8 4 = x

5 0.72
=
8 x 2 3 2
2 = 0.12m Ans.
= . x
(b) Phase difference with time is given by

2 5 2
= . t
= Fig. 9.13 T
8
5 1 1
= Ans. Here T = f = 500 = 0.002s
4
Ex. 9 A wave of frequency 500 cycles/s has a phase velocity of 2 3
360 m/s . (a) How far apart are two points 60° out of phase? (b) What = 10
0.002
is the phase difference between two displacements at a certain
= 3.14 rad = 180° Ans.
point at times 10 –3 s apart?

9.10 PARTICLE VELOCITY AND ACCELERATION


Particle velocity : In the process of wave motion, the particle velocity changes with time.
It can be obtained by differentiating displacement of the particle w.r.t. time.
dy d
vp = = [ A sin( t kx )]
dt dt
or vp = A cos( t kx) …(1)

or vp = A sin t kx …(2)
2
(i) Clearly particle velocity vp changes simple harmonically with time while wave
velocity v = f remain constant.
Fig. 9.14
(ii) The particle velocity leads displacement in phase by radian.
2
(iii) The maximum particle velocity, v0 = A.
dy
Slope of displacement curve: It is defined by .
dx
dy d
Thus slope, = [ A sin( t kx )]
dx dx
dy
or = kA cos( t kx) …(3)
dx
580 MECHANICS, HEAT, THERMODYNAMICS & WAVES
Dividing equation (1) by (3) , we get
vp A
= = –v
dy / dx kA

dy
or vp = v …(4)
dx
or particle velocity at a point = – (wave velocity) × (slope of displacement curve at
that point)
Particle acceleration : The acceleration of the particle can be obtained by differentiating
particle velocity. Thus
dv p d
a = = [ A cos( t kx)]
dt dt

or a = 2
A sin( t kx )
As y = A sin( t kx )

a = 2
y

Also a = 2
A sin( t kx )
(i) The maximum value of particle acceleration a0 = 2A.

(ii) The particle acceleration leads particle velocity by and displacement by


2
radian.

Note:
1. For a wave travelling along positive x - axis, we can write
y = A0y sin ( t – kx) for transverse wave
and y = A0x sin ( t – kx) for longitudinal wave.
2. In wave equation displacement y can be replaced by pressure, electric field,
magnetic field. For electric field we can write
E = E0 sin ( t – kx)
dy
3. As vp = v , so for positive slope, the velocity is negative and vice-versa.
dx

Fig. 9.15

9.11 WAVE EQUATION, y =A sin (kx – t)


The equation
y = A sin (kx – t) …(1)
can be used to find the displacements of all the particles of the wave as a function of time.
If this equation is used to represent a wave in stretched string, then it can tell us the
shape of the wave at any given time and how that shape changes as the wave moves
Fig. 9.16 along the string.
WAVE - I 581
9.12 THE SPEED OF A TRAVELLING WAVE
Consider a wave travelling along positive x - direction. Fig. 9.17 shows two snap shots
of the wave at a small interval of time t. Let x is the movement of entire wave pattern
in time t , then wave speed is defined as
x
v = …(2)
t
As the wave move, each point of the moving wave form, retain its displacement. In figure
point P moves to P in time t, but its displacement y remain constant. Fig. 9.17
Therefore, sin (kx – t) = constant
or (kx – t) = a constant …(3)
Differentiating equation (3) w.r.t. time, we get
dx
k = 0
dt

dx
or = v= …(4)
dt k
Note:
It should be remembered that although argument (kx – t) is constant, but both x and t
are changing.

Equation (1) represents a wave moving along positive x -direction. A wave travelling in
the negative x -direction is described by the equation
y = A sin[ (kx t )] …(5)
d
For wave speed, [ ( kx t )] = 0
dt
dx
or = v= , …(6)
dt k
here negative indicates that wave is moving along negative x-direction.
Thus general equation of a travelling harmonic wave (transverse or longitudinal) can be
written as:
y (x, t) = y = g (kx t) …(7)

Note:
The wave equations y = A sin( t kx ) and y = A sin(kx t ) have phase difference of
radian, but when we speak of a moving wave, we mean displacement of all the particles
at any time, and therefore y = A sin(kx t ) should be used according to the rule.

9.13 NON-SINUSOIDAL WAVES


T
1. Square wave: In square wave, there is a sharp over shot at t = 0 and at t = .
2
For a square wave (at x = 0)
T
y = +A for 0 t
2
T Fig. 9.18. Square wave.
and = 0 for t= ,
2
T
and = –A for t T
2
582 MECHANICS, HEAT, THERMODYNAMICS & WAVES
2. Saw-tooth wave: It can be represented by the equation
t
At x = 0 y= A 1 for 0 t T
T
y=A, at t = 0,
and y = 0, at t = T/2
Fig. 9.19. Saw-tooth wave. Differential equation of wave:
An equation will represent a wave if it satisfy the following differential equation.
2 2
y y
= v2
2 2
t x
where v is the wave velocity or phase velocity.

Note:
If any function of x and t satisfy the differential equation but if not finite, then it will not
represent a wave. For example y = n x vt ) etc. satisfy the differential equation , but not
represent wave.

Ex. 10 Show that the function y ( x , t ) = Ae - B ( x - vt )2 represents and differentiating expression (i) twice w.r.t. x , we have
a travelling wave. 2
y
= 2
Sol. x2
Given y = B ( x vt ) 2 ...(i) 2
Ae 2
y 2 y
Clearly, = v
y t 2 x2
= B ( x vt ) 2
t Ae ( 2 B )( x vt ) ( v)
Thus expression (i) is a solution of the one-dimensional wave
B ( x vt )2 equation.
= 2 ABv( x vt )e
Similar treatment can be done for (ii), (iii) and (iv).
2
y
and = Note: The expression (iv) satisfies differential equation of
t2 wave, but it does not represent progressive wave. It represents
stationary wave.
B ( x vt ) 2 B ( x vt )2 (v) Differentiating expression (v) twice w.r.t. we have
2 ABv ( x vt )e ( 2 B )( x vt )( v ) e ( v)
2
y
= 2 ABv 2e B ( x vt )2 = –2v2
[2 B ( x vt ) 2
1] … (ii) t2
y and differentiating expression (v) twice w.r.t. x, we have
Now = B ( x vt )2
x 2 ABe ( x vt ) 2
y
2 = 2
2 x
y
and = 2 ABv 2e B ( x vt ) 2
[2 B ( x vt )2 1] ...(iii)
x2 2 2
y y
On comparing equations (ii) and (iii), we get Clearly 2
v2
, so the expression (v) is not a solution
2
t x
2 2
y 2 y of the one dimensional wave equation.
= v
t 2
x2 (vi) Differentiating expression (vi) twice w.r.t. t, we have
Also the given function is finite for each value of t, so it will represent a 2
y
wave. 2 = v2 sin 2 x cos vt = v2 y
t
Ex. 11 Show that (i) y = x2 + v2t2, (ii) y = (x + vt)2, (iii) y = (x – vt)2
and (iv) y = 2 sin x cos vt are each a solution of one dimensional 2
y
wave equation but not (v) y = x2 – v2 t2 and (vi) y = sin 2x cos vt. and 2 = 4sin 2 x cos vt = 4y
x
Sol.
(i) Differentiating expression (i) twice w.r.t.t, we have 2
y 2
y
Clearly 2
v2
, and therefore the expression is not a
2
2 t x
y
= 2v 2
t2 solution of the one dimensional wave equation.
WAVE - I 583
Ex. 12 The shape of a wave pulse at time t is given by the Ex. 13 The equation of a wave travelling on a string stretched
along the x-axis is given by
2x
function f (x) = 2 , where a = 1 cm–2. The wave is travelling
1 ax x t 2
– +
along positive x -axis with velocity of 4 cm/s . Graph the wave function a T
y = Ae
at times, t = 0s, 2s and 3s. where is the maximum of pulse located at t = T?
Sol. Sol.
Wave pulse at any time t can be obtained by putting x – vt in place of x. The maximum of the pulse is y = A, when
Thus we wave
x t 2
T = 1
e

x t
or = 0
a T

x
At t = T, 1 = 0
a
x = –a Ans.
Ex. 14 Fig. 9.21 shows a snapshot of a vibrating string at t = 0.
The particle P is observed moving up with velocity 20 3 cm/s. The
tangent at P makes an angle 60° with the x -axis. Find

Fig. 9.20
Fig. 9.21
2( x vt ) (a) the direction in which wave is moving,
y =
1 a ( x vt ) 2 (b) equation of wave.

2x Sol.
At t = 0 s, y = (a) Suppose y = A sin (kx – t + ) …(i)
[1 (1) x 2 ]
Given (from figure), at t = 0, x = 0; y = 2 2 and A = 4
2x
=
1 x2 2 2 = 4 sin (0 – 0 + 0)

2[ x 4 2] 3
which gives 0 = or
At t = 2 s, y = 1 (1){x 4 2}2 4 4
We know that particle velocity
2( x 8)
= dy
1 ( x 8) 2 vp = v
dx

2( x 4 3) or 20 3 = v tan 60
At t = 3s, y = 1 (1){x 4 3} 2 v = – 20 cm/s
It indicates that wave is travelling along negative x - axis.
2( x 12) (b) From the figure, = (5.5 – 1.5) = 4 cm
= 2
1 ( x 12) v 20
f = = = 5s–1
The wave function at given times are shown in Fig. 9.20 . 4
584 MECHANICS, HEAT, THERMODYNAMICS & WAVES
Propagation constant Sol.
2 2
k = = =
4 2 Given y = 3.0sin 36t 0.018 x ...(i)
and = 2 f=2 ×5 4
= 10 rad/s The standard equation of a harmonic wave travelling along negative x -
On substituting these values in equation (i), we get direction is
x y = A sin( t kx ) ...(ii)
y = 4sin 10 t
2 4
On comparing equations (i) and (ii), we have
x 3 = 36 rad/s, k = 0.018/ m
and = 4sin 10 t Ans.
2 4
Ex. 15 For a travelling harmonic wave y = 2.0 cos (10t – 0.0080x and 0 =
4
rad
+ 0.35), where x and y are in cm and t in s. What is the phase
difference between oscillatory motion at two points separated by a 36
(i) v = =
3 k 0.018
distance of (i) 4m, (ii) 0.5 m (iii) , (iv) ?
2 4 = 2000 cm/s = 20 m/s
Sol. (ii) A = 3.0 cm
Given y = 2.0 cos (10t – 0.0080x +0.35)
The standard equation of travelling harmonic wave can be written as 36
f = = = 5.73 s–1
y = A cos ( t – kx + ) 2 2
On comparing two equations, we have
= 10 rad/s
and k = 0.0080 m–1 (iii) Initial phase 0 = rad
4
2 (iv) Least distance between two successive crests
or = 0.0080
2 2
2 2 = = = = 349.0 cm
= = m k 0.018
0.0080 0.0080 100
= 3.49 m Ans.
2
=
0.80
Ex. 17 A travelling harmonic wave on a string is described by

2
Phase difference = x y = 7.5 sin 0.0050 x + 12 t + .
4
2 (i) What are the displacement and velocity of oscillation of a
(i) When x = 4 m, = 4 =3.2 rad
2 / 0.80 point at x = 1cm, and t = 1s? Is this velocity equal to the
velocity of wave propagation?
2
(ii) When x = 0.5 m, = 0.5 = 0.40 rad (ii) Locate the points of the string, which have the same transverse
2 / 0.80 displacement and velocity as the x = 1cm point t = 2s, 5s, 11s.
2 Sol.
(iii) When x= , = = rad
2 2
Given y = 7.5sin 0.0050 x 12t ...(i)
3 2 3 3 4
(iv) When x = , = = rad.
4 4 2
The standard equation of a travelling wave is
Ex. 16 y = A sin( t + kx + 0) ...(ii)
A transverse harmonic wave on a string is described by y (x, t) = 3.0
On comparing equations (i) & (ii), we get
sin (36t + 0.018x + ), where x, y are in cm and t in s. The positive
4
A= 7.5 cm, = 12 rad/s , k = 0.0050 cm–1 and 0 = rad.
direction of x is from left to right. 4
(i) Is this a travelling or a stationary wave? If it is travelling, (i) At x = 1cm and t = 1s, displacement of the particle
what are the speed and direction of its propagation.
(ii) What are its amplitude and frequency?
(iii) What is the initial phase at the origin? y = 7.5sin 0.0050 1 12 1
4
(iv) What is the least distance between two successive crest in the
wave? = 7.5 sin 12.79 = 1.67 cm Ans.
WAVE - I 585
The velocity of the particle Ex. 18 The Fig. 9.22 shows two snap shots, each of a wave
dy travelling along a particular string . The phase for the waves are
vp = given by
dt

d
= 7.5sin 0.0050 x 12t
dt 4

= 7.5 12cos 0.0050 x 12t


4

90cos 0.0050 x 12t Fig. 9.22


=
4 (a) 4x – 8t
At x = 1 cm and t =1s (b) 8x –16t. Which phase corresponds to which wave in the figure?
Sol.
vp = 90cos 0.0050 1 12 1
4 The standard equation of a plane progressive wave is y = A sin ( kx t) ,
= 90 cos 12.79 = 87.76 cm/s the phase of the wave is ( kx t) .
Velocity of propagation
(a) Comparing 4x –8t with ( kx t ) , we get
12 = 8 and k = 4
v = = = 2400 m/s
k 0.0050
2 2
Clearly velocity of the particle is not equal to the velocity of 1 = = =
k 4 2
wave.
2 (b) Comparing 8x – 16t with ( kx t ) , we get
(ii) As k = = 0.0050 = 16 and k = 8

2 2 2
= = 1256.64 cm 2 = = =
0.0050 k 8 4
Clearly 1 = 2 2, snap-shots 1 and 2 correspond to a and b
All points located at distance (where is an integer) from the
respectively.
point x = 1 cm have the same transverse displacement and velocity.

9.14 SPEED OF TRANSVERSE WAVE


If a wave is to propagate through a medium, it must cause the particles of the medium to
oscillate as it passes. The oscillations of the medium possess kinetic energy and potential
energy both. For that to happen, the medium must possess mass for kinetic energy and
elasticity for potential energy. Thus mass and elasticity of the medium determine the
wave speed.
Consider a transverse wave moving from left to right along a string with speed v. For the
purpose, we can choose the reference frame attached to moving pulse. In this frame the
string appears to move from right to left (see Fig. 9.23).
Fig. 9.23. A pulse viewed in a frame
Consider a small element of the string of length , forming an arc of a circle of radius R attached to the pulse.
and subtending an angle at the centre of that circle. A force F pulls the string on this

element at each end. The net force 2Fsin acts vertically downwards on the pulse. The
2
particles on the arc (pulse) are made to rotate in a circle due to this force exerted by
neighbouring parts of the element. By Newton’s second law

mv 2
2 F sin =
2 R
If is the mass per unit length of the string, then mass of the element, m = . For

small , sin = . In view of these relations, we get


2 2
586 MECHANICS, HEAT, THERMODYNAMICS & WAVES

( )v 2
2F =
2 R

As = ,
R
F = v2

F
or v =

Note:
1. The wave shape in stretched string is sinusoidal, but on being small element, we
have assumed it as circular.
2. If A is the area of cross- section of string and is the density, then

F F
= (A 1 ) = A , v= . As = stress, v = Stress / Density .
A A

Speed of transverse waves in solid


The speed of transverse wave in solid medium is determined by modulus of rigidity of
the material of the solid and density of the solid (determine the inertia property). Thus
the speed of transverse wave in solid is given by

v =

Ex. 19 A copper wire is held at the two ends by rigid supports. At Sol.
30°C, the wire is just taut with negligible tension. Find the speed The force at any section of the rope is the weight suspended from the
of transverse waves in the wire at 10°C. section. If FA and FB are the tensions at ends A and B respectively, then
( =1.7 × 10–5/°C, Y = 1.4 × 10111N/m2 and = 9 × 103kg/m3)
FA = 2g and FB = 8g
Sol.
The thermal stress in the wire corresponds to change in temperature t is
F = Y t
If A is the cross-sectional area of the wire, then tension produced in the
wire
F = fA = Y tA
Speed of transverse wave in the wire
F Y tA
v = =
( A 1)
Fig. 9.24
Y t If vA and vB are the speeds of wave at ends A and B respectively, then
=
vA f A FA /
vB = f = FB /
11 5 B
1.4 10 1.7 10 10
=
9 103
A FA 2g 1
= 51.42 m/s Ans. or = FB = 8g
=
B 2
Ex. 20 A uniform rope of length 12m and mass 6 kg hangs
vertically from a rigid support. A block of mass 2 kg is attached to B = 2 A
the free end of the rope. A transverse pulse of wavelength 0.06 m is = 2 × 0.06
produced at the lower end of the rope, what is the wavelength of the = 0.12 m Ans.
pulse when it reaches the top of the rope?
WAVE - I 587
Ex. 21 A uniform rope of mass 0.1 and length 2.45 m hangs Here F = 100 N and = m = m0 + x
from a ceiling.
100
(i) Find the speed of transverse wave in the rope at a point 0.5 m v =
distance from lower end. (m0 x)
(ii) Calculate the time taken by a transverse wave to travel the
full length of the rope. dx 1
or = 10
Sol. dt ( m0 x )1/ 2
Given length of the rope = 2.45m . If is the mass per unit length of the
1
rope, then mass of the y length of the rope, m = y or dt = ( m0 x)1/ 2 dx
Tension at the point y from the free end F = mg = yg 10

t 10
1
dt = 10 (m0 x )1/ 2 dx
0 0

10
1 ( m0 x)3/ 2
t = 10 (3/ 2)
0

1
Fig. 9.25 = ( m0 10)3/ 2 m0 3/ 2
15
F 1 2 3
The speed of transverse wave, v = = 3
(10 9 10 10)3/ 2 (10 2 )3/ 2
15 9 10
yg = 0.23 s Ans.
= = gy Ex. 23 The amplitude of a wave disturbance propagating along
(i) At y = 0.5m , 1
positive x -axis is given by y = at t = 0 and
v = 9.8 0.5 = 2.21 m/s Ans. (1 + x 2 )

dy 1
(ii) We can write = gy y= at t = 4 s, where x and y are in metre. The shape of
dt 1 + ( x - 2)2
dy wave disturbance does not change with time. Find velocity of the
or dt = gy wave.
Sol.
t 2.45
dy The equation of the wave at any time t can be obtained by putting (x – vt)
or dt =
gy in place of x in the given expression, so we have
0 0
1
2 2.45 y = ...(i)
| y |0 1 ( x vt ) 2
t = g
1
2 Given y = at t = 4s ...(ii)
= 2.45 0 1 ( x 2) 2
g
On comparing equations (i) and (ii) , we get
= 1s Ans. vt = 2
Ex. 22 A wave pulse starts propagating in the positive x direction
2 2
along a non-uniform wire of length 10 m with a mass per unit As t = 4s, v = = = 0.5 m/s Ans.
length given by m = m0 + x and under a tension of 100 N. Find the t 4
time taken by the pulse to travel from the lighter end (x = 0) to the Ex. 24 A pulse is propagating on a long stretched string along
heavier end. (m0 = 10–2 kg/ m and = 9 × 10–3kg/m2 ). its length taken as positive x -axis. Shape of the string at
Sol. t = 0 is given by,
The speed of transverse wave is given by
y = a2 - x2 when | x | a
F
v = = 0 when | x | 0
Study the propagation of this pulse if it travelling in positive
x- direction with v.
Fig. 9.26
588 MECHANICS, HEAT, THERMODYNAMICS & WAVES
Sol. Ex. 25 A circular loop of string rotates about its axis on a
The given equation can be written as frictionless horizontal plane at a uniform rate so that the tangential
y2 = a2 – x2 speed of any particle of the string is v. If a small transverse
or x + y2 = a2
2 disturbance is produced at a point of the loop, with what speed
(relative to the string) will this disturbance travel on the string?
It represents a circular shape of the pulse.
Sol.
Suppose F is the tension in the string due to its rotation. Choose a small
element of the string of length . If is the mass per unit length of the
string, then mass of the element,
m = = (R ).
Using Newton’s second law for the element, we have

mv 2
2 F sin =
2 R

For small , sin =


2 2

2F mv 2
=
Fig. 9.27 2 R
The wave shape at any time can be obtained by putting (x – vt) in place
of x in the given equation, so we can write v2
or F = ( R )
R
y = a2 ( x vt ) 2 F = v2
when | ( x vt ) | a The speed of the disturbance Fig. 9.28

= 0 when | x vt | 0 F v2
= =v Ans.
The wave shape at any time t is shown in Fig. 9.27.

9.15 SOUND WAVES


Sound waves, we mean the longitudinal waves in air which, when strike the ear, produce
the sensation of hearing. The human ear is sensitive to waves in the frequency range
from about 20 to 20000 Hz.
The sound waves in an elastic medium can be described by a wave function of
displacement
y = A cos(kx – t), ...(1)
here cosine function is taken to make the final wave equation positive. In practice, it is
easier to measure pressure variation in a sound wave than the displacement, so it becomes
necessary to develop a relation between these two quantities.
Figure shows an oscillating element of air of thickness x and cross-sectional area S.
Let the element is displaced towards right by an amount y.
The volume of the element
V = S x
The change in volume of the element due to its displacement y
V = S y.
This change in volume occurs because the displacement of the two faces of the element
are not equal. The bulk modulus B of the medium is given by

P
B =
V
V
Fig. 9.29

V
or P = B
V
WAVE - I 589
After substituting the values of V and V in above equation, we get
S y
P = B
S x

y
= B
x

y y
For small x, . The symbol is used for partial differentiation.
x x

y
P = B ...(2)
x
Substituting y from equation (1) , we get
A cos(kx t)
P = B
x
= ABk sin (kx – t) ...(3)
The maximum amount by which pressure differs (usually from atmospheric pressure),
Fig. 9.30
that is maximum valueof P, is called the pressure amplitude, denoted Pm, so equation
(3) can be written as;
P = Pm sin(kx – t) ...(4)
where Pm = ABk. The equation (4) is also called pressure wave. The equation can also
be written as;
P = Pm cos(kx – t – 2) ...(5)
If P0 be the normal pressure of the air, then pressure varies between ( P0 – Pm ) to
( P0 Pm )or ( P0 ABK ) to ( P0 ABk ).

Relationship between P and vp


The particle velocity is given by the equation
vP = A cos( t kx) ...(6)

The maximum velocity, called the velocity amplitude v0, is given by v0P = A
and pressure amplitude P0 =ABk,
P0 Bk
=
v0

Bv0 P
As = v, so P0 =
k v

Bulk modulus×velocity amplitude


Pressure amplitude =
Wave speed

9.16 THE SPEED OF LONGITUDINAL WAVE


Consider a long tube filled with air. Let a single pulse travels towards left with a speed v.
To understand easily, let us take a frame attached to the moving pulse. In this frame pulse
appears stationary and the air is moving towards right with a speed v.
Let the normal air pressure be P and the pressure inside the pulse be P + P, taking P
positive for the compression. Take an element of air of thickness x and face area S,
moving toward the pulse at a speed v. As the element of air enters the pulse, its speed
changes from v to v + v, in which v is negative. The speed decreases due to a region
of higher pressure.
590 MECHANICS, HEAT, THERMODYNAMICS & WAVES

Fig. 9.31
If t is the time in which rear face of air element reaches the pulse, then x = v d ...(1)
The net force on the element
F net = PS ( P P)S
or F net = PS ...(2)
The mass of the air element
m = density × volume
= ( S x ) = Sv t ...(3)
Using Newton’s second law for the element of mass m, we have
F net = m × a
v
or PS = ( Sv t )
t
or P = v v
P
or v =
v
P
or v2 =
( v / v)
...(4)

The volume of the air element V = S x = Sv t , and the compression as it enters the
pulse,
V = S v t.
Thus
V S v t v
volumetric strain = = ... (5)
V Sv t v
Substituting this value in equation (4), we have
P
v2 =
( V /V )
=B

After rearranging , we get


B
v = ...(6)

9.17 SPEED OF SOUND : NEWTON’S FORMULA


By his observations, Newton obtained a formula for speed of sound in air as :
P
v = , ...(1)
WAVE - I 591
where P is the isothermal elasticity of the air. He argued that when sound propagates
through air, the temperature of air remain constant. By Newton’s formula the speed of
sound at one atmosphere is

1.013 105
v = 280m/s
1.29
This value is less than the experimental value 332 m/s. Hence Newton’s formula requires
some correction, which was made by Laplace in 1816.

9.18 LAPLACE’S CORRECTION


French scientist Laplace pointed out that when sound propagates in air the heat of the
medium remain constant instead of temperature. So he replaced isothermal elasticity by
adiabatic elasticity, Bad. The corrected formula is :

Bad
v = ...(2)

For the adiabatic change, PV = constant


Differentiating both sides, we get
P ( V – 1) dV + V dP = 0
or PdV +VdP = 0
dP
or = P
dV
V

dP
As dV = B ad
V
B ad = P ...(3)
where = CP / Cv ,is the ratio of specific heats.
Hence Laplace formula for the speed of sound in air (gas) is

P
v = ...(4)

7
For air = , so the speed of sound in air at STP will be
4

P 7
v = = 280
5
= 332 m/s
This value is in very close agreement with the experimental value.

Note: Speed of sound in different


mediums
4
B
1 Speed of longitudinal wave in a solid is given by, v = 3 , where is the Steel 5960(m/s)
Solid
Aluminium 6420
modulus of rigidity and B is the bulk modulus. Liquid Water 1498
Y Air 340
2. Speed of longitudinal wave in solid rod is given by, v = , where Y is the Gas
Oxygen 316
Young’s modulus of the material of the rod.
592 MECHANICS, HEAT, THERMODYNAMICS & WAVES
Ex. 26 At a pressure of 105 N/m2, the volume strain of water is Speed of sound in water
5× 10–5.Calculate the speed of sound in water. Density of water is
103 kg/m3.
B 2 109
v = =
Sol. Bulk modulus of water 103

Normalstress = 1414 m/s Ans.


B = Volumestrain

105
= 5 = 2 109 N / m2
5 10

9.19 FACTORS AFFECTING SPEED OF SOUND IN GAS


(i) Pressure: The speed of sound in a gas is given by
P
v =

m
We know, PV = nRT = RT
M
m
At constant temperature, P V = RT
M
m RT
P =
V M
RT
or P =
M
P
or = constant

i.e, with the change in pressure, the density also changes in such proportion, so
P
that remains constant. Hence pressure has no effect on the speed of sound in

a gas.
(ii) Effect of density : For two gases of densities 1 and 2 at same pressure with 1
and 2,
v1 1 2
v2
= .
2 1

P RT
(iii) Temperature : We have got =
M

RT
v =
M
Clearly v T
Hence the speed of sound in a gas is proportional to the square root of its
absolute temperature. If v0 and vt are the velocities of sound in gas at 0°C and
t°C respectively, then
R (273 0)
v0 =
M

R (273 t )
and vt =
M
WAVE - I 593

1
vt 273 t 2
v0 273
1
t 2
1
273
For small value of t
1 t
vt v0 1
2 273
v0t
or vt = v0
546
But v0 = 332 m/s,
332t
vt v0 = = 0.61t
546
When t = 1°C, vt v0 = 0.61 m/s
Hence the velocity of sound in air increases by 0.61 m for every 1°C rise in
temperature.
(iv) Humidity : With the increase in humidity, the density of air decreases. As the
speed of sound in air is
1
v ,

the speed of sound will increase.


(v) Frequency : With the change in frequency of the sound wave, wavelength
inversely changes, so that f v (constant). Thus the speed of sound is
independent of its frequency.
(vi) Wind : As the sound is carried by air, so its speed is affected by the wind
velocity. Suppose the wind is blowing with a velocity v at an angle with the
direction of propagation of the sound. Clearly, the component of wind velocity in
the direction of sound is v cos .
Resultant velocity of sound = v v cos
When the wind blows in the direction of sound ( 0) , resultant velocity Fig. 9.32
=v+v
When wind blows in the opposite direction of sound ( 180 ) , resultant velocity
=v–v

Ex. 27 At what temperature will the speed of sound be double Ex. 28 A tuning fork of frequency 220 Hz produces sound waves
its value at 273°K? of wavelength 1.5 m in air at STP. Calculate the increase in
Sol. wavelength, when temperature of air is 27°C.
Sol.
v2 T2
We have, = Given f =220 Hz, 0 = 1.5 m at T0 = 273 K
v1 T1
Given v2 = 2v 1 Speed of sound at STP, v0 = f = 220 1.5 = 330 m/s.
0
Final temperature, T = 273 + 27 = 300 K
2v1 T2 Let v the speed of sound at this temperature, then
v1 = T1
v T
or T2 = 4T1 = 4 (273) = 1092 K Ans. =
v0 T0
594 MECHANICS, HEAT, THERMODYNAMICS & WAVES

T
Ex. 30 The absolute temperature of air in a region linearly
v = v0 increases from T1 and T2 in a space of width d. Find the time taken
T0 by sound wave to goes through the region in terms of T1, T2, d and
the speed v of sound at 273 K.
300 Sol.
= 330 = 346.1 m/s
273 The variation of temperature with the distance is shown in Fig. 9.33.
Consider a section of air at a distance x from the end at temperature T1.
v 346.1
Final wavelength, = f = 220 = 1.57 m T2 T1
The temperature at this section T = T1 x
d
The increase in wavelength = 0 = 1.57 – 1.50 = 0.07 m Ans.
Ex. 29 A sample of oxygen at NTP has volume V and a sample
of hydrogen at NTP has volume 4V. Both the gases are mixed and
the mixture is maintained at NTP. If the speed of sound in hydrogen
at NTP is 1270 m/s, calculate the speed of sound in the mixture.
Sol.
If VH and Vm are the velocities in hydrogen and mixture respectively, then
Fig. 9.33
vm The speed of sound at any temperature is given by
H
vH = ...(i)
m RT
vt =
M
oVo H VH
Density of mixture, m = Vo VH R 273
At 273 K, v =
M
where o and Vo are the density and volume of the oxygen.
T
vt = v
o Vo 273
H VH 1
H VH
m = 1
Vo dx v T2 T1 2
VH 1 or = T1 x
VH dt 273 d

t d
o Vo dt 273 dx
1 or = 1
m H VH 0 v 0
or = T2 T1 2
H Vo T1 x
1 d
VH
d
1
1
1 16 T2 T1 2
4 T1 x
= 1 =4 d
1 2 273 0
4 or t =
v T2 T1
H 1 d
or =
m 4
From equation (i), 2d 273 T2 T1
=
1 vH v T2 T1
vm = vH =
4 2
2d 273
1270 = v T2 Ans.
= = 635 m/s Ans. T1
2

9.20 ENERGY OF A PROGRESSIVE WAVE


We know that, in wave motion, the energy derived from the source is transferred from
one part of the medium to other part of the medium. The work done by the source in any
way gets associated with two kinds of the energy of the medium; kinetic energy and
potential energy.
WAVE - I 595
1. In taut string, the forces due to tension in the string continuously do work to
transfer energy from regions with energy to regions with no energy. The oscillating
string element possesses both its maximum kinetic energy and its maximum elastic
potential energy at y = 0. The oscillating element has maximum speed and hence
kinetic energy at y = 0 and also maximum stretching and hence maximum potential
energy at y = 0 . The regions of the string at maximum displacement have no
energy.
Explanation:
It is clear from the figure that the string element (b), its length has its normal Fig. 9.34
undisturbed value dx, so its potential energy is minimum. However when the
element passes through its y = 0 position, it is stretched to its maximum, and
possess maximum potential energy.
2. In case of sound wave energy is transferred due to work done by the source by
creating pressure difference in the medium.

9.21 POWER TRANSMISSION


Consider a thin element of the medium of mass dm parallel to the wave front (it may be
string element or air element). If is the mass per unit length of the medium, then dm =
dx.
Kinetic energy: The kinetic energy dK associated with the element
1 1
dK = dmvP2 = dx vP2 ...(1)
2 2
where vp is the speed of the oscillating element . vP can be obtained as;
y
vp = = [ A sin(kx t )]
t t
= A cos(kx t)
Fig. 9.35
1 2 2 2
dK = ( dx ) A cos (kx t)
2
The rate at which kinetic energy is carried by the wave
dK 1 dx 2 2
= A cos 2 (kx t)
dt 2 dt

dK 1
or = v 2 A2 cos 2 (kx t) ...(2)
dt 2
The average rate at which kinetic energy is carried can be obtained as:
The average value of cos 2 (kx t ) over the wavelength.

cos 2 (kx t )dx


0 1
=
2
dx
0

dK 1 2 2
Power, P kinetic = = v A ...(3)
dt av 4
Potential energy : The force acting on the element
F = (dm) × acceleration
2
y
= ( dx)
2
t
596 MECHANICS, HEAT, THERMODYNAMICS & WAVES
2
= dx [ A sin(kx t )]
t2
2
= dx A sin(kx t)

dx 2 y
=
The work done in a small displacement dy of the element
dW = Fdy = dx 2
ydy
Work done during the displacement 0 to y
y

W = dx 2 ydy
0

1 2 2
= dx y
2
1 2
= dx [ A sin( kx t )]2
2
1
dx 2 A2 sin 2 (kx
= t )]
2
This work done must be stored up in the medium in the form of potential energy. Thus
potential energy
1 2 2
U = dx A sin 2 (kx t)
2
The rate at which potential energy carried
dU 1 dx 2 2
= A sin 2 (kx t)
dt 2 dt
1
=v 2 A2 sin 2 (kx t)
2
The average rate at which potential energy carried can be obtained as:
The average value of sin 2(kx – t over the wavelength

sin 2 (kx t )dx


0 1
=
2
dx
0

dU 1
Power, P potential = v 2 A2 = ...(4)
dt4 av
Total power transmitted: The average power, which is the average rate at which energy
of both kinds is transmitted by the wave, is then
Pav = P kinetic + P potential
1
or Pav = v 2 A2 ...(5)
2
If is the volume density and S is the area across which power transmitted, then
= S
1 2 2 1 2 2
Pav = ( S )v A = vS A ...(6)
2 2
WAVE - I 597
9.22 INTENSITY OF SOUND
The intensity I of a sound wave (or any other wave) at any point of a surface is the sound
energy transferred through unit surface area perpendicular to the direction of propagation
of wave in unit time.
Or it can be defined as the power transferred through unit surface area perpendicular to
the direction of propagation of wave. If P is the power transferred through surface area
S, then intensity of wave can be written as
P
I =
S
SI unit of intensity is W/m2. We have already derived that
1 2 2
P = vS A
2
P 1
I = = v 2 A2 ...(1)
S 2
For sound wave
Pm = ABk
Pm
A =
Bk
Substituting this value in equation (1), we get
2
1 2 Pm
I = v
2 Bk

1 2 Pm2
= v
2 B2 k 2

As K = and B = v 2
v
2
1 2 Pm
I = v
2 2
B2
v2

v Pm2 Pm2
I = = ...(2)
2B 2 v

Variation of intensity with distance


Let us assume that mechanical energy of the sound waves is conserved as they spread
from the source.
1. The plane or one dimensional wave travels without change in intensity, i.e., with
its amplitude undiminished, the intensity of the wave remains the same throughout.
For power of source P and surface S, the intensity
P
I =
S
As S remain constant, so I remain constant.
2. The power P of the line source will spread over a cylindrical surface. The surface Fig. 9.36. A plane wave
area of the cylinder of radius r is 2 rL, where L is the length of the line source .
The intensity is given by
P P
I = = ...(1)
S 2 rL
598 MECHANICS, HEAT, THERMODYNAMICS & WAVES

1
Clearly, I .
r

As 1
I A2 , A
r
3. The power P of the point source will spread over the spherical surface. The
intensity of the source at a distance r from the source is given by
P
I = ...(2)
4 r2
1 1
Clearly I .As I A2 , A .
2 r
r
Note:
In view of these facts, we can write the following wave equations
A
(i) y= sin(kx t ) for plane progressive wave.
r
A
(ii) y= sin(kx t ) for waves from a line source.
r
Fig. 9.37
A
(iii) y= sin(kx t ) for waves from point or spherical source.
r

Ex. 31 The maximum pressure amplitude Pm that the human 28


=
ear can tolerate in loud sounds is about 28 Pa. What is the 343 1.21 2 1000
displacement amplitude A for such a sound in air of density = 1.1 × 10–5 m Ans.
= 1.21 kg/m3, at a frequency of 1000 Hz and a speed of 343 m/s? Ex. 32 A line source of sound of length 10 m, emitting a pulse
Sol. of sound that travels radially outward from the source. The power
The pressure amplitude is related to displacement amplitude as of the source is P = 1.0 × 104 W. What is the intensity I of the sound
when it reaches a distance of 10m from the source.
Pm = ABk
Sol.
The intensity at a distance r from a line source is given by
= Av 2
v P
I =
= Av 2 f 2 rL
Displacement amplitude
1.0 104
Pm = = 15.92 W/m2 Ans.
A = v (2 f ) 2 10 10

Sound level: The decibel scale


The lowest intensity of sound that can be perceived by the human ear is called threshold
of hearing . For a sound of frequency 10 kHz, the threshold of hearing is, I 0 = 10–12 W/m2.
The ratio of intensities of the loudest to faintest sound is 1012. Humans can hear over a
large range of intensities.
Instead of expressing the intensity I of a sound wave, it is more convenient to express of
its sound level , which is defined as

I
= (10 dB )log
I0
Here dB is the abbreviation for decibel, the unit of sound level. The unit bel was introduced
in honour of Alexander Graham Bell.
WAVE - I 599
( 2 – 1): Let 1and 2 are the sound levels corresponding to sound intensities I1
and I2 respectively, then
I1
1 = 10 log
I0

I2
and 2 = 10 log
I0

I2 I1
2 1 = 10 log log
I0 I0

I2
or 2 1 = 10 log
I1

Ex. 33 If the intensity is increased by a factor 100, by how Ex. 34 A sound level at a point 5.0 away from a point source is
many decibels is the sound level increased? 40 dB. What will be the level at a point 50 m away from the source?
Sol. Sol.
We know that The ratio of intensities of sound

I2 I1 r22 50
2
2 1 = 10 log I = = = 100
1 I2 r12 5
= 10 log 100
= 10 × 2 = 20 dB Ans. I2
2 1 = 10 log
SOUND LEVELS OF DIFFERENT SOUNDS I1
= 10 log 100 = 10 × 2
Source of sound Sound levels in decibels
= 20 dB Ans.
1. Threshold of hearing 0
2. Rustle of leaves 10
3. Whisper 15-20
4. Normal conversation 60-65
5. Heavy traffic 70-80
6. Roaring of loin 90
7. Thunder 100-110
8. Painful sound 130 and above
9. Rocket launch 160

9.23 DOPPLER EFFECT


You might have observed that when a train blowing its whistle approaches you, standing
on a railway platform, the pitch of the whistle appears to rise and it appears to drop as
the train moves away from you. Doppler was the first to analyse this apparent change in
frequency and he stated that whenever there is a relative motion between source of
sound, the observer and the medium; the frequency of sound as received by the observer
is different from the frequency of sound emitted by the source. This apparent change in
the frequency of sound due to relative motion between source and observer is called
Doppler effect.
Doppler effect is a wave phenomenon, it holds for sound waves as well as for light
waves. Doppler effect in sound depends on three factors:
(i) Velocity of the source vs (ii) Velocity of the observer vo (iii) Velocity of the medium
or wind v .
600 MECHANICS, HEAT, THERMODYNAMICS & WAVES

Note:
Doppler effect is noticeable only when the relative velocity between the source and the
observer is an appreciable fraction of the wave speed.

Apparent frequency when the source moves towards the


stationary observer :
Consider a source produces sound of frequency f. If v is the speed of sound in air then
the wavelength of the sound wave
v
= = vT
f
Now suppose the source S moves towards stationary observer O with a speed vs. In one
time period T, S moves a distance vsT, before it emits next pulse. As a result the wavelength
becomes
' = vT – vsT
( v vs )
=
f
Because of this changed wavelength, frequency of the sound appears to change. If f is
the apparent frequency, then

v v
f = = f v v ... (1)
' s
If the source moves away from the observer, then
v
Fig. 9.38 f = f ...(2)
v vs
Alternate method:
At t = 0, suppose the source is at a distance L from the observer and emits a compressional
pulse. It reaches the observer at time
L
t1 =
v
The source emits next compressional pulse after a time T. In the mean time, the source
has moved a distance vsT towards the observer and is now a distance L – vsT from the
observer. The next compressional pulse reaches the observer at time
L vsT
t2 = T
v
The time interval between two successive compression pulses
L vsT L
T' = t 2 t1 = T
v v

vs v vs
= T 1 =T
v v
The apparent frequency
1 fv 1
f' = = v v f
T' s T
Apparent frequency when observer moves towards the
stationary observer:
Consider a source produces sound of frequency f. If v is the speed of sound in air, then
the wavelength of the sound wave
WAVE - I 601

v
= f
Now suppose the observer O moves towards stationary source with a speed of vO. The
speed of sound relative to observer
v ' = v vO
As the source is stationary, so wavelength of the sound wave remain same. Because of
the increased relative speed of the sound, the frequency appears to increase . Thus
apparent frequency
v' v vO
f' = =
v/ f

v vO
or f' = f ...(3)
v
If the observer moves away from the stationary source, then
v vO
f' = f ...(4)
v
General Doppler effect equation:
The apparent frequency can be written as ;
v vO
f' = f ...(5)
v vs

9.24 DOPPLER EFFECT IN LIGHT


The speed of light c is invariant irrespective of the motion of source or observer. Therefore
the apparent change in frequency or wavelength of light does not depend on the fact
whether source is moving or observer is moving . It depends on relative velocity between
them.
The change in wavelength can be obtained by
v
= ...(1)
c
where c speed of light
v velocity of source w.r.t. observer
actual wavelength of light
f
As f = c (constant), so = ...(2)
f
Red shift and blue shift:
When source of light moves away from the observer the wavelength of light coming
from the source appears to increase . It is known as red shift, and ' = .
When source is approaching towards the observer, then the wavelength of light coming
from the source is appears to decrease. It is known as blue shift, and ' = .
Doppler effect in sound is asymmetric:
Suppose a source of sound moves towards a stationary observer with a speed v ' , then
the observed frequency
v
f' = f
v v'
Now if the observer moves towards the stationary source with the same speed v ' , then
the observed frequency
v v'
f" = f
v
602 MECHANICS, HEAT, THERMODYNAMICS & WAVES
Clearly, f ' f " . That is the observed frequency in two cases is different, although the
relative speed between them is same . For this reason, the Doppler effect in sound is said
to asymmetric. However, the Doppler effect in light is symmetric. This is because the
observed frequency or wavelength depends only on relative speed between source and
observer.

Note:
No Doppler effect is observed in the following situations:
(i) When both the source and the observer move in the same direction with the same
speed.
(ii) When both the source and the observer are at rest and wind is alone is blowing.
(iii) When the distance between source and observer remain constant. When either
the source or the observer is at the centre of a circle and other is moving along it
with an uniform speed.
(iv) The Doppler effect is noticeable when vo or vs < v. It does not hold when the
speed of the source or the observer becomes equal or greater than the speed of
the wave.

9.25 SOME IMPORTANT CASES OF DOPPLER EFFECT


v vo vs vo
1. f' = f s o
v vs

v vs
and '=
v

v vo vs
2. f'=f v v vo
s

v vs
and '= v

v vo
3. f' = f v v vs vo
s

v vs
and '=
v

v vo
4. f' = f v v vs v0
s

v vs
and '=
v
5. Moving source crosses a stationary observer
v
Apparent frequency before crossing, f1 = f
v vs
Fig. 9.39
v
Apparent frequency after crossing, f2 = f
v vs

Change in apparent frequency, f = f1 f2


WAVE - I 603

1 1 2 fvvs
= fv v v v vs = 2 2
s v vs

2 fvvs
=
v 2 vs 2

2 fvs
for vs v f =
v
6. Observer crosses a stationary source

v vO
Apparent frequency before crossing f1 = f
v

v vO Fig. 9.40
Apparent frequency after crossing f2 = f
v

2 fvO
Change in apparent frequency f = f1 f2 =
v
7. When source is moving in a direction making an angle w.r.t. the observer
Velocity of source towards observer – vscos . The apparent frequency heard by
the observer at rest

v
f' = f v v cos
s

As source moves along line ABC,


at B; f' = f. Fig. 9.41
90 ,

v
At C, f' = f v v cos .
s

8. Apparent frequency before crossing the function J

v vo cos 2
f' = f v vs cos 1 Fig. 9.42
9. (a) When source moves on a circular path and observer is standing at the centre
of the circular path. Since there is no relative motion between source and
observer, so the frequency perceived by the observer at the centre will be
same as the frequency of the source.
(b) Observer is standing very far away from the circular path.
When the source approaching towards the observer
Fig. 9.45
v
f' = f v v
s

v
and f' = f v v
s

when source is receding from the observer. Fig. 9.46


604 MECHANICS, HEAT, THERMODYNAMICS & WAVES
10. Source is moving towards a wall :
(a) Observer in between source and wall.
The frequency of direct sound perceived
v
f1 = f
v vs
Fig. 9.47 The frequency of reflected sound perceived
v
f2 = f
v vs
f = 0
(b) Source in between observer and wall: The frequency perceived of direct
v
sound f1 = f
v vs
The frequency perceived of reflected sound
Fig. 9.48 v
f2 = f
(v v s )

1 1 2 fvvs
f = f1 f2 = fv = 2 2
v vs v vs v vs
(c) When observer is moving together with source:
The frequency of sound perceived at wall
v
f1 = f
v vs

Fig. 9.49 Frequency of the reflected sound perceived


v vo
f2 = f1
v

v vo
= f
v vs
Also vS = v o

11. If wind is blowing with velocity v , then

(v v ) v0
f' = f (v v ) v
s
Take (+) sign with v when wind is blowing from source towards observer, and
when wind is blowing from observer toward source.

Ex. 35 A source emitting a sound of frequency f is placed at a


large distance from an observer. The source starts moving towards
the observer with a uniform acceleration a . Find the frequency
heard by the observer corresponding to the wave emitted just after
the source starts. The speed of sound in the medium is v.
Sol.
Let initially source is at a distance L from the observer. Let first wave Fig. 9.50
front starts from the source at t = 0. The first wave front will reach the 1 2
The distance moved by the source in this time aT .
L 2
observer t1 = . The second similar wave front is emitted after a time T.
v
WAVE - I 605
Time of motion of sound = time of motion of train
1 2
The separation between source and observer becomes L aT .
2 1 (1 – x ) x
or 1200 40 (1200 40) =
That it is the distance of second wavefront from the observer. 40
After solving, we get x = 0.0646 km
1 2
L aT Distance from the hill = 1–x
Time taken by second wavefront to reach the observer t = 2 = 1– 0.0646 km
v = 0.9354 km
The frequency as heard by the driver:
Let f1 is the frequency as observed at the hill

(v v )
f1 = f
( v v ) vs

Fig. 9.51 1200 40


Time taken by second wavefront from initial position of source = 580
1200 40 40
1 2 599.3 Hz.
L aT
t2 = 2 This frequency is reflected back and wind opposes the sound
T
v
(v v ) v0
Time interval between two successive wavefronts is f2 = f1
(v v )
1 a 2
t = t2 t1 = T T 1200 40 40
2 v
= 599.3
Hence , frequency received by observer 1200 40
= 620 Hz Ans.
1 1
f' = = Ex. 37 Fig. 9.53 shows a source of sound moving along the
t 1 a 2
T T x - axis at a speed of 22 m/s continuously emitting a sound of
2 v frequency 2.0 kHz which travels in air at a speed of 330 m/s.
A listener Q stands on the y -axis at a distance of 330 m from the
2vf 2 1 origin. At t = 0, the source crosses the origin at P. (a) When does the
or f' = as T
2vf a f sound emitted from the source at P reach the listener at Q? (b)
What will be the frequency heard by the listener at this instant ?
Ex. 36 A train approaching a hill at a speed 40 km/ h sounds a (c) Where will the source be at this instant?
whistle of frequency 580 Hz, when it is at distance of 1 km from the
hill. Wind with speed of 40 km/h is blowing in the direction of
Sol.
motion of the train. Find (a) frequency of whistle as heard by an
observer on the hill (b) the distance from the hill at which echo
from the hill is heard by the driver and its frequency. Velocity of
sound in air = 1200 km/h.
Sol.

Fig. 9.53
(a) Sound produced at P takes time to reach
330
Q = = 1s
330
(b) The frequency of sound as heard by listener = 2 kHz.
(c) When sound arrive at Q, the source will move a distance
Fig. 9.52 x = vs × t
(a) The frequency of sound as heard by the observer at hill = 22 × 1
= 22 m Ans.
(v v ) Ex. 38 A source emitting sound at frequency 4000 Hz, is moving
f1 = f
( v v ) vs along the y - axis with a speed of 22 m/s . A listener is situated on the
ground at the position (660 m, 0). Find the frequency of the sound
1200 40 received by the listener at the instant the source crosses the origin.
= 580
1200 40 40 Speed of sound in air = 330 m/s .
= 599.3 Hz Ans. Sol.
(b) Let x is the distance moved by the train in the duration sound come Let sound produced at P will reach the listener at the instant when source
back after reflection from hill. crosses the origin.
606 MECHANICS, HEAT, THERMODYNAMICS & WAVES
Time of motion of sound from P to listener
f v v
= time of motion of source from P to origin = f = f
f c c
y y 2 660 2 44 44 Now moon becomes source of frequency f1 = (f + f) the shift in
or = cos
2
22 330 44 660 2 661.5 frequency in reflected light is observed,

After solving vf
f =
y = 44 m c
The frequency heard by the listener
v
v Therefore total shift observed = 2 f = 2f
f c
f' = v vs cos
9 v
103 = 2 5 10
330 3 108
= 4000 Fig. 9.54
330 22cos
v = 30 m/s Ans.
= 4018 Hz Ans.
Ex. 40 A bat is fitting about in a cave, navigating via ultrasonic
Ex. 39 An astronaut is approaching the moon. He sends a radio bleeps. Assume that the sound emission frequency of the bat is
signal of frequency 5 × 109 Hz and finds that the frequency shift in 40 kHz. During one fast sweep directly towards a flat wall surface,
echo received is 103Hz. Find his speed of approach. the bat is moving at 0.03 times the speed of sound in air. What
Sol. The frequency shift as observed on moon frequency does the bat hear reflected off the wall?
Sol.
The frequency of the sound reflected from the wall as perceived by the
bat
v vo v 0.03v
= f 40
f' v vs = v 0.03v
= 42.47 kHz Ans.

Fig. 9.55

9.26 CHARACTERISTICS OF SOUND


Sounds produce by adults and children are different , still both the sounds travel at the
same speed. This is due to the different characteristics associated with the sound. Pitch
is one of the characteristics.

Pitch : Pitch is the characteristic which distinguishes between shrill and grave sound.
The faster the vibration of source, the higher is the frequency and the higher is the pitch.
Pitch of sound produced by children is greater than the pitch of sound produced by
adults.

Fig. 9.56 Loudness : It is the sensation produced on the ears. It depends on the amplitude of the
sound wave. The sound produced by greater energy has larger amplitude and therefore
more loudness. The following figure represents wave shapes of a loud and a soft sound
of the same frequency.
Musical sound: A sound which produces pleasing effect on the ears is called musical
sound.
Musical interval: The ratio of the frequencies of two notes is called musical interval.
Two notes with musical interval 1:1 are called unision. Some other common musical
intervals are;
(a) Octave (1: 2) (b) majortone (8: 9 )
Fig. 9.57 (c) minortone (9: 10) (d) semitone(15 : 16)
WAVE - I 607
Musical scale: A series of notes arranged such that their fundamental frequencies have
definite ratios is called a musical scale. The most widely used musical scale is diatonic
scale. It consists of eight notes, called sargam. The first note of frequency 256 Hz is
called keynote or fundamental and last note of frequency 512 Hz is an octave of the first
note.
Indian name: Sa Re Ga Ma Pa Dha Ni Sa
1 2
Frequency 341 426
(Hz): 256 288 320
3
384
3
480 512
Intervals: 9/8 10/9 6/15 9/8 10/9 9/8 16/15
Quality or timbre:
Quality of sound distinguishes one sound from another having the same pitch and
loudness. The sound which is more pleasant is said to be a rich quality. The sounds of
sitar and violin can be distinguished by their quality.
9.27 REFLECTION OF SOUND
When wave strikes to any boundary, it reflected back into the same medium. The wave is
reflected in such a way that angle of reflection is equal to the angle of incidence of the
wave. If i and r are the angle of incidence and angle of reflection, then by law of reflection,
we have
i = r.
Echo : Fig. 9.58
If we clap in front of a large wall or hill, we will hear the same sound again a little later .
This repetition of sound after reflection from large obstruction is called echo. The sensation
of sound persists in our brain nearly for 0.1s. To hear an echo the time interval between
original sound and the reflected sound must be at least 0.1s.
Let x be minimum distance between source of sound and obstruction. If v is the speed of
sound in air, then time taken by sound to reach the obstruction and back to the listner is
2x
t =
v
At room temperature, v =340 m/s
2x
t =
340
To hear echo, Fig. 9.59
2x
= 0.1
340
0.1 340
or x =
2
or x = 17m
9.28 REVERBERATION
The sound created in a big hall will persist by repeated reflections from the walls and roof
until it is reduced to a value where it is no longer audible. The persistence of audible
sound after the source has ceased to emit sound is called reverberation.
In a picture hall or in auditorium, excessive reverberation causes overlapping of sounds
and therefore is highly undesirable. To control reverberation, the walls and roof of the
structure are generally covered with sound absorbing materials like, fiberboard, rough
plaster etc.
Reverberation time :
The time for which sound persists after the source has stopped producing sound is
called reverberation time (T).
Fig. 9.63
608 MECHANICS, HEAT, THERMODYNAMICS & WAVES
According to Sabine law, the reverberation of a hall is given by

0.16V
T=
ai si

Where V is the volume of the hall and aisi is the total absorption of the hall. Here s and
a are the surface area and absorption coefficient respectively.
Method for controlling reverberation time
(i) Covering walls and doors with sound absorbent materials like asbestos, card
board etc.
(ii) Providing rough texture paint on the walls.
(iii) Providing curtains on the doors and windows
(iv) By increasing number of audience.
(v) Floor with rough tiles.
Acoustical requirements of a building
These are as follows:
(i) There should be no echo.
(ii) The reverberation time neither be too low nor too high. For lecture room, it is 2
second.
(iii) There should no extraneous sound in the building.
(iv) The total quality of sound should not be altered.

9.29 RANGE OF HEARING


Normal human ears can hear the sound of frequency 20 Hz to 20000 Hz. Sound of
frequency less than 20 Hz is called infrasonic . Sound of frequency greater than 20000 Hz
is called ultrasound. Children under the age of five and dogs, owls can hear upto 25 kHz.
Whales and elephants produce sound in the infrasonic range. Rhinoceroes make
communication between themselves by using a frequency as low as 5 Hz.

9.30 ULTRASOUND
Frequencies higher than 20000 Hz are called ultrasound. Ultrasound can be produced by
Galton’s whistle. Some animals, such as dolphins, porpoises can produced ultrasound.
Bats can produce and hear ultrasound.
Applications of Ultrasound
On being high frequency waves, ultrasound possesses high intensity, and therefore can
penetrate any solid or liquid medium.
1. Ultrasound can kill bacteria and therefore can be used for water purification.
2. To detect cracks in metal and in thick walls: Ultrasound can be used to detect
cracks in walls of huge structure like atomic power plant. The cracks or holes
inside the metal blocks or RCC walls which are invisible from outside reduces the
strength of the structure.
Ultrasonic waves are allowed to pass through the walls and detectors are used to
detect the transmitted waves .If there is a crack in the wall, the ultrasound gets
reflected back indicating the presence of defect (see Fig. 9.64).
Fig. 9.64 3. Echocardiography:
By making ultrasound of some specific intensity, these are made to reflect from
WAVE - I 609
various parts of the heart and form its image. This technique is known as
echocardioigraphy.
4. Ultrasound may be used to break stones formed in the kidney. The crushed stone
later get flushed out with urine.
5. Sonography:
Ultrasonography is used for examination of the factor during frequency to detect
congenial defects and growth abnormalities.
9.31 SONAR
SONAR stands for SOund Navigation And Ranging. SONAR is a device which
is used to find depth of sea or to detect the position of submarine hidden inside
water. Sonar consists of a transmitter and a detector. They are installed in a ship
(see Fig. 9.65).

Fig. 9.65. Ultrasound sent by the transmitter and received by the detector.
The transmitter produces ultrasonic waves and transmit them. These waves propagate
through water and after striking from the object inside water, get reflected back and are
recorded by the detector. The distance of the object (submarine etc) can be calculated
by knowing the speed of sound in water and the time interval between transmission and
reception of the ultrasound in water . The total distance travelled by ultrasound is 2d.
2d = v × t
vt
or d =
2
9.32 SHOCK WAVES
If an object moves with a speed greater than the speed of sound, then it is called
supersonic. Mach studied such objects and introduced a dimensionless parameter, called
Mach number, which is defined as:
speed of object
Mach number =
speed of sound

v0
or M =
v
M = 2 means, the speed of object is twice the speed of sound. As speed of sound in air
is nearly 1200 km/h, so v0 = 2400 km/h. When such an object moves , it produces
energetic disturbance (wave front) in the backward direction of motion of the object .
The wave front extend in three dimensions and forms a cone called the Mach cone. A
shock wave is said to exist along the surface of this cone. Shock waves are too energetic,
Fig. 9.66. Shock waves
610 MECHANICS, HEAT, THERMODYNAMICS & WAVES
they can break the glass panels or can damage the buildings. Figure shows a shock
wave and the half cone angle called the Mach cone angle, is given by

vt 1
sin = = (Mach cone angle)
v0 t M

1 1
or = sin
M

Ex. 41 Two men are equi distance from the face of a plane
vertical cliff and are 300 m apart. One of them fires a pistol, the 5
x = 30 1 = 8.33 m
other hears the echo one second after hearing the direct sound. 18
The velocity of sound is 330 m/s . Calculate the distance of the The distance travelled by sound in 1 second
man from the cliff.
2 = 330 × 1 = 330 m
Sol. Let two men are standing at A and B, and the distance of each man
= 165 m
from the cliff is x.
2
2 x
From the figure y =
2

2
8.33
= 1652
2
= 164.95 m
Fig. 9.60 Thus the distance between two rows of the buildings 2y = 329.9 m
The time taken by direct sound from A to B, Ans.
Ex. 43 Why is the sound produced in air not heard by a person
300
t1 = s deep inside the water?
330
Sol.
The time taken by the reflected sound
The speed of sound in water is nearly four times the speed of sound in
2x air. From Snell’s law
t2 =
330

According to the given condition


t2 – t1 = 1

2x 300
or = 1 or x = 315 m Ans.
330 330

Ex. 42 A road runs between two parallel rows of buildings. A Fig. 9.62
motorist moving just in the middle with a velocity of 30 km/h,
sin i va 1
sounds the horn. He hears an echo one second after sounding the
a µw =
sin r
= v =
4
= 0.25
horn. Find the distance between the two rows of the buildings. The w
velocity of sound = 330 m/s .
Critical angle c = sin 1 ( µ) = sin 1 (0.25) = 14°.
Sol.
Thus most of the sound produced in air and incident at an angle i 14
Suppose 2 y is the distance between two rows of the buildings. The
distance travelled by car in 1 second gets reflected back in air and very small amount is refracted into water.
Hence a person inside water cannot hear the sound produced in air.

Fig. 9.61
WAVE - I 611

Review of formulae & Important Points


1. The velocity of wave in a medium is given by v f 7. Speed of transverse wave in stretched string

2. Equation of plane progressive wave, y A sin( t kx) F stress


v
Y
8. Power transmitted
1 2 2
Pav vS A
2
9. Intensity of wave
P 1
I v 2 A2 2 2 f 2 A2 v
S 2
10. Variation of intensity with distance
Above equation represents displacement of the particles at a 1
(i) For point source, I
distance x from the origin as time passes, and y A sin( kx t ), r2
represents the displacements of all the particles of the wave at
any time. In general we can write, 1
(ii) For line source, I
y A sin( t kx r
0)
11. Sound level : The decibel scale
If at t = 0, x = 0, y = y0, then
sin 1 ( y0 A) . I
0 10log decibel
I0
y
I2
+A and 2 1 10log
I1

O x 12. Doppler effect in sound


/4 /2 (i) When source moves towards stationay observer
3 /4
–A v
f' f
v vs
3. Other equations of wave
(ii) When observer moves towards stationary source
2
(i) y A sin (vt x)
v v0
f' f , '
v
t x
(ii) y A sin 2 In general,
T
2 2 v v0
y y f' f
Differential equation of a wave : v2 v vs
t2 x2
4. Phase difference, 13. Doppler effect in light
(i) Between two particles at any time f v
2 f c
x
where v relative speed between source and observer.
(ii) Between two times of a particle c speed of light.
2 14. Sound of frequency greater than 20000 Hz is called ultrasound.
t Ultrasound can be used for navigation and sonography.
T
5. Sound waves or pressure waves 15. Mach number

P Pm sin(kx t ), Pm ABk Speed of the object(v0 )


M
6. Speed of sound waves in air Speed of the sound(v)
Mach angle is defined as :
P
v , for air = 1.4. 1
sin
M
RT
Also v
M
612 MECHANICS, HEAT, THERMODYNAMICS & WAVES

MCQ Type 1 Exercise 9.1


Level -1
Only one option correct 8. The distance between two consecutive crests in a wave train
1. Ultrasonic, infrasonic and audible waves travel through a medium produced in a string is 5 cm. If 2 complete waves pass through any
with speeds vu, vi and va respectively, then point per second, the velocity of the wave is
(a) vu, vi and va are equal (b) vu va vi (a) 10 cm/s (b) 2.5 cm/s
(c) vu va vi (d) va vu and vu = vi (c) 5 cm/s (d) 15 cm/s
2. A tuning fork of frequency 480 Hz is used to vibrate a sonometer 9. When sound waves travel from air to water, which of the following
wire having natural frequency 410 Hz. The wire will vibrate with remains constant ?
a frequency (a) Velocity (b) Frequency
(a) 410 Hz (b) 480 Hz (c) Wavelength (d) All the above
(c) 820 Hz (d) 960 Hz 10. The frequency of a sound wave is f and its velocity is v. If the
3. Figure shows the displacement of a string element located at, say, frequency is increased to 4 f, the velocity of the wave will be
x = 0 as a function of time.The time moment(s) at which element
(a) v (b) 2 v
is moving downward
y v
(c) 4v (d)
4
11. What will be the wave velocity, if the radar gives 54 waves per min
b c
and wavelength of the given wave is 10 m
t
a d (a) 4 m/s (b) 6 m/s
(c) 9 m/s (d) 5 m/s
(a) a (b) b 12. Velocity of sound in air
(c) c (d) d I. Increases with temperature.
4. The following four waves are sent along strings (x in metre and t in II. Decreases with temperature.
second). Which of them is moving fastest ? III. Increases with pressure.
(1) y1 = (1 mm) sin (x – 3t) (2) y2 = (2 mm) sin (4x – t) IV. Is independent of pressure.
(3) y3 = (3 mm) sin (2x – t) (4) y4 = (4 mm) sin (x – 2t) V. Is independent of temperature.
(a) (1) (b) (2) Choose the correct answer
(c) (3) (d) (4)
(a) Only I and II are true (b) Only I and III are true
5. Which of the following statements is wrong ?
(c) Only II and III are true (d) Only I and IV are true
(a) Sound travels in straight line
13. v1 and v2 are the velocities of sound at the same temperature in
(b) Sound is a form of energy
two monoatomic gases of densities 1 and 2 respectively. If
(c) Sound travels in the form of waves
(d) Sound travels faster in vacuum than in air 1 1
= , then the ratio of velocities v1 and v2 will be
6. What is the wavelength of the wave shown in figure, where each 2 4
segment of the wave has length b (a) 1 : 2 (b) 4 : 1
(c) 2 : 1 (d) 1 : 4
14. The temperature at which the speed of sound in air becomes
double of its value at 0°C is
b (a) 273 K (b) 546 K
(a) 4 b (b) 5 b (c) 1092 K (d) 0 K
(c) 6 b (d) 7 b 15. Water waves are
7. A man sets his watch by a whistle that is 2 km away. How much (a) Longitudinal
will his watch be in error (speed of sound in air 330 m/s) (b) Transverse
(a) 3 s fast (b) 3 s slow (c) Both longitudinal and transverse
(c) 6 s fast (d) 6 s slow (d) Neither longitudinal nor transverse

Answer Key 1 (a) 3 (c) 5 (d) 7 (d) 9 (b) 11 (c) 13 (c) 15 (c)
Sol. from page 624 2 (b) 4 (a) 6 (d) 8 (a) 10 (a) 12 (d) 14 (c)
WAVE - I 613
16. Sound travels in rocks in the form of 25. A wave of frequency 400 Hz has a phase velocity of 300 m/s. Two
(a) Longitudinal elastic waves only points on this wave are out of phase by 60°. The separation
(b) Transverse elastic waves only between these two points is
(a) 1.25 cm (b) 12.5 cm
(c) Both longitudinal and transverse elastic waves
(c) 0.25 cm (d) 2.25 cm
(d) Non-elastic waves
26. A whistle giving out 450 Hz approaches a stationary observer at
17. The waves in which the particles of the medium vibrate in a direction
a speed of 33 m/s. The frequency heard by the observer in Hz is
perpendicular to the direction of wave motion is known as
(a) 409 (b) 429
(a) Transverse waves (b) Longitudinal waves
(c) 517 (d) 500
(c) Propagated waves (d) None of these
27. A whistle of frequency 500 Hz tied to the end of a string of length
18. The rate of transfer of energy in a wave depends 1.2 m revolves at 400 rev/min. A listener standing some distance
(a) Directly on the square of the wave amplitude and square of away in the plane of rotation of whistle hears frequencies in the
the wave frequency range (speed of sound = 340 m/s) :
(b) Directly on the square of the wave amplitude and root of the (a) 436 to 586 (b) 426 to 574
wave frequency (c) 426 to 584 (d) 436 to 674
(c) Directly on the wave amplitude and square of the wave 28. A train moves towards a stationary observer with speed 34 m/s.
frequency The train sounds a whistle and its frequency registered by the
(d) None of these observer is f1. If the train’s speed is reduced to 17 m/s, the frequency
19. Which of the following is not the transverse wave ? registered is f2. If the speed of sound is 340 m/s, then the ratio
(a) X - rays (b) - rays f1 / f2 is
(c) Visible light wave (d) Sound wave in a gas 18 1
(a) (b)
20. Sound waves of wavelength greater than that of audible sound are 19 2
called
19
(a) Seismic waves (b) Sonic waves (c) 2 (d)
18
(c) Ultrasonic waves (d) Infrasonic waves
21. Oxygen is 16 times heavier than hydrogen. Equal volumes of 29. A small source of sound moves on a circle as shown in the figure
hydrogen and oxygen are mixed. The ratio of the velocity of sound and an observer is standing on O. Let f1, f2 and f3 be the frequencies
in the mixture to that in oxygen is heard when the source is at A, B and C respectively. Then

32 17 B
(a) (b) C
17 32

1 A
(c) 8 (d)
8 (a) f1 > f2 > f3 (b) f2 > f3 > f1
22. ‘SONAR’ emits which of the follwoing waves (c) f1 = f2 > f3 (d) f2 > f1 > f3
(a) Radio waves (b) Ultrasonic waves 30. Intensity level of a sound of intensity I is 30 dB. The ratio I / I0 is
(c) Light waves (d) Magnetic waves (where I0 is the threshold of hearing )
23. A travelling wave in a stretched string is described by the equation (a) 3000 (b) 1000
y = A sin (kx - t). The maximum particle velocity is (c) 300 (d) 30
31. Each of the properties of sound listed in column A primarily
(a) A (b) depends on one of the quantities in column B. Choose the matching
k
pairs from two columns
d x Column A Column B
(c) (d)
dk t Pitch Waveform
24. The displacement y of a wave travelling in the x-direction is given Quality Frequency
Loudness Intensity
by y = 10–4 sin (600 t – 2 x + ) metre, where x is expressed in
3 (a) Pitch-waveform, Quality-frequency, Loudness- intensity
metre and t in second. The speed of the wave-motion in m/s is
(b) Pitch-frequency, Quality-waveform, Loudness- intensity
(a) 200 (b) 300
(c) Pitch-intensity, Quality-waveform, Loudness- frequency
(c) 600 (d) 1200
(d) Pitch-waveform, Quality-intensity, Loudness- frequency

Answer Key 16 (c) 18 (a) 20 (d) 22 (b) 24 (b) 26 (d) 28 (d) 30 (b)
Sol. from page 624 17 (a) 19 (d) 21 (a) 23 (a) 25 (b) 27 (a) 29 (b) 31 (b)
614 MECHANICS, HEAT, THERMODYNAMICS & WAVES
32. A star is moving away from the earth with a velocity of 100 km/s. 37. A transverse wave propagating in a string is decribed by the equation
If the velocity of light is 3 x 108 m/s, then the shift of its spectral y = 0.021 sin(x + 30t), where x and y are in metre and t in second.
line of wavelength 5700 Å due to Doppler’s effect will be If the linear density of the vibrating string is 1.3 × 10–4 kg/m, the
(a) 0.63 Å (b) 1.90 Å tension in the string is
(c) 3.80 Å (d) 5.79 Å (a) 0.21 N (b) 0.12 N
33. Two sound waves having a phase difference of 60° have path
(c) 0.64 N (d) 2.1 N
difference of
(a) 2 (b) /2 38. The phase difference between two waves represented by
(c) /6 (d) /3 y1 = 10–6 sin [100t + (x/50) + 0.5]m
34. A wave of frequency 500 Hz has velocity 360 m/sec. The distance y2 = 10–6 cos [100t + (x/50)]m
between two nearest points 60° out of phase, is
where x is expressed in metre and t is expressed in second, is
(a) 0.6 cm (b) 12 cm
approximately.
(c) 60 cm (d) 120 cm
35. On increasing the tension of a stretched string by 2.5 N, the (a) 1.5 rad (b) 1.07 rad
frequency is altered in the ratio 3 : 2. The original stretching force is (c) 2.07 rad (d) 0.5 rad
(a) 6 N (b) 2 N 39. A motor cycle starts from rest and accelerates along a straight path
(c) 4 N (d) 5 N at 2m/s2. At the starting point of the motor cycle there is a stationary
36. The particles of a medium vibrate about their mean positions electric siren. How far has the motor cycle gone when the driver
whenever a wave travels through that medium. The phase difference hears the frequency of the siren at 94% of its value when the
between the vibrations of two such particles motor cycle was at rest (Speed of sound = 330 ms–1)
(a) Varies with time
(a) 49 m (b) 98 m
(b) Varies with distance separating them
(c) Varies with time as well as distance (c) 147 m (d) 196 m
(d) Is always zero

Answer Key 32 (b) 34 (b) 36 (b) 38 (b)


Sol. from page 624 33 (c) 35 (b) 37 (b) 39 (b)

Level -2
1. A transverse wave is described by the equation (b) A wave travelling in the negative x direction with a velocity
of 1.5 m/s.
x
y = y0 sin 2 ft .The maximum particle velocity is four (c) A wave travelling in the negative x direction with a wavelength
of 0.2 m.
times the wave velocity if (d) A wave travelling in the positive x direction with a wavelength
of 0.2 m.
y0 y0
(a) = (b) = 4. A siren placed at a railway platform is emitting sound of frequency
4 2
5 kHz. A passenger sitting in a moving train A records a frequency
(c) = y0 (d) = 2 y0 of 5.5 kHz while the train approaches the siren. During his return
2. Which one of the following does not represent a travelling wave ? journey in a different train B the records a frequency of 6.0 kHz
(a) y = sin (x – vt) (b) y = ym sin k (x + vt) while approaching the same siren. The ratio of the velocity of train
B to that of train A is
(c) y = ym log (x – vt) (d) y = f (x2 – vt2)
3. A wave represented by the given equation 242
(a) (b) 2
252
y = A sin (10 x + 15 t + ), where x is in metre and t is in second.
3 5 11
(c) (d)
The expression represents 6 6
(a) A wave travelling in the positive x direction with a velocity
of 1.5 m/s.

Answer Key (b) (d) (b) (c)


1 2 3 4
Sol. from page 625
WAVE - I 615
5. A train has just completed a U-curve in a track which is a semicircle. 11. When a longitudinal wave propagates through a medium, the
The engine at the forward end of the semicircular part of the track particles of the medium execute simple harmonic oscillations about
while the last carriage is at the rear end of the semicircular track. their mean positions. These oscillations of a particle are
The driver blows a whistle of frequency 200 Hz. Velocity of characterised by an invariant
sound is 340 m/s. Then the apparent frequency as observed by a
(a) Kinetic energy
passenger in the middle of a train when the speed of the train is
30 m/s is (b) Potential energy
(c) Sum of kinetic energy and potential energy
(a) 209 Hz (d) Difference between kinetic energy and potential energy
12. The amplitude of a wave disturbance propagating in the positive
(b) 288 Hz
1 1
x-direction is given by y at t = 0 and y at
(c) 200 Hz 1 x2 1 x –1
2

(d) 181 Hz t = 2s, where x and y are in metre. The shape of the wave disturbance
does not change during the propagation. The velocity of the wave
6. The displacement-time graphs for two sound waves A and B are is
shown in the figure. Then the ratio of their intensities IA / IB is (a) 0.25 m/s (b) 2.5 m/s
equal to
y (c) 0.5 m/s (d) 5 m/s
(a) 1 : 4 A 13. A person speaking normally produces a sound intensity of 40 dB
2
B at a distance of 1 m. If the threshold intensity for reasonable
1
(b) 1 : 16 audibility is 20 dB, the maximum distance at which he can be
0 t heard clearly is
(c) 1:2 -1 (a) 4 m (b) 5 m
(c) 10 m (d) 20 m
-2
(d) 1:1 14. The rope shown at an instant is carrying a wave travelling towards
right, created by a source vibrating at a frequency f. Consider the
7. In a plane progressive harmonic wave particle speed is always less following statements
than the wave speed if

(a) Amplitude of wave is less than .


2

(b) Amplitude of wave is greater than . a b c d e


2
(c) Amplitude of wave is less than

(d) Amplitude of wave is greater than . I. the speed of the wave is 4 f × ab


8. The ratio of intensities between two coherent sound sources is 4
4 : 1. The difference of loudness in decibels (dB) between maximum II. the medium at a will be in the same phase as d after s
3f
and minimum intensities, when they interfere in space is
(a) 10 log (2) (b) 20 log (3) 3
(c) 10 log (3) (d) 20 log (2) III. the phase difference between b and e is
2
9. Two men are equidistance from the phase of a plane vertical cliff
and are 300 m apart. One of them fires a pistol, the other hears the Which of these statements are correct ?
echo one second after hearing the direct sound. The velocity of (a) I, II and III (b) II only
sound is 330 m/s. The distance of the men from the cliff is (c) I and III (d) III only
(a) 300 m (b) 415 m 15. A uniform rope of length 12m and mass 6 kg hangs vertically from
(c) 350 m (d) 315 m a rigid support. A block of mass 2kg is attached to the free end of
10. A source of sound of frequency 600 Hz is placed inside water. The
the rope. A transverse pulse of wavelength 0.06m is produced at
speed of sound in water is 1500 m/s and in air is 300 m/s. The
the lower end of the rope. The wavelength of the pulse when it
frequency of sound recorded by an observer who is standing in air is
reaches the top of the rope is
(a) 200 Hz (b) 3000 Hz
(c) 120 Hz (d) 600 Hz (a) 0.012 m (b) 0.06 m
(c) 0.24 m (d) 0.12 m

Answer Key 5 (c) 7 (a) 9 (d) 11 (c) 13 (c) 15 (d)


Sol. from page 625 6 (d) 8 (b) 10 (d) 12 (c) 14 (c)
616 MECHANICS, HEAT, THERMODYNAMICS & WAVES

MCQType 2 Exercise 9.2


1. The intensity of a progressing plane wave in loss-free medium is 8. A mixture of two diatomic gases exists in a closed cylinder. The
(a) directly proportional to the square of amplitude of the wave volumes and velocities in the two gases are V1, V2 and c1, c2
(b) directly proportional to the velocity of the wave respectively. 1 and 2 are the densities of the two gases. Then
(c) directly proportional to the square of frequency of the wave
(d) inversely proportional to the density of the medium 1V2 2V1
2. A transverse sinusoidal wave of amplitude a, wavelength and (a) the density of the mixture of gases is
1 2
frequency f is travelling on a stretched string. The maximum speed
of any point on the string is v/10, where v is the speed of
1V1 2V2
propagation of the wave. If a = 10–3 m and v = 10 ms–1, then (b) the density of the mixture of gases is V1 V2
and f are given by
(a) = 2 × 10–2 m (b) = 10–3 m
V1 V2
103 (c) the velocity of sound in the mixture is c1c2
(c) f Hz (d) f = 104 Hz V1c22 V2c12
2
3. A sound wave of frequency f travels horizontally to the right. It is
V1c12 V2c22
reflected from a large vertical plane surface moving to the left with (d) the velocity of sound in the mixture is c1c2
a speed v. The speed of sound in the medium is c, then V1 V2

f (c v) 9. In the figure shown an observer O1 floats (static) on water surface


(a) the frequency of the reflected wave is with ears in air while another observer O2 is moving upwards with
c v
constant velocity v1 = v/5 in water.
c ( c v) The source moves down with constant velocity vs = v/5 and
(b) The wavelength of the reflected wave is f (c v)
emits sound of frequency f.
(c) the number of waves striking the surface per second is The velocity of sound in air is v and that in water is 4v. For the
situation shown in figure
f (c v)
c S
(d) the number of beats heard by a stationary listener to the left
fv O1 vs= v/5
of the reflecting surface is
c v
4. In a wave motion y = a sin (kx – t), y can represent O2 v1= v/5
(a) electric field (b) magnetic field
(c) displacement (d) pressure (a) The wavelength of the sound received by O1 is 4v/ 5f
5. The equation y = 4 + 2 sin (6t – 3x ) represents a wave motion with
(b) The wavelength of the sound received by O1 is v/ f
(a) amplitude 6 units (b) amplitude 2 units
(c) The frequency of the sound received by O2 is 21f / 16
(c) wave speed 2 units (d) wave speed 1/2 units
6. In a plane progressive harmonic wave (d) The wavelength of the sound received by O2 is 16v/ 5f
(a) phase difference between displacement and acceleration of 10. A stationary observer receives a sound of frequency f0 = 2000 Hz.
particle is zero The apparent frequency f varies with time as shown in figure.
(b) phase difference between displacement and acceleration of Speed of sound = 300 m/s. Choose the correct alternative (s)?
particle is
f(Hz)
(c) phase difference between displacement and velocity of
particle is /2
(d) phase difference between velocity and acceleration of particle fm
is /2 2000
7. The speed of sound in a gas 1800
(a) doesn't depend on the pressure of the gas t(s)
(b) varies directly with the square root of th e absolute
temperature of the gas (a) speed of source is 66.7 m/s
(c) varies inversely with the square root of the density of the (b) fm shown in figure cannot be greater than 2500 Hz
gas (c) speed of source is 33.33 m/s
(d) changes with the change in frequency of the sound wave (d) fm shown in figure cannot be greater than 2250 Hz

Answer Key 1 (a, b, c) 3 (a, b, c) 5 (b, c) 7 (a, b, c) 9 (a, c, d)


Sol. from page 626 2 (a, c) 4 (a, b, c, d) 6 (b, c, d) 8 (b, c) 10 (c, d)
WAVE - I 617
11. Choose the correct option (s) ? 12. As a wave propagates
(a) When a source of sound moves towards a stationary observer, (a) the wave intensity remains constant for a plane wave.
the wavelength of the sound as heard by the observer is less (b) the wave intensity decreases as the inverse of the distance
than the original wavelength of the source. from the source for a spherical wave.
(b) When both observer and the source of sound moves towards (c) the wave intensity decreases as the inverse square of the
each other, the wavelength of the sound as heard by the distance from the source for a spherical wave.
observer is less than the wavelength of the original sound. (d) total intensity of the spherical wave over the spherical surface
(c) When both observer and the source of sound moves away centered at the source remains constant at all times.
from each other, the wavelength of the sound as heard by the
observer is less than the wavelength of the original sound.
(d) When an observer moves away from a stationary source, the
wavelength of the sound heard by the observer is less than
the wavelength of the original sound.

Answer Key
11 (a, b, c) 12 (c, d)
Sol. from page 626

Statement Questions Exercise 9.3


Read the two statements carefully to mark the correct option out of the options given below:
(a) If both the statements are true and the statement - 2 is the correct explanation of statement - 1.
(b) If both the statements are true but statement - 2 is not the correct explanation of the statement - 1.
(c) If statement - 1 true but statement - 2 is false.
(d) If statement - 1 is false but statement - 2 is true.

1. Statement 1 6. Statement 1
Transverse wave are not produced in fluids. Sound travels faster on a hot summer day than on a cold winter
Statement 2 day.
Fluid possess no rigidity. Statement 2
2. Statement 1 Velocity of sound is directly proportional to the square root of its
Sound wave can not propagate through vacuum but light waves absolute temperature.
can. 7. Statement 1
Statement 2 The base of Laplace correction was that exchange of heat between
Sound waves cannot be polarised but light waves can be polarised. the region of compression and rarefaction in air is not possible.
3. Statement 1 Statement 2
Particle velocity and wave velocity both are independent of time. Air is a bad conductor of heat and velocity of sound in air is large.
Statement 2 8. Statement 1
For the propagation of wave motion, the medium must have the Compression and rarefaction involve changes in density and
properties of elasticity and inertia. pressure.
4. Statement 1 Statement 2
The change in air pressure affect the speed of sound. When particles are compressed, density of medium increases and
Statement 2 when they are rarefied, density of medium decreases.
The speed of sound in a gas is proportional to the square root of 9. Statement 1
pressure. In the case of a stationary wave, a person hear a loud sound at the
5. Statement 1 nodes as compared to the antinodes.
The speed of sound in solids is maximum though their density is Statement 2
large. In a stationary wave all the particles of the medium vibrate in
Statement 2 phase.
The modulus of elasticity of solid is large.

Answer Key 1 (a) 3 (d) 5 (a) 7 (c) 9 (c)


Sol. from page 627 2 (b) 4 (d) 6 (a) 8 (a)
618 MECHANICS, HEAT, THERMODYNAMICS & WAVES

Passage & Matrix Exercise 9.4

Passage for (Q. 1 - 3) : 5. The distribution of the sound intensity of the whistle as observed
Answer the following questions using the informations given below. by the passengers in train A is best represented by
Molecular weight of air = 28.8
Molecular weight of water vapour = 18

Intensity
of dry air = 1.4
(a)
of water vapour = 1.33
Standard pressure = 760 mm of Hg
Standard temperature = 0°C f1 f 2 Frequency
Vapour pressure at 0°C = 4.8 mm of Hg
Velocity of sound in air at STP = 332 m/s

Intensity
1. If m and d be the densities of the moist and dry air respectively, (b)
then
(a) m= 0.251 d (b) m = 0.997 d f1 f 2 Frequency
(c) m = 0.755 d (d) m = 0.355 d.
2. If m and d be the adiabatic exponent for moist and dry air
Intensity
respectively then
(c)
m m
(a) 0.99 (b) 0.95
d d f1 f 2 Frequency

m m
0.92 0.75
Intensity

(c) (d)
d d
3. The speed of sound in moist air at STP is (d)
(a) 329.5 m/s (b) 330.25 m/s
(c) 331.7 m/s (d) 333.7 m/s f1 f 2 Frequency

Passage for (Q. 4 - 6) : 6. The spread of frequency as observed by the passengers in train B
Two trains A and B are moving with speeds 20 m/s and 30 m/s respectively is
in the same direction on the same straight track, with B ahead of A. The (a) 310 Hz (b) 330 Hz
engines are at the front ends. The engine of train A blows a long whistle. (c) 350 Hz (d) 290 Hz
Assume that the sound of the whistle is composed of components varying Passage for (Q. 7 - 9) :
in frequency from f1 = 800 Hz to f2 = 1120 Hz, as shown in the figure. The
A train standing at the outer signal of a railway station blows a whistle of
spread in the frequency (highest frequency – lowest frequency) is thus
frequency 400 Hz in still air.
320 Hz. The speed of sound in still air is 340 m/s.
7. What is the frequency of the whistle for a platform observer when
the train approaches the platform with a speed of 10 m/s
Intensity

(a) 300 Hz (b) 412.12 Hz


(c) 350 Hz (d) 360.50 Hz

8. What is the frequency of the whistle for a platform observer when


f1 f 2 Frequency
the train recedes from the platform with a speed of 10 m/s ?
4. The speed of sound of the whistle is (a) 400.60 Hz (b) 528.75 Hz
(a) 340 m/s for passengers in A and 310 m/s for passengers in B (c) 574.65 Hz (d) none of these
(b) 360 m/s for passengers in A and 310 m/s for passengers in B 9. What is the frequency of sound in each case ?
(c) 310 m/s for passengers in A and 360 m/s for passengers in B (a) 340 Hz (b) 350 Hz
(d) 340 m/s for passengers in both the trains (c) 360 Hz (d) 370 H

Answer Key 1 (b) 3 (c) 5 (a) 7 (b) 9 (a)


Sol. from page 627 2 (a) 4 (b) 6 (a) 8 (c)
WAVE - I 619
Passage for (Q. 10 - 12) : Passage for (Q. 13 - 15) :
A heavy but uniform rope of length L is suspended from a ceiling. A long wire PQR is made by joining two wires PQ and QR of equal radii.
PQ has length 4.8 m and mass 0.06 kg. QR has length 2.56 m and mass
10. Write the velocity of a transverse wave travelling on the string as 0.2 kg. The wire PQR is under a tension of 80 N. A sinusoidal wave–
a function of the distance from the lower end. pulse of amplitude 3.5 cm is sent along the wire PQ from the end P. No
(a) gx (b) g/x power is dissipated during the propagation of the wave pulse. Calculate
13. The time taken by the wave–pulse to reach the other end R of the
(c) 2gx (d) none of these
wire.
11. If the rope is given a sudden sideways jerk at the bottom, how long (a) 0.145 (b) 0.245
will it take for the pulse to reach the ceiling ? (c) 0.450 (d) 0.600
(a) 4L / g (b) 2L / g 14. The amplitude of the reflected wave–pulses after the incident
wave–pulse crosses the joint Q is
(c) L / 2g (d) L / 3g (a) 1.5 cm (b) 2.5 cm
12. A particle is dropped from the ceiling at the instant the bottom end (c) 3.0 cm (d) 4.5 cm
is given the jerk. Where will the particle meet the pulse ? 15. The amplitude of the transmitted wave–pulses after the incident
(a) L / 3 (b) L / 2 wave–pulse crosses the joint Q.
(c) L / 4 (d) L / 5 (a) 1.0 cm (b) 1.5 cm
(c) 2.0 cm (d) 2.5 cm

16. Match the columns I and II


Column I Column II
A. y = 4 sin (5x – 4t) + 3 cos (4t – 5x + /6) (p) Particles at every position are performing SHM
x x
B. y 10cos t sin (100) t (q) Equation of travelling wave
330 330
C. y = 10 sin (2 x – 120t) + 10 cos (120t + 2 x) (r) Equation of standing wave
D. y = 10 sin (2 x – 120t) + 8 cos (118t – 59/30 x) (s) Equation of Beats
17.. In the equation, y = A sin 2 (ax + bt + /4) match the following:
Column I Column II
A. Frequency of wave (p) a
B. Wavelength of wave (q) b
1
C. Phase difference between two points distance apart (r)
4a
D. Phase difference of a point after a (s) /2
1
time interval of (t) none
8b
18. Regarding speed of sound in gas match the following
Coloumn I Coloumn II
A. Temperature of gas is made 4 times and pressure 2 times (p) speed becomes 2 2 times

B. Only pressure is made 4 times without change in temperature (q) speed becomes 2 times
C. Only temperature is changed to 4 times (r) speed remains unchanged
D. Molecular mass of the gas is made 4 times (s) speed remains half
19. Source has frequency f. Source and observer both have same speed. For the apparent frequency observed by observer match the following
Column -1 Column -2
A. Observer is approaching the source but source is receding (p) more than f
from the observer
B. Observer and source both approaching towards each other (q) less than f
C. Observer and source both receding from each other (r) equal to f
D. Source is approaching but observer is receding

Answer Key 10 (a) 12 (a) 15 (c) 16 A (p, q); B (s); C (p, r); D (s) 18 A (q); B (r); C (q); D (s)
Sol. from page 627 11 (a) 13 (a) 17 A (q); B (t); C (s); D (t) 19 A (r); B (p);C (q);D (r)
620 MECHANICS, HEAT, THERMODYNAMICS & WAVES

Subjective Integer Type Exercise 9.5


Solutions from page 629
1. Speed of sound in air is 332 m/s at STP. What will be its value in
hydrogen at STP, if density of hydrogen at STP is 1/16th that of
air ?
Ans. 1328 m/s.
2. A gas is a mixture of two parts by volume of hydrogen and one
part by volume of nitrogen. If the velocity of sound in hydrogen at
0°C is 1300 m/s, find the velocity of sound in the gaseous mixture
at 27°C.
Ans. 591 m/s. Ans. 680 Hz.
3. The sirens of two fire engines have a frequency of 600 Hz each. A 7. A string of length 40 cm and weighing 10 g is attached to a spring
man hears the sirens from the two engines, one approaching him at one end and to a fixed wall at the other end. The spring has a
with a speed of 36 km/h and the other going away from him at a spring constant of 160 N/m and is stretched by 1.0 cm. If a wave
speed of 54 km/h. The difference in frequency of two sirens heard pulse is produced on the string near the wall, the time it will take
83.2 1
by the man is Hg . Find the value of x. ? Take the speed of to reach the spring is . The value of x is ?
x x
sound to be 340 m/s. Ans. 20 s.
Ans. 2. Hz. 8. A wire of 9.8 × 10–3 kg mass per metre passes over a frictionless
4. A particle on a stretched string supporting a travelling wave, takes pulley fixed on the top of an inclined frictionless plane which
5.0 ms to move from its mean position to the extreme position. makes an angle of 30° with the horizontal . Masses M1 and M2 are
The distance between two consecutive particles, which are at their tied at the two ends of the wire. The mass M1 rests on the plane
mean positions Q, is 2.0 cm. Find the wave speed. and the mass M2 hangs freely vertically downwards. The whole
Ans. 2.0 m/s. system is in equilibrium.Now a transverse wave propagate along
5. An aeroplane is going towards east at a speed of 510 km/h at a the wire with a velocity of 100 m/s. Find the value of masses M1.
height of 2000 m. At a certain instant, the sound of the plane
heared by a ground observer appears to come from a point vertically
above it. Where is the plane at this instant? Speed of the sound in
air = 340 m/s.
Ans. 833 m.
6. A train approaching a railway crossing at a speed of 120 km/h
sounds a short whistle at frequency 640 Hz when it is 300 m away
from the crossing. The speed of sound in air is 340 m/s. What will Ans. M1 = 20 kg.
be the freq uency heard by a person standing on a road 9. Two sound waves have intensities 5 × 10 –10 W/m 2 and
perpendicular to the track through the crossing at a distance of 1 × 10–6 W/m2. By how many decibels is the louder sound above
400 m from the crossing ? the other ? Ans. 33 dB.
WAVE - I 621

Subjective Exercise 9.6


Solutions from page 630

1. You have learnt that a travelling wave in one dimension is 6. A displacement wave is represented by y = 0.25 × 10 –3 sin
represented by a function y = f (x, t) where x and t must appear (500 t – 0.025 x) , where y, t and x are in cm, sec and metre
in the combination x – vt or x + vt, i.e. y = f (x ± vt). Is the converse respectively. Deduce (i) amplitude (ii) period (iii) angular
true ? Examine if the following functions for y can possibly frequency, and (iv) wavelength. Also deduce the amplitude of
represent a travelling wave : paticle velocity and particle acceleration.
(i) (x – vt)2 Ans. (i) A = 0.25 × 10–3 cm (ii) 0.01257 s (iii) 500 rad/s
(ii) log [(x+ vt)/x0] (iv) 251.2 cm (v) 0.125 cm/s (vi) 62.5 cm/s2.
(iii) exp [–(x + vt) / x0] 7. One end of a long string of linear mass density 8.0 × 10–3 kg/m is
connected to an electrically driven tuning fork of frequency 256
(iv) 1 / (x + vt)
Hz. The other end passes over a pulley and is tied to a pan
Ans. Functions (i), (ii) and (iv) are not finite for all values of x containing a mass of 90 kg.The pulley end absorbs all the
and t, hence they cannot represent a travelling wave. Only function incoming energy so that reflected waves at this end have negligible
(iii) satisfies the condition to represent a travelling wave. amplitude. At t = 0, the left end (fork end) of the string x = 0 has
2. Given below are some functions of x and t to represent the zero transverse displacement (y = 0) and is moving along positive
displacement (transverse or longitudinal) of an elastic wave. y– direction. The amplitude of the wave is 5.0 cm.Write down the
State which of these represent (i) a travelling wave, (ii) a stationary transverse displacement y as function of x and t that describes the
wave or (iii) none at all : wave on the string.
(a) y = 2 cos (3x) sin (10 t) Ans. y = 0.05 sin (16.1 × 102 t – 4.84 x), x and y are in m.

(b) y = 2 x vt 8. A SONAR system fixed in a submarine operates at a frequency


40.0 kHz. An enemy submarine moves towards the SONAR with
(c) y = 3 sin (5x – 0.5t) + 4 cos (5x – 0.5t) a speed of 360 km/h. What is the frequency of sound reflected by
(d) y = cos x sin t + cos 2x sin 2t a submarine? Take the speed of sound in water to be 1450 m/s.
Ans. 45.93 kHz.
Ans. (a) stationary wave (b) it cannot represent any type of
wave (c) it represents a travelling wave of amplitude 5 unit. (d) it 9. A whistle of frequency 540 Hz rotates in a circle of radius 2 m at
represents the superposition of two stationary waves. an angular speed of 15 rad/s. What is the lowest and highest
frequency heard by a listener a long distance away at rest w.r.t.
3. Explain why or how :
centre of the circle ? Can the apparent frequency be ever equal to
(a) in a sound wave, a displacement node is a pressure antinode the actual frequency ? Take v = 330 m/s.
and vice versa
Ans. 495 Hz, 594 Hz, yes.
(b) bats can ascertain distance, directions, nature, and sizes of
10. The displacement of the particle at x = 0 of a stretched string
the obstacles without any “eyes”
carrying a wave in the positive x– direction is given by f(t) = A sin
(c) a violin note and sitar note may have the same frequency, (t / T). The wave speed is v. Write the wave equation.
yet we can distinguish between the two notes
(d) Solids can support both longitudinal and transverse waves, t x
Ans. f ( x ,T ) A sin – .
but only longitudinal waves can propagate in gases, and T vT
(e) the shape of a pulse gets distorted during propagation in a 11. A wave pulse is travelling on a string with a speed v towards the
dispersive medium. positive x – axis. The shape of the string at t = 0 is given by
Ans. (d) gases do not possess shear elasticity. g(x) = A sin (x/a), where A and a are constants.
(e) the shape of a pulse gets distorted during propagation in (a) What are the dimension of A and a?
a dispersive medium. (b) Write the equation of the wave for a general time t, if the
4. What is the ratio of the velocity of sound in hydrogen ( = 7/5) to wave speed is v.
that in helium gas ( = 5/3) at the same temperature ?
x vt
Ans. 42 / 5. Ans. (a) (L, L) (b) f (x, t) = A sin .
a
5. The equation of a plane progressive wave is
12. A wave propagates on a string in the positive x – direction at a
y = 10 sin 2 (t – 0.005 x) velocity v. The shape of the string at t = t0 is given by g(x, t0) = A
where y and x are in cm and t in second. Calculate the amplitude, sin (x/a). Write the wave equation for a general time t.
frequency, wavelength and velocity of the wave.
x – v( t – t 0 )
Ans. A = 10 cm , f = 1 Hz, = 200 cm, v = 200 cm/s. Ans. f ( x , t ) A sin .
a
622 MECHANICS, HEAT, THERMODYNAMICS & WAVES
13. Figure shows a plot of the transverse displacements of the particles 19. (a) Find the speed of sound in a mixture of 1 mole of helium and
of a string at t = 0 through which a travelling wave is passing in the 2 mole of oxygen at 27°C.
positive x– direction. The wave speed is 20 cm/s. Find (b) If the temperature is raised by 1 K from 300 K, find the
(a) the amplitude, (b) the wavelength percentage change in the speed of sound in the gaseous mixture
(c) the wave number and (d) the frequency of the wave. (R = 8.31 J/mol K).
Ans. (a) 400.9 m/s (b) 0.167 % .
20. A source of sound is moving along a circular orbit of radius 3 m
with an angular velocity of 10 rad/s. A sound detector located far
away from the source is executing linear simple harmonic motion
along the line BD with an amplitude BC = CD = 6 m. The frequency
of oscillation of the detector is 5/ per second. The source is at the
point A when the detector is at the point B. If the source emits a
continuous sound wave of frequency 340 Hz, find the maximum
Ans. (a) 1.0 cm (b) 4 cm (c) 1.6 cm–1 (d) 5 Hz .
and the minimum frequencies recorded by the detector.
14. Calculate the bulk modulus of air from the following data about a
sound wave of wavelength 35 cm travelling in air. The pressure at
a point varies between (1.0 × 105 ± 14) Pa and the particles of the
air vibrate in simple harmonic motion of amplitude 5.5 × 10–6 m.
Ans. 1.4 × 105 N/m2.
15. A bat emitting an ultrasonic wave of frequency 4.5 × 104 Hz flies
at a speed of 6 m/s between two parallel walls. Find the two
frequencies heard by the bat and the beat frequency between the
two. The speed of sound is 330 m/s. Ans. 442 Hz, 255 Hz .
Ans. 4.67 × 104 Hz, 4.34 × 104 Hz, 3270 Hz. 21. A band playing music at a frequency f is moving towards a wall at
a speed vb. A motorist is following the band with a speed vm.
16. A small source of sound S of frequency 500 Hz is attached to the
If v is the speed of sound, obtain an expression for the beat
end of a light string and is whirled in a vertical circle of radius
frequency heard by the motorist.
1.6 m. The string just remains tight when the source is at the
highest point. 2vb (v vm ) f
Ans. x =
(a) An observer is located in the same vertical plane at a large v 2 – vb 2
distance at the same height as the centre of the circle. T h e
speed of sound in air = 330 m/s and g = 10 m/s2. Find the 22. A boat is travelling in a river with a speed 10 m/s along the stream
maximum frequency heard by the observer. flowing with a speed 2 m/s. From this boat, a sound transmitter
is lowered into the river through a rigid support.The wavelength
(b) An observer is situated at a large distance vertically above of the sound emitted from the transmitter inside the water is 14.45
the centre of the circle. Find the frequencies heard by the mm. Assume that attenuation of sound in water and air is negligible.
observer corresponding to the sound emitted by the source
(a) What will be the frequency detected by a receiver kept inside
when it is at the same height as the centre.
downstream ?
(b) The transmitter and the receiver are now pulled up into air.
The air is blowing with a speed 5 m/s in the direction opposite
the river stream. Determine the frequency of the sound
detected by the receiver.(Temperature of the air and water =
20°C; density of river water = 103 kg/m3; Bulk modulus of
the water = 2.088 × 109 Pa; Gas constant R = 8.31 J /mol–K;
Ans. (a) 506 Hz (b) 490 Hz and 511 Hz. Mean molecular mass of air = 28.8 x 10–3 kg/mol; Cp / Cv for
17. A transverse mechanical harmonic wave is travelling on a string. air = 1.4)
Maximum velocity and maximum acceleration of a particle on the Ans. (a) 105 Hz, 100696 Hz, (b) 103040 Hz.
string are 3 m/s and 90 m/s2 respectively. If the wave is travelling
23. A train is running exacting on a semicircular rails at a speed of v/20
with a speed of 20 m/s on the string. Write wave function describing where v is speed of sound in air. At an instant the driver of the
the wave. Ans. y = 0.1 sin (30 t 1.5 x). engine at one end of semicircle and guard at the other end. At this
18. Two gases with different densities but same atomicity are mixed instant the driver blows a whistle whose real frequency is 400 Hz.
in proportions V 1 and V2 by volume. Prove that the velocity of Calculate the frequency of the whistle as heard by the guard, a
sound in a mixture will be given by passanger in the middle of the train and a workman on the track in
front of the train.
V1 V2 Ans. 400 Hz, 400 Hz, 421 Hz.
c = c1c2
V1c2 2 V2c12

where c1 and c2 are velocities of sound in pure gases respectively.


WAVE - I 623

Solutions Exercise 9.1Level -1


1. (a) Speed of sound does not change with frequency f = constant.
Mo MH oVo H VH
2. (b) The sonometer wire is forced to vibrate by tunning fork, so 21. (a) mixture = =
its frequency becomes 480 Hz. Vo VH Vo VH

dy
3 (c) At point c, the slope is negative, so the element at this H VH
dt oVo 1
o Vo
point moves downward. =
4. (a) On comparing the given equation with, V
Vo 1 H
y = Asin(kx – t), we get Vo
3
v1 = = = 3 m/s 1
k 1 1 1
mixture 16 17
1 1 2 or = =
o 1 1 32
v2 = m/s; v3 = m/s; v4 = = 2m/s;
4 2 1
5. (d) Sound cannot be travelled in vacuum. vmixture o 32
6. (d) By the observation you can find = 7b. vo = =
mixture 17
7. (d) The time taken by sound to travel 2km,
22. (b) SONAR emits ultrasonic waves.
2000 23. (a) The maximum particle velocity is given by vmax = A.
t= 6 s (slow)
300 24. (b) On comparing with standard equation of wave
8. (a) v = f = 2 × 5 = 10 cm/s. y = A sin ( t – kx + 0), we get
9. (b) Frequency is the fundamental characteristic which does not = 600 rad/s and k = 2/m
change from one to other medium. 600
10. (a) Speed of sound will not change with frequency. If frequency v = = = 300 m/s.
k 2
of sound changes, its wavelength is also changes, and so
f = v, constant in a medium. v 300
25. (b) = f = = 0.75m
54 400
11. (c) v=f = × 10 = 9 m/s.
60
(60180) 0.75
P x= × = = 0.125m
RT 2 2
12. (d) v = , so v T . Also is constant and so,
M
v 332
P 26. (d) f¢ = f v – vs = 450 500 Hz.
332 – 33
v= , is a constant.
400
27. (a) v = r = (2 f ) r = 2 1.2 = 50.24 m/s.
P v1 2 60
13. (c) As v = , v2 = = 4 = 2.
1 340
v
f1 = f = 500 = 436 Hz.
RT RT v – vs 340 50.24
14. (c) As v0 = and 2v0 =
M M v 340
and f2 = f = 500 = 586 Hz.
T v – vs 340 – 50.24
=4 T =4T = 4 × 273 = 1092 K.
T 340 340
v
15. (c) Sound waves in water are transverse at the surface and 28. (d) f1 = f = f =f×
v – vs 340 – 34 306
longitudinal along depth.
16. (c) Sound can travel longitudinally as well as transversly in v 340 340
solids. and f2 = f = f =f×
v – vs 340 – 17 323
17. (a) Transverse waves.
2 2 2 f1 19
18. (a) The intensity of wave, I = 2 f A v , so I f 2 and
f 2 = 18
I A2 .
29. (b) At A, source is moving away, and at B it is approaching the
19. (d) Sound waves in air are longitudinal.
observer, and so
20. (d) The wavelength of infrasonic wave is shorter than audible
f1 < f3 < f2
sound.
624 MECHANICS, HEAT, THERMODYNAMICS & WAVES
I
30. (b) We know that = 10 log I 3 F 2.5
0 35. (b) =
2 F
I
or 30 = 10 log I on solving, F = 2N.
0
2
I 36. (b) = x ; so phase difference between the vibrations of
3
I 0 = 10 = 1000
two such particles varies with distance between them.
31. (b) Explanation is in the theory of the chapter.
v F 30
32. (b) = 37. (b) v = = =
c k 1
F = 302 × = 302 × 1.3 × 10–4
v 100 103 5700
= = = 1.90Å = 0.12N
c 3 108
38. (b) = 2 – 1= 2 – 0.5
60
= 1.07 rad.
180
33. (c) x= = = 2m/s2
2 2 6 v – v0
39. (b) f¢ = f M x M
18 v
360
34. (b) = = m
500 25 330 – v0
or 0.94 =
18 330
x= = 3 25 v 0 19.8 m/s
2 2 Now 19.82 = 0 + 2 × 2 × x x = 98m.
= 0.12 m

Solutions Exercise 9.1Level -2


1. (b) [v P] max = 4v 5. (c) The line of motion of engine is perpendicular to line of motion
or A = 4v of the observer at that instant, so f¢ = f = 200 Hz.
or 2 f y0 = 4 × (f ) 6. (d) A A = 2, fA = f
y0 and A B = 1, fB = 2f
=
2 IA f A2 AA2
2. (d) To represent a travelling wave, the equation must satisfy the = =1
IB f B2 AB2
differential equation 7. (a) vP < v
or A < f
2 2
y y f
2 = v2 2 or A < or
t x 2
2
y 2
( x2 vt 2 ) A1 4 2
So = = –2v, 8. (b) A2 = =
t 2
t 2 1 1

2 2 2
y I max A1 A2 2 1
and = 2 = 2 = 2 =9
x2 I min A1 – A2 2 –1
The given equation does not satify the condition of travelling = 10 log 9 = 20 log 3
wave.
3. (b) y = Asin(10 x + 15 t + /3), 150 2 x 2 330
On comparing with standard equation, we get 9. (d) – = 1
300 330
k = 10 and = 15 .
x = 315 m.
15 10. (d) Frequency of the sound will not change in water.
v = = = 1.5 m/s.
k 10 11. (c) The sum of their KE and PE is a constant.
v vA 12. (c) The general equation of wave disturbance can be written as
4. (c) 5.5 × 103 = 5 × 103 v ...(i) 1
y = 2 . So
v vB 1 x – vt
and 6 × 103 = 5 × 103 ...(ii)
v vt = 1 at t = 2,
On substituting, v = 332 and simplifying, we get 1 1
v = = = 0.5 m/s.
vB t 2
vA = 5/6.
WAVE - I 625
I1 I1 14. (c) (I) = 4(ab); v=f = f × 4 (ab)
13. (c) 40 = 10log , = 104
I0 I0 3T 3
(II) t = =
4 4n
I2 I2
and 20 = 10log , = 102 2 2 3 3
I0 I0 (III) = x= =
4 2
I1 B
= 100. vA
I2 2g
15. (d) f = = / 0.06 ...(i)
A
I1 r22 r22 6 kg
vB
Also = 100 = = 8g
I2 r12 2 Also f = = / B ...(ii)
1 B
A
or r2 = 10 m. From above equations, we get 2g
B = 0.12 m

Solutions Exercise 9.2


2 2 2 8. (b,c) Solution is given in exercise 9.6, question number 18
1. (a,b,c) I = 2 f A v 9. (a, c, d) For observer O1,
v 10 v vs v v/5 4v
2. (a, c) a = = =1
10 10 1
f f 5f
1 1 For O2, there is change of medium, hence at the surface of
= = = 103 water, keeping frequency unchanged
a 10 –3
v 4v 16v
w 103 w 4 a
or f = = hz. a w 5f
2 2
velocity of wave relative to observer
v 10 f ''
Also = –2
f = 103 / 2 = 2 ×10 m. w

v
c v 4v
f S 5 21v 5 f 21 f
3. (a, b, c) f1 =
c 5 16v 16
w

c c v 10. (c, d) The graph shows the situation shown in figure. The
and f2 = f1 = f observed frequency will initially be more than the natural
c–v c–v
frequency. When the source is at P, observed frequency is
The wavelength of reflected wave equal to its natural frequency i.e., 2000 Hz.
c c c–v
= v
f2 = f c v For region AP: f f0
v vs cos
The number of waves striking the surface per second
v vs P
f c v A B
f1 = For PB: f f0
v vs cos
c
4. (a,b,c,d) y may be electric field, magnetic field, displacement or Minimum value of f will be
pressure. C
v
5. (b, c) The equation, y = 4 + 2 sin(6t – 3x) f min f0 when cos =1
Here A = 2, = 6, k = 3 v vs

6 300
v = = = = 2 unit. or 1800 2000
k 3 300 vs
6. (b,c,d) y = Asin(kx – t),
Velocity v = – Acos (kx – t) Solving this, we get, vs = 33.33 m/s
and maximum value of f can be
= A sin kx – t v
2 f max f0
v vs when cos =1
= – 2Acos (kx – t)
Acceleration a
= 2Asin (kx + t + )
300
Clearly, phase difference between displacement and or f max 2000 2250 Hz
acceleration is and phase difference between 300 33.33
displacement and velocity is /2. 11. (a, b, c) As per Doppler's effect, the apparent frequency heard
7. (a,b,c) Explaination in the theory of the chapter. by an observer is given by
626 MECHANICS, HEAT, THERMODYNAMICS & WAVES

v vo v vo
f' f , where symbols have their usual In option (d), f ' f f' f '
v vs v
meanings. 12. (c,d) Explaination is given in theory.

v
In option (a), f ' v vs
f f' f '

v vo
In option (b), f ' v vs
f f' f '

v vo
In option (c), f ' v vs
f f' f '

Solutions Exercise-9.3
1. (a) Statement-2 is the answer of statement-1. 5. (a) Statement-2 is the answer of statement-1.
2. (b) Light waves on being electromagnetic need no material 6. (a) Statement-2 is the answer of statement-1.
medium for their propagation. 7. (c) When sound propagate, the medium remains adiabatic and
3. (d) Particle velocity, vP = Acos( t – kx), which depends on so Q = 0.
time, v P/ . 8. (a) Statement-2 is the answer of statement-1.
9. (c) At nodes pressure is maximum. Particles within a loop vibrate
4. (d) With change in pressure, density of medium also changes in phase.
and so P / remains constant.

Solutions Exercise-9.4
Passage (Q. 1 - 3) : 2. (a) The adiabatic exponent for a mixture of gases is given by

RT P n1 n2 ... n1 n2
...
1. (b) The velocity of sound is given by c = = mix. 1 1 1 2 1
where n1, n2.... are number of moles of respective components.
In this case we have to determine mix as well as mix . Now,
Denoting moist air by m and dry air by d, we have
1 n1 1 n2 1
mP dP 1 n1 n2 ... 1 n1 n2 ... 1
cm .cd mix. 1 2
m d
P p 1 p 1
= P 1 P 1
m d 1 2
or cm cd
d m pi
Moist air can be assumed to consist of two components dry where P = mole fraction
total
component and water vapour.
1 (P p) p
(P p)M d pM m Thus
m mix. 1 = P (1.4 1) P(1.33 1)
RT RT
2 p
PM d or mix. = 1 1
d 5 5P
RT
m 5 2 p
m P p p Mm Hence = 1
d 7 7 5P
d P P Md
p
p 18 = 1 0.057 = 0.99
P
=1 1
P 28.8
p p
p 3. (c) cm = cd 1 0.057 1 0.375
= 1 0.375 = 0.997 P P
P
p
m 0.997 d = 332 1 0.318
P
WAVE - I 627
Passage for (Q. 13 – 15) :
4.58 13. (a) 14. (a) 15. (c)
= 332 1 0.318
760
0.06
= 331.7 m/s = kg/m
1 4.8
Passage (Q. 4 - 6) :
4. (b) The speed of sound depends on the frame of reference of the 0.2
observer. and = kg/m
2 2.56
5. (a) Since all the passengers in train A are moving with a velocity of
The velocity of wave in wire
20 m/s therefore the distribution of sound intensity of the whistle
by the passengers in train A is uniform. F
PQ, v1 =
v v0 340 30 31 1
6. (a) f1 = f v v = 800 = 800
s 340 20 32
80
= = 80 m/s.
v v0 0.06 / 4.8
31
f2 = f v v = 1120 The velocity of wave in wire
s 32
F
31 31 QR, v2 =
f2 – f1 = (1120 – 800) × = 320 = 310 Hz. 2
32 32
Passage (Q. 7 – 9) :
80
7. (b) 8. (c) 9. (a) = = 32 m/s.
The speed of sound will be same in both the cases i.e., 340 m/s. (0.2 / 2.56)
In the case when train is approaching the platform, the apparent (a) The time taken by the wave-pulse to reach the other end
frequency
PQ QR
v t = v1 v2
f1 = f v vs
4.8 2.56
= = 0.14 s.
340 80 32
= 400 = 412.12 Hz. (b) The amplitude of the reflected and transverse wave are given
340 10
by
When train receds, the apparent frequency
v2 v1
v Ar = Ai
f2 = f v v v2 v1
s

2v2
340 and At = Ai
= 600 = 574.65 Hz. v1 v2
340 15
After substiting the values, we get
Thus f = f1 f2 41.6Hz.
Ar = 1.5 cm,
Passage (Q. 10 – 12) : At = 2.0 cm
10. (a) 11. (a) 12 (a) 16. (A) p,q, (B) s, (C) p, r, (D) s
(a) If is the mass per limit length of rope, then tension in the
rope at a distance x from the free end y 4sin (5 x 4t ) 3cos 4t 5x
(A)
F = xg. 6
The velocity of transverse wave is super position of two coherent waves, so their equivalent
will be an another travelling wave
F
v = x x
(B) y 10cos t sin(100) t
330 330
xg Lets check at any point, say at x = 0, y = (10 cos t) sin (100t)
=
at any point amplitude is changing sinusoidally. so this is
equation of beats.
= xg . x
(C) y 10sin (2 x 120t ) 10cos (120t 2 x)
dx = superposition of two coherent waves travelling in opposite
(b) We have = xg
dt direction.
equation of standing waves
or x – 1/2 dx = g dt
(D) y 10sin (2 x 120t ) 8cos (118t 59 / 30 x)
L t
or x 1/ 2
dx = g dt = superposition of two waves whose frequency are slightly
0 0 different
t = 4L / g ( 1 120, 2 118) equation of beats
628 MECHANICS, HEAT, THERMODYNAMICS & WAVES
17. (A) q, (B) t, (C) s, (D) t
2 1
2 1 . t (2 b)
k 2 a T 8a 4
a
18. (A) q, (B) r, (C) q , (D) s
2 1
2 b T or f b RT P
T b v=
M
2 1
. x (2 a) Speed does not change with change in pressure unless
4a 2 temperature is changed.

Solutions Exercise-9.5
vH air T
1. We have 4. Given, = 0.005
vair H 4
v vair T = 0.028
= 1328 m/s. Ans. 1
and f = = 50 Hz.
vm T
H
2. We have vH = ...(1)
m
Also, = 2 cm.
2
mH mN
Here l = 4 cm.
m = V VN
H Wave speed, v = f = 50 × 0.04 = 2 m/s
5. The situation is shown in figure.
H VH NV N
= The time taken by the sound to reach the ground observer,
VH VN
x
N VN
H VH 1
H VH
=
VN y = 2000 m
VH 1
VH

28 1
1
m 2 2 16
= 1 =
H 1 3
2
Now from equation (i), we have 2000
t = = 5.88 s
340
3
[v m ] 0 = vH = 5
16 The distance, x = vt = 510 5.88 = 833 m
18
1300 3
= 325 3 6. From the geometry of the figure
4
3
t cos . =
5
[v m ] 27 = ( vm ) 0 1 591m/s.
546 The frequency heard by the person
3. The speed of sound will be same in both the cases i.e., 340 m/s. v
In the case when train is approaching the platform, the apparent f ' = f v v cos .
frequency s

v 5
Here v s = 120 = 33.3 m/s.
f1 = f v v 18
s
340
340 f ' = 640 = 680 Hz.
= 400 = 412.12 Hz. 3
340 10 340 33.3
5
When train receds, the apparent frequency 7. The tension in the string
v F = kx = 160 × 0.01 = 1.6 N
f2 = f v v
s
40 cm
340
= 600 = 574.65 Hz. The mass per unit length of the string
340 15
3
10 10
Thus f = f1 f2 41.6Hz. = = 0.025 kg/m
0.40
WAVE - I 629
The speed of transverse wave For the equilibrium of the system
F = M2g
F1.6 = M1g sin 30°
v = =
= 8 m/s.
0.025 or 98 = M2g
The time taken by wave pulse to reach the spring = M1g sin30°
M 1 = 20 kg
0.40 and M 2 = 20 kg.
t = = 0.05 s
8
8. Given, the velocity of transverse wave I2
9. We know that – = 10log I
v = 100 2 1
1

F F 10 6
= =
0.8 10 3 = 10log 10 = 33 dB.
5 10
F = 98 N.

Solutions Exercise-9.6
1. Any function will represent wave motion, if it satisfies, the (d) Solids possess both bulk modulus and shear modulus. So
differential equation both longitudinal and transverse waves propagate through
them.
2 2
y 2 y
= v (e) When a pulse passes through a dispersive medium, the
2 2
t x wavelength of wave changes, and so the shape of the pulse
(i) Given; y = (x – vt)2 changes.
4. We kmow that, velocity of sound in gas is given by
2
y
= 2
x2 P
v =
2
y
and 2 = 2v2.
t vH 2 H2 He
vH e =
Clearly the given function will represents a travelling wave. He H2
Do the other part similarly.
2. (a) The given function is the product of two reparate harmonic 7/5 4
functions of x and t, so it represents a stationary wave. = = 42 / 5.
5/3 2
(b) If does not satisfy the differential equation of wave so it will
5. Given, y = 10 sin 2 ( t – 0.005 x)
not represents any type of wave.
= 10 sin (2 t – 0.005 × 2 x)
(c) The given functions can be expressed in the form
y = A sin ( + ) On comparing with the standard equation of the travelling wave,
we get
where A = 32 42 = 5, y = A sin ( t – kx),
= 5x – 0.5 t A = 10 cm,
= 2
1 4
and = tan
3
or f =
2
So, it will represent a travelling wave.
(d) If represents the superposition of two stationary waves; = 1 Hz.
one represented by cos x sin t and other by cos 2x sin 2t. k = 0.005 × 2
3. (a) At displacement node, the variation of pressure is maximum.
Hence the displacement node is the pressure antinode and As k = ,
vice-versa. v
(b) Bats can produced and detect ultrasonic waves. Bats notice
2
the time of reflected waves and they then estimate the v = = = 200 cm/s.
k 0.005 2
distance of the object from them. From the intensity of
reflected waves, they can estimate the nature and size of the
2
object. Also k = = 0.005 × 2
(c) These instruments produce different overtones. Hence quality
produced by them will be different. = 200 cm.
630 MECHANICS, HEAT, THERMODYNAMICS & WAVES
6. Given, y = 0.25 × 10–3 sin (500 t – 0.025 x)
v
Compare this equation withy = A sin ( t – kx), fmax = f v v
s
we get A = 0.25 × 10–3 cm,
= 500 rad/s. 350
= 540 = 594 Hz.
330 30
2 2
Time period T = = = 0.01257 s 10. Given, y = A sin (t / T)
500
Also k = 0.025 /m t t x
To make the wave equation, should be replaced by ,
T T vT
2
As k = which now becomes the functions of x & T. Thus wave equations
becomes
2 2
= = = 251.2 cm t x
k 0.025 y = A sin .
T vT
Amplitude of particle velocity
11. (a) The dimensions of A must be the dimensions of length. i.e.,
v ma x = A L. Also dimensions of
= 500 × 0.25 × 10–3 = 0.125 m/s x
2A = 1,
and a max = a
= 5002 × 0.25 × 10–3 =62.5 cm/s2. dimensions of a will be the dimensions of x. i.e., L.
7. Velocity of the transverse wave
x x vt
(b) To make the wave equation should be replaced by .
F 90 g a a
v = = 3 Thus the wave equation becomes
8 10
x vt
900 y = f (x, t) = A sin .
a
= 3 = 335.4 m/s
8 10
x
= 2 f 12. Given, y = g (x, t0) = A sin ...(i)
a
= 2 × 256 = 1610 rad/s
The wave equation will be
1610
k = = = 4.8/m x v(t t0 )
v 335.4 y = g (x, t) = A sin ...(ii)
a
The equation of the wave
If we place t = t0 in equation (ii), the equation (i) will be obtained.
y = A sin ( t – kx)
13. From the figure, the amplitude of the wave
= 0.05 sin (16.1× 102 t – 4.8 x)
A = 1.0 cm, and = 4 cm,
8. The frequency of the reflected sound
2p 2 1
v v0 wave number k = = 1.6cm
l 4
f' = f v v
s
v 20
Frequency f = = = 5 Hz.
3 1450 100 4
= (40 10 )
1450 100
2 2
14. The wave number, k = = = 18 m–1
= 45.93 × 103 Hz. 0.35
9. The speed of the whistle A = 5.5 × 10–6m.
vs = r Given Pmax = 14 N/m2.
= 15 × 2 = 30 m/s.
or ABk = 14
v
fmin = f v v 14
s B =
AK

350 14
= 540 = 495 Hz. = 1.4 ´105 N/m 2 .
330 30 5.5 10 6
18
WAVE - I 631
The velocity of sound in a mixture is given by
v v0
15. fmin = f
v vs P
c = .

4 330 6
= 4.5 10
330 6 c V1 V2
1
Also c1 = c = c1c2 .
= 4 . 34 × 104 Hz. V1c22 V2c12

v v0
fmax = f
v vs 19. (a) of the mixture of gases is given by

n1 n2 n1 n2
4 330 6 = 1 1
= 4.5 10 1 1 2
330 6

= 4.67 × 104 Hz. 1 2


1 2
= 5 7
1 1 1
16. Velocity of the source, vs = gl = 10 1.6 = 4 m/s. 3 5
The maximum frequency, = 19/13.
v 330 Molecular mass of the mixture of gases
f' = f 500
v vs = 330 4
= 506 Hz.
n1M1 n2 M 2
M = n1 n2
17. Given, v ma x = A = 3 .....(i)
and a max = 2A = 90 .....(ii) 1 4 2 32
From above equations, we have = = 68/3 g.
1 2
= 30 rad/s The velocity of sound in the mixture of gases
and A = 0.1 m
The propagation constant, RT
v =
M
30
k = = = 1.5.
A 20 (19 /13) 8.31 300
= 3 = 400.9 m/s
Thus wave functions can be written as (68/ 3) 10
y = A sin ( t ± kx) = 0.1 sin (30 t ± 1.5 x).
RT
(b) We have v = = kT 1/2
P P M
18. We can write C 1 = and C2 =
1 2
v 1 T 1 1
100 = 100 = 100 = 0.167 %
v 2 T 2 300
1 C22
= 20. The maximum velocity of the detector
2 C12
v0 = A
If is the density of the mixture of gases, then = 2 fA

5
m1 m2 1V1 2V2 = 2 6 = 60 m/s.
= V1 V2 = V1 V2
The velocity of the source,
vs = r = 10 × 3 = 30 m/s.
2 V2
1V1 1 If source is going clockwise on the circular orbit, then maximum
= 1 V1 will be heard at E and minimum frequency at F.
V1 V2
v v0
Thus fmin = f
v vs
c12 V2
V1 1 V1 (V1C22 V2C12 )
or = c22 V1 = 340 60
1 C22 (V1 V2 ) = 340 255Hz.
V1 V2 340 30
632 MECHANICS, HEAT, THERMODYNAMICS & WAVES

v v0 B 2.088 109
and fmax = f
v vs v = = = 1445 m/s.
103

340 60 Frequency of wave


= 340 442 Hz.
340 30 1445
v
f = = 3 = 105 Hz.
21. The frequency noticed at the wall 14.45 10
Apparent frequency
v
f1 = f ...(i)
v vb (v )
f' = f
This frequency is reflected back and picked by motorist, then (v ) u

v vm 1445 2
f2 = f1 ....(ii) 5
v = 10 = 100696.
(1445 2) 10
From equations (i) and (ii), we have (b) Speed of sound in air

v vm RT
f 2= f v vb
M B v =
M
The frequency of sound noticed directly.
1.4 8.3 293
= 3 = 344 m/s.
v vm 28.8 10
f' = f ....(iii)
v vb
Now apparent frequency
Beat frequency
v
f' = f
2b(v vm ) (v ) u
f = f2 – f ' = f
v 2 – vb2
5 344 5
22. (a) Given,speed of the boat, = 10 = 103040 Hz.
(344 5) 10

23. In first two cases the observed frequency will be equal to the real
u frequency. i.e., 400 Hz.
Apparent frequency as observed by workman

W v D
f' = f
v vs
u = 10 m/s.
speed of river stream = 2 m/s. v M
= 400
Wavelength of sound wave inside water v
v
20
= 14.45 mm.
G
Speed of sound wave inside water,
20
= 400 = 421 Hz.
19
634 MECHANICS, HEAT, THERMODYNAMICS & WAVES
10.1 REFLECTION AND REFRACTION OF SOUND WAVES
Experiments show that the speed of light wave in air is greater than that in water. While
the speed of sound wave is greater in water. These results show that water is denser
medium for light wave but rarer for sound wave. Lloyd by his experiment proved that
when light wave reflected from mirror, the reflected light wave suffers a phase change of
radian. This is true for sound wave also. Frequency of wave is a fundamental property,
which remain constant in different mediums. If f, and v are the frequency, wavelength
and speed in air and f , and v are the corresponding quantities in water, then
for light wave; v < v, f = f, < and
for sound wave; v > v, f = f, >

Light wave; v < v, < Sound wave; v > v, >

Suppose an incident light wave Suppose an incident sound wave


yi = A sin (kax – t), then the equations for yi = A sin (kax – t), then the equations of reflected and
reflected and transmitted waves are: transmitted waves are:
yr = – Ar sin (kax + t – ) = Ar sin (kax + t) yr = – Ar sin(kax + t) and
and yt = At sin (k x – t) yt = At sin (k x – t)
Also A = Ar + At. Here ka and k are propagation Also A = Ar + At
constants for air and water respectively.

When sound wave enters from water to air, it get reflected from the interface with a phase
change of radian. So we have
yi = A sin (k x – t),
yr = – Ar sin ( k x + t – )
And yt = At sin (kax – t)
Also A = Ar + At

Refraction:
If i and r are the angles of incident and angle of refraction respectively, then for light
wave;

sin i v
= .
sin r v

Here, v ' v, r i
WAVE - II 635
And for sound wave;

sin i v
= .
sin r v
Here v ' v, r i
Here straight lines are showing directions of propagation of wave.

10.2 REFLECTION AND TRANSMISSION OF TRANSVERSE WAVE IN


STRETCHED STRING
Reflection of a wave from a fixed boundary:
Consider a wave pulse travelling along a string connected to a rigid support, such as a
wall. As the pulse reaches the wall, it exerts an upward force on the wall. By Newton’s
third law, the wall exerts an equal amount of force on the string in downward direction.
Because of this downward force, an invert pulse produces which travels in reverse
direction. Thus a crest is reflected as a trough.
Hence when a wave is reflected from a fixed boundary, it is reflected back with a phase
difference of radian. If an incident wave is represented by
yi = A sin (kx – t),
then the reflected wave can be represented as
yr = A sin [(– kx – t) + ]
= A sin (kx + t).
Reflection of a wave from a free boundary :
Consider a wave pulse travelling along a string connected to a light ring, which can slide
without friction up and down on a vertical rod. As the crest produced in the string
reaches at the ring, it rises above its mean position. In the process string stretches, and
so it pulls back the ring to its mean position. Because of this a crest produces and
reflected back without any phase change.
Hence when a wave is reflected from a free boundary, it suffers no phase change.
Suppose an incident wave is represented by
yi = A sin(kx – t),
then the reflected wave can be represented as
yr = – A sin (kx + t).
From the above discussion, it can be concluded that at the fixed boundary, the string end
has no displacement, so a node is formed at the boundary. In case of free boundary, the
displacement is maximum (2A), so an antinode is formed at the boundary.
Reflection at fixed boundary Reflection at free
(for light wave) boundary (for light wave)
1. Displacement, velocity undergo a 1. Displacement, particle velocity
phase change of rad. undergo no phase change.
2. Strain and pressure variation undergo 2. Strain variation undergo a phase
no phase change. change of rad.
Reflection and transmission :
(i) Consider a combined string made of two parts: string A and string B, part A is

F
thinner than B. As speed of wave v = , so vA > vB, i.e., a wave pulse travels

faster in thinner string. For a wave pulse travelling from string A to B, the joints
behaves as a rigid boundary, so the reflected wave pulse suffers a phase change
of radian, while there is no phase change in transmitted wave pulse.
636 MECHANICS, HEAT, THERMODYNAMICS & WAVES

Suppose an incident wave is represented by


y = A sin(k A x t)
then reflected and transmitted waves can be represented by
yr = Ar sin(k A x t )
and yt = At sin(k B x t) .
Also A = Ar At , fA = fB, A B
(ii) Now consider a wave pulse travelling from string B to A. Here joints behaves as
a free boundary, and so the reflected wave pulse suffers no phase change.
Transmitted wave pulse also has no phase change.
Suppose an incident wave pulse is represented by
yi = A sin( k B x t)
then reflected and transmitted wave pulse can be represented as
yr = Ar sin(k B x t)
and yt = At sin(k A x t)
also A = Ar At , fB = fA, B A.

Ex. 1 Two wires of different linear mass densities are joined, As k =


v
and constant
consider junction to be at x = 0 . An incident wave yi = A i sin
( t – k1x) is travelling to the right from the region x 0 . At the v2 v1
Ar = Ai
boundary the wave is partly reflected and partly transmitted. Find v2 v1
the reflected and transmitted amplitudes in terms of the incident
amplitude.
2v2
Sol. and At = v1 v2
Ai
Given yi = Ai sin( t k1x)
Medium 1 Medium 2 Ex. 2 A string of length 20 cm and linear mass density 0.40 g/cm
is fixed at both ends and is kept under a tension of 16N. A wave
x = 0 pulse is produced at t = 0 near an end as shown in figure, which
The equation of reflected and transmitted waves are travels towards the other end. When will the string have the shape
yr = Ar sin( t + k1x), shown in the figure again?
yt = Atsin ( t – k2x) Sol.
We have Ar + At = Ai ...(i)
and yr + yt = yi ...(ii)
Differentiating equation (ii) partially
yr yt yi
=
x x x

or [ Ar sin( t k1x )] [ At sin( t k 2 x )] =


x x

Ai sin( t k1x )
x
Fig. 10.10
or Ar k1 cos t At k2 cos t = Ai k1 cos t at x = 0
0.40
Given µ = 0.40 g/cm = 100 = 0.040 kg/m
or Ar k1 At k2 = A i k1 ...(iii) 1000
Solving equations (i) & (iii) , we get Wave speed in the stretched string
k1 k 2 F
Ar = Ai 16
k1 k2 v = = = 20 m/s
0.040

2 k1 The string will regain its shape after travelling the pulse a distance
and At = Ai Ans. = 20 + 20 = 40 cm
k1 k2
0.40
Thus time spend t = = 0.02 s Ans.
20
WAVE - II 637
10.3 SUPERPOSITION OF WAVES
In an orchetra, we can differentiate the sounds of different musical instruments playing
simultaneously. The antenna of our television receiver is set in motion by the resultant
effect of many electromagnetic waves from different broadcasting stations. Above
examples show the independent behaviour of the waves and superposition of waves.
Independent behaviour of waves: When number of waves travel through a region at the
same time, each wave travels independently of the other i.e., as if all other waves were
absent.
Principle of superposition of waves: The principle states that when a number of waves
travel through a medium simultaneously, the resultant displacement of any particle of the
medium at any given time is equal to the algebraic sum of the displacements due to the
individual waves.
If y1, y2,..., yn are the displacements produced by waves acting separately, then the
resultant displacement, when all the waves act simultaneously is given by the algebraic
sum
y = y1 + y2 + ... + yn (Principle of superposition)

Note:
1. The principle of superposition is valid for small displacements.
2. The principle is valid for the quantities like displacement, velocity, pressure,
momentum etc. but not for kinetic energy.
3. The algebraic sum is application only for one dimensional waves. For other waves
(2D or 3D waves), it will by y = y1 y2 ... yn .

(i) Superposition of two identical pulses travelling towards


each other: To understand easily, consider two pulses moving towards each
other with a constant speed of 1 m/s . Figure shows the positions of the pulses after
every second. They cross each other between t = 2 s and 3s. At t = 2.5 s the two
pulses superpose in such a way that the displacement of the resultant pulse is twice
the displacement of either pulse, i.e., yres = y + y = 2y.
638 MECHANICS, HEAT, THERMODYNAMICS & WAVES
(ii) Superposition of two pulses of equal and opposite shapes
travelling towards each other:
Consider two equal and opposite pulses coming towards each other with a speed of
1m/s . Figure shows the positions of the pulses t = 0, 1.5s, 2.5s, 3.5s & 4.5s. At 2.5 s
the two pulses superpose in such a way that the displacement of the resultant pulse
is zero, i.e., yres = y – y = 0. This again shows that the resultant displacement is the
algebraic sum of individual waves.

From the graphical treatment of superposition of wave pulses, it can be concluded


that the two pulses continue to retain their individual identity after crossing each
other. However, at the instant they cross each other, the appearance of the wave
profile is different from the shape of individual pulse.
There are three types of superposition in our study.
(i) When two or more waves of same frequency travel simultaneously in the same
direction or nearly along the same direction in a medium, they superpose on each
other, give new disturbance, called interference of waves.
(ii) When two or more waves of slightly different frequencies ( f1 ~ f 2 | 10) travel
with the same speed in the same direction in a medium then superpose on each
other, give beats.
(iii) When two identical waves travel with the same speed in the opposite directions in
a medium, they superpose on each other, give stationary wave.
WAVE - II 639
Ex. 3 Fig. 10.13 shows two rectangular wave pulses travelling in opposite directions along a string at t = 0 . Sketch the wave functions
for t = 1, 2 and 3s.
Sol.

10.4 INTERFERENCE
Consider two harmonic waves of same frequency (coherent waves). Suppose A1 and A2
be the amplitudes of the waves and is the phase difference between them. It is assumed
that the waves are plane and move almost along a line. Thus wave equations are
y1 = A1 sin (kx – t) ...(1)
and y2 = A2 sin (kx – t + ) ...(2)

2
where = x.

When both the waves travel simultaneously, the resultant wave at P can be obtained by Fig. 10.14
principle of superposition, i.e.,
y = y 1 + y2
= A1 sin(kx t ) A2 sin kx t

= A1 sin(kx t ) [ A2 sin(kx t ) cos A2 cos(kx t )sin ]

= ( A1 A2 cos )sin( kx t ) A2 sin cos( kx t)

Let A1 A2 cos = R cos ...(3)

and A2 sin = R sin ...(4)


Squaring and adding equations (3) and (4) , we get

R = A12 A22 2 A1 A2 cos ...(5)

As I A2 , I = I1 I 2 2 I1 I 2 cos ...(6)
640 MECHANICS, HEAT, THERMODYNAMICS & WAVES

A2 sin
Also tan = A1 A2 cos
...(7)

The resultant wave becomes


y = R sin (kx t) ...(8)
Clearly the resultant wave has the same frequency and speed as the interfereing waves
Two types of interference
(i) Constructive interference:
The intensity of the resultant wave is maximum at P, if
cos = +1
or =
2 n , where n = 0, 1, 2, 3, .......
As 2 phase difference is equal to a path difference , so
x = n
Thus from (5), Rmax = A1 +A2

(ii) Destructive interference: The intensity of the resultant wave will be


minimum at P, if cos = –1
or = (2n 1) , where n = 1, 2, 3, ....

and x = (2n 1)
2
Thus from (5), Rmin = A 1 – A2
The ratio of maximum to minimum intensities

I max ( A1 A2 ) 2
= … (9)
I min ( A1 A2 )2

Note:
The distance between maxima and next minima = .
2
Conditions of sustained interference:
1. Mathematically interference phenomenon can takes place between two waves of
same frequency and different amplitudes. But for obserable interference, the
amplitudes of the waves should be equal. In this case,
A1 = A2 = A
R2 = A2 A2 2 AA cos

or R2 = 2 A2 (1 cos )

= 2 A2 2 cos2
2

R2 = 4 A2 cos2 ... (10)


2
Write R2 = I and 4A2 = I o

I = I o cos 2 ... (11)


2
WAVE - II 641
From equation (11), the maximum intensity is 4A2 and minimum intensity is zero. In the
phenomenon of interference the energy is not destroyed but is only redistributed from
the positions of minimum intensity to those of maximum intensity. At the maximum intensity
positions the intensity due to the two waves should be 2A2 but it actually 4A2. As shown
in figure, the intensity varies from 0 to 4A2 and the average is still 2A2. It is equal to a
uniform intensity of 2A2 which will be present in the absence of interference phenomenon
between two waves. Hence the formation of maxima and minima is due to interference of
waves is in accordance with the law of conservation of energy.
2. The phase difference between the two waves must remain constant, it will not
change with time. If the phase difference between the waves continuously
changes,then the positions of the maximum and minimum intensities do not remain
fix.
3. The waves must travel in the same direction because with increase in obliquity
between the waves, the resultant intensity decreases.

10.5 INTERFERENCE OF SOUND WAVES: QUINKE’S TUBE


Two independent sources of sound can not be coherent because they can not maintain
the constant phase difference. In practice the coherent sources can be produced from a
single source, as in case of Quinke’s tube. Source so produced remain coherent because
they produced from the same source. Quinke’s tube consists of two U - tubes, one can
slide over the other. In figure tube D can slide inside the tube ABC. When a source of
sound (vibrating tuning fork) is held near the opening at A, the sound waves travel
through the tube along two paths ABC and ADC. The resultant sound produced at C
due to the interference will be maximum or minimum depending upon their path difference.
The path difference between the two sounds.
x = ADC ABC

If x = n , then maximum sound will be heard, and if x = (2n 1) , then minimum


2
sound will be heard. But gradually sliding the tube D outwards or inwards maximum and
minimum can be noticed at regular intervals.

Note:
Each x cm slide of tube will cause a path difference of 2x. Thus for a maxima and next
minima

= 2x or = 4x.
2

Ex. 4 The ratio of intensities of two interfering waves is 4 : 1. of the second source can be varied electronically between 2000 and
What will be the ratio of maximum and minimum intensities in 5000 Hz. Find the frequencies at which maxima of intensity are
their interference? detected. The speed of sound in air 340 m/s.

Sol.
If A1 and A2 be the amplitudes of the waves, then

I1 A12 4
I2 = 2 =
A2 1

A1 2
or A2 =
1

I max ( A1 A2 )2 (2 1) 2
= 2 = = 9 Ans.
I min ( A1 A2 ) (2 1) 2

Ex. 5 Fig.10.20. shows a tube structure in which a sound signal Fig. 10.19
is sent from one end and is received at the other end. The frequency
642 MECHANICS, HEAT, THERMODYNAMICS & WAVES
Sol. Ex. 7 In a Quinke’s experiment, the sound intensity has a
The path difference between the sounds going through the tubes minimum value I at a particular position. As the sliding tube is
x = ABCD – AED pulled out by a distance of 16.5 mm, the intensity increase to a
maximum of 9I. Take the speed of sound in air to be 330 m/s. (a)
= (10 20 10) – ( 500 10) Find the frequency of the sound source. (b) Find the ratio of the
= 7.64 cm amplitudes of the two waves arriving at the detector assuming that
The wavelength of the sound it does not change much between the position of minimum and
v 340 maximum intensity.
= f = f m Sol.
For maximum intensity, (a) The separation between minimum and next maximum is /2. For
the slide of tube by 16.5 mm will cause a path difference of 33 mm.
x = n
340 = 33 mm = 66 mm
or 7.64 × 10–2 = n 2
f
v 330
340 and f = 3 = 5.0 kHz Ans.
f = n 66 10
2
7.64 10
= n 4450 Hz I max 9I ( A1 A2 )2
(b) = =
Here n 1 . For n = 1, f = 4450 Hz and for n = 2, f = 8900 Hz. Thus the I min I ( A1 A2 ) 2
frequency within the specified range is 4450 Hz.
Ex. 6 Two simple harmonic sources separated by 20 m oscillate A1 A2
or A1 A2 = 3
according to the equation
y1 = 0.06sin t(m) A1
or A2 = 2 Ans.
and y2 = 0.02sin t(m)
These sources induce simple harmonic waves along a rod of speed Ex. 8 A source of sound S and a detector D are placed at some
3 m/s. Determine the equation of motion of a particle 12 m from the distance from one another. A big cardboard is placed near the detector
first source and 8 m from the second.
and perpendicular to the line SD as shown in the Fig. 10.21. It is
Sol. The sources are separated as shown in figure gradually moved away and it is found that the intensity changes
from maximum to minimum as the board is moved through a
distance of 20 cm. Find the frequency of the sound emitted. The
velocity of sound in air is 336 m/s.

Sol.
Fig. 10.20
The equation of oscillation of a source at a distance x can be written as
y = A sin (t x / v)
For source S1, x = 12 m
12
y1 = 0.06sin t
3
= 0.06sin (t 4) = 0.06 sin t
and for source S2, x = – 8 m Fig. 10.21
8 The distance between maximum to next minimum is /2. When cardboard
y2 = 0.02sin t 0.02sin t 8/3 is displaced by 20 cm the path difference produces by 40 cm.
3
/2 = 40 cm
8 2 or = 80 cm
= 0.02sin t 0.02sin t
3 3 v 336
Frequency of sound f = =
The resultant displacement 0.80
= 420 Hz Ans.
2
y = y1 y2 = 0.06sin t 0.02sin t
3
Ex. 9 Two sources of sound S1 and S2, emitting waves of equal
wavelength 20.0 cm are placed with a separation of 20.0 cm between
2 2 them. A detector can be moved on a line parallel to S1S2 and at a
= 0.06sin t 0.02 sin t cos cos t sin
distance of 20.0 cm from it. Initially the detector is equidistance
3 3
from the two sources. Assuming that the waves emitted by the
= 0.06sin t 0.02 sin t 1/ 2 cos t 3/2 sources are in phase, find the minimum distance through which
the detector should be shifted to detect a minimum of sound.
= 0.05 sin t – 0.0173 cos t Ans.
WAVE - II 643
S1S2 lies in its plane and middle point of S 1S2 is at the centre of
Sol. the wire. Find the angular position on the wire for which
constructive interference takes place.
Sol.

Fig. 10.22
Let the detector is shifted by x
The path difference between the sounds reach at D,

x = 20 2 (10 x)2 20 2 (10 x) 2 Fig. 10.24 Fig. 10.25


As detector detect the minima for the shifted position Since is small, so the path difference x = 3 cos
x = /2 xm = 0 for = 90° at B
20 and xmax = 3 , for = 0 at A
or 20 2 (10 x)2 20 2 (10 x) 2 = 2
For constructive interference x can be, 0, , 2 , 3 , .......
After solving, we get x = 12.6 cm Ans. There are two positions between A and B where constructive interference
Ex. 10 Two coherent narrow source emitting sound of occurs. These are
wavelength in the same phase are placed parallel to each other at x = and 2
a small separation of 2 . The sound is detected by moving a detector =
3 cos
on the screen at a distance D (>> ) from the source S1 as shown
in Fig. 10.23. Find the distance x such that the intensity at P is 1 1
or cos = 1 = cos 1/ 3
equal to the intensity at O. 3
Sol. and 3 cos = 2 = cos 1 (2 / 3)
2
The maximum path difference between the sounds can be 2 , and minimum
can be zero, when x tends to infinity. So there is only one position Similar positions in other quadrants can also be obtained.
between them at which intensity is maximum, for which path difference
between sound at P is Ex. 12 Three identical radio sources each of intensity I0 ,
vibrating in phase, are positioned, along y-axis, as shown in
Fig. 10.27. The frequency of each source is 10 8 Hz, separation
between two adjacent sources is d = 1 km. A detector moves parallel
to y-axis at a perpendicular distance D-from the sources. The
separation between two consecutive minima registered is 10 km.
(a) Determine the radiation wavelength and the distance D, the
resultant intensity at maxima and minima.
(b) If the source S 2 stops radiating, what should be the new
separation between the remaining two sources so that
positions of maxima are unchanged?
Sol.
Fig. 10.23
Possible position x Path difference x Intensity
x=0 2 Maximum
x=? Maximum
x= 0 Maximum
From the figure, path difference x = 2 cos
D
where cos = 2
D x2

D
2 =
2
D x2

or x = 3D Ans. Fig. 10.27


Ex. 11 Figure shows two coherent sources S1 and S2 which emit For D >>d, the waves from all the sources travel approximately
sound of wavelength in phase. The separation between the sources in the same direction. The path difference between any two
is 3 . A circular wire of large radius is placed is such a way that consecutive sources is dsin . The corresponding phase difference
644 MECHANICS, HEAT, THERMODYNAMICS & WAVES
2 D
= (d sin ) = kd sin . The equations of the three waves arriving The separation between two consecutive minima also be .
d
at P can be written as Wavelength of radiations,
y1 = A sin( kx ), c 3 108
= f = = 3m Ans.
y2 = A sin(kx) , 108

y3 = A sin( kx ) (10 103 ) 1000


d 10000
D = = = km
The resultant wave is given by 3 3
(a) Resultant intensity at maxima
y = y1 y2 y3 Imax = I0 [ 1 + 2cos(2 n)]2
= A[sin(kx ) sin( kx )] A sin kx = 9I0
Resultant intensity at minima
= 2 A sin kx cos( ) A sin kx 2n
Imin = I0[1 + 2 cos( )]2 = 0, where =
; n = 1, 3,.....
= A(1 2cos )sin kx 3
(b) Let d be the new distance between the adjacent sources, then
The amplitude of resultant wave
y1 = A sin( kx ') ; y2 = A sin(kx ')
R = A(1 2cos )
where = kd 'sin
As I R2 and I 0 A2 , we have The new amplitudeR = 2A cos (kd sin )
The intensity I = R '2 = 4 A2 cos 2 (kd 'sin )
I = I 0 [1 2cos( kd sin )]2
For maxima = 4 I 0 2 cos 2 ( kd 'sin )

cos(kd sin ) = 1 For maxima cos( kd 'sin ) = 1


or kd 'sin = n
or kd sin = 2n
2
2n n or d 'sin = n
sin = =
kd d
2 y 'n
or d' = n
yn D
From figure sin tan = , where yn denotes position of n th
D
n D
maxima. Therefore y 'n =
2d '
yn n As position of maxima is unchanged
=
D d yn = y 'n
D D D
or yn = n , where n = 0, 1, 2, ..... n = n
d d 2d '
Separation between two consecutive maximas d 1
d' = = km Ans.
D 2 2
= yn yn 1 =
d

10.6 INTERFERENCE IN TIME : BEATS


When two or more sound waves of nearly same frequency and amplitude travel along
the same path in the same directions, the intensity of the resultant sound wave at any
point in the medium rises and falls (technically known as waxing and waning of sound)
alternately with time.
The periodic variations in the intensity of sound due to the superposition of sound
waves of slightly different frequencies are called beats. One rise and one fall of the
intensity constitute a beat. The number of beat produced per second is called beats
frequency. Thus
beat frequency = difference in frequency of the two superposing waves.
or fbeat = f1 ~ f 2
WAVE - II 645
Condition of audible beats:
The persistance of hearing of our ear is (1/10)th of a second. So our ears can not
differentiate the sound variation more than 10 in a second. Hence beats heard will not be
distinct if the number of beats produced are more than 10 per second.

Formation of beats by graphical method


Figure shows two sound waves of frequencies f1 and f2. The frequency f1 is slightly
greater than f2.
At t = t1, the two waves meet in the same phase at a given point, so they produce
maximum intensity with the passes of time, the phase difference between the two waves
increases and so the two curves gradually become out of phase.
At t = t2, the two waves are in exactly opposite phase, so they produce minimum sound
intensity. Thereafter the phase difference goes on decreasing with time. At t = t3, the first
wave complete one more vibration than the second wave. And so they again in same phase
and produce maximum intensity, and so on. The resultant wave as obtained by the
superposition of the two waves is shown in Fig. 10.28 (b). The dashed envelopes above
and below it show the variation of the amplitude of the wave. The time interval from t1 to t3
is one beat period, because during this duration only one beat is formed. Clearly if
t3 – t1 = 1s, then beat frequency fbeat = f1 – f2 = 1.
Analytical treatment of beats
Consider two sound waves of equal amplitudes and travelling in a medium in the same
direction but of frequencies f1 and f2 (f1 being slightly greater than f2). The pressure
variations due to two waves at a given points (for simplicity x = 0) may be represented as
P1 = Pm sin 1t = Pm sin 2 f1t

and P2
Pm sin 2t = = Pm sin 2 f 2t
By the principle of superposition, the net pressure variation
P = P1 P2
= Pm sin 2 f1t Pm sin 2 f 2t

f1 f2
= 2 Pm cos 2 t.
2

f1 f2
sin 2 t
2

f1 f2 f1 f2
Here = f av and = f mod
2 2
P = 2 Pm cos(2 f mod t )sin(2 f av t )
or P = R sin(2 f av t ) … (1)

where R = 2 Pm cos(2 f modt ) is the amplitude of the pressure variation. As f1 is slightly


greater than f2, so fmod << fav, i.e., R varies very slowly with time. Hence equation (1)
represents a wave of rapid oscillation of average frequency fav modulated by a slowly
varying oscillation of frequency fmod.
The amplitude of pressure variation R of the resultant wave will be maximum, when
cos 2 f mod t = 1 , and R = 2 Pm
or 2 f mod t = n , n = 0, 1, 2, ....

f1 f2
or 2 t = n
2
646 MECHANICS, HEAT, THERMODYNAMICS & WAVES

n
or t = … (2)
f1 f2

1 2
The instant of maxima t = 0, , ,......
f1 f 2 f1 f2
Time interval between two successive maximas
1
t =
f1 f2
Similarly, the amplitude of pressure variation will be minimum, when
cos(2 f mod t ) = 0, and R = 0

f1 f2
or 2 t = (2n 1) , n = 0, 1, 2, ......
2 2
(2n 1)
or t = 2( f1 f 2 )
… (3)

1 3 5
The instant of minimas t = , , ,...
2( f1 f 2 ) 2( f1 f 2 ) 2( f1 f2 )
The interval between two successive minimas
1
t = f1 f2
Clearly, both maxima and minima of intensity occur alternately. Technically one maximum
followed by a minimum is called a beat. Thus beat frequency
1
fbeat = = f1 f2 .
t
Variation of intensity of resulting wave at a point
The intensity of sound wave is given by

Pm 2
I =
2 v
For the resulting sound wave

[2 Pm cos(2 f mod t )]2


I =
2 v

4 Pm2 cos 2 (2 f mod t )


=
2 v

or I = I 0 [cos 2{2 f mod t} 1]

4 Pm 2
where I0 = 2 v

Clearly the intensity of the resulting wave varies between 0 to I0 with a frequency which
is double the frequency of pressure amplitude R.
Tuning fork :
A straight rod is bent to give U-shaped structure is known as tuning fork. The tuning
form vibrates in three portions as shown in (fig 10.29)
WAVE - II 647
The free ends of the fork behaves as antinodes. The junction of U-shaped rod and stem
also behaves as an antinode. Tuning fork is an important source of standard frequency.
It can be set into vibration when one of the prongs is struck against a hard rubber pad.
Tuning forks of frequencies 256, 288, 320, 341.33, 384, 426.66, 480 and 512 Hz are commonly
manufactured.
For a tuning fork of rectangular cross-section, the fundamental frequency is given by

Yt
f = A
2

where Y Young’s modulus


density of material of tuning fork
t thickness of rod
length of the tuning fork
Facts to know :
1. Why does a tuning fork have two prongs? Would the tuning fork be of any use,
if one of the prongs is cut off ?
When a tuning fork is set into vibrations, its two prongs vibrate in opposite
phases. One prong of the fork is forced to vibrate by the other and vice-versa. Fig.10.30
Hence by holding its stem in the hand, a tuning fork can be set into vibrations and
no external force is required to maintain its vibrations.
If one of the prongs is cut-off, the vibrations of the tuning fork will soon die out
and can be maintained only by some external periodic force.
2. What is difference between a tone and a note?
A sound of single frequency is called a tone. A combination of tones of different Fig.10.31
frequencies is called a note.
3. What will be the speed of sound in a perfectly rigid rod?
The Young’s modulus of elasticity of a perfectly rigid rod is infinite, so speed of
sound in the rod will be infinite.
Determination of frequency of tuning fork by using beats :
Suppose a tuning fork A of known frequency fA and fork B of unknown frequency fB.
When the two forks are sounded together, they produce fb beats per second. Then
fB = fA
f b or f A – f b
The correct frequency may be determined by any of the following two methods :
(a) Loading method : Put the little wax to the prong of the fork B (decreases
Fig.10.32
the frequency of B) and again note the number of beats per second. If the beats
frequency decreases on loading the prong of the fork B of unknown in frequency,
then its frequency is given by
fB = fA fb
On the other hand, if the beats frequency increases after loading, then
fB = fA fb
(b) Filing method : If the prong of the tuning form B is filed, its frequency
increases. Again, note the number of beats per second. If on filing the prong of B,
the beat frequency decreases, then
fB = fA fb
On the other hand, if beat frequency increases on filing, then f B = f A fb .
648 MECHANICS, HEAT, THERMODYNAMICS & WAVES
Ex. 13 A tuning fork of unknown frequency gives 4 beats with According to the principle of superposition of waves, we have
a tuning fork of frequency 310 Hz. It gives the same number of y = y1 y2 y3
beats on filing. Find the unknown frequency.
= A[sin 2 ( f 1)t sin 2 ft sin 2 ( f 1)t ]
Sol.
The unknown frequency of the tuning fork can be = A[sin 2 ( f 1)t sin 2 ( f 1)t sin 2 ft ]
= 310 4 {2 ( f 1)t 2 (f 1)t}
= 2 A sin
or f = 314 or 306 Hz 2
Suppose f = 314 Hz
{2 ( f 1)t 2 (f 1)t}
On filing, let it becomes = 318 Hz. cos A sin 2 ft
2
When it sounded together with a fork of frequency 310 Hz, beats
frequency will be more than 4 per second. Therefore unknown frequency = 2 A sin(2 ft ).cos(2 t ) A sin(2 ft )
can not be 314 Hz.
= [2 A cos(2 t ) A]sin(2 ft )
Now suppose f = 306 Hz.
On filing, let it becomes = 314 Hz. = R sin(2 ft )
When it sounded again with a fork of frequency 310 Hz it gives 4 beats where R = [2 A cos(2 t ) A]
per second. So unknown frequency must be 306 Hz.
Ex. 14 When a tuning fork A of unknown frequency is sounded The intensity of the wave, I R2
with another tuning fork B of frequency 256 Hz, 3 beats per second
[2 A cos(2 t ) A]2
are observed. After that A is loaded with wax and sounded, the beat
frequency decreases. Find the frequency of the tuning fork A. The intensity of the resulting wave is maximum or minimum,
Sol. dI
if =0
The frequency of the fork B can be dt
= 256 3 Hz or 2[2cos(2 t ) 1] [–2sin(2 t ) 2 ] = 0
= 259 Hz or 253 Hz
which gives either,
Suppose the frequency of the fork B
= 259 Hz 2cos(2 t ) 1 = 0
On loading, its frequency let becomes or sin(2 t ) = 0
= 257 Hz.
Now it will give less number of beats with the fork B. So the frequency or cos(2 t ) = – 1/2
of fork A must be 259 Hz. or 2 t = n
Ex. 15 A set of 56 tuning forks are arranged in series of
2
increasing frequencies. Each fork gives 4 beats/s with preceding or 2 t = 2 n
one. If the frequency of last fork is 2 times that of the first, what is 3
the frequency of 40th fork?
n
Sol. or t =
2
Suppose the frequency of the first fork is f, then frequency of the last
fork 1
or t = n , where n = 0, 1, 2, 3,
= f (56 1) 4 = f + 220 3
where n = 0, 1, 2, 3
Given (f 220) = 2f
1 2 4 5 7
f = 220 Hz Ans. t = , , , , ,....
Now the frequency of the 40th fork 3 3 3 3 3
= f + (40 – 1) × 4 1 3
= 220 + 39 × 4 t = 0, , 1, , 2,.....
2 2
= 376 Hz Ans.
Ex. 16 There are three sources of sound of equal intensities As R = A[2cos(2 t ) 1]
with frequency (f – 1), f, (f + 1) Hz. What is the beat frequency As sin(2 t ) = 0
heard if all the sources are switched on simultaneously ? I = 0
Sol.
Let following are the waves (at x = 0) produced by the sources
cos(2 t ) 1 sin 2 (2 t ) = 1

y1 = A sin 2 ( f 1)t I = A[2cos 2 t 1]2


y2 = A sin 2 ft = A[2 ( 1) 1]2
and y3 = A sin 2 ( f 1)t = 9A2 , A2, 9 A2, A2, ........
WAVE - II 649
Clearly in 1 second there are two maximas and two minimas, so beat
frequency = 2 per second.

Note:
In practice the intensity A2 in comparison to 9A2 is not noticeable,
and therefore can be neglected. Then beat frequency in this case
will be 1 beat/s.

Fig. 10.33

10.7 STATIONARY WAVES


When two identical waves of same frequency travel in opposite directions with the same
speed along the same path superpose each other give rise to a new wave. The resultant
wave does not travel in the either direction and therefore is called stationary or standing
wave.
The resultant wave oscillates in the same fixed position. Some particles of the medium
remain permanently at rest. Their positions are called nodes. Some other particles always
have maximum displacement. Their positions are called antinode. The positions of nodes
and antinode do not change with time. In stationary waves, there is no transfer of energy
along the medium in either direction.
Necessary condition for formation of stationary waves :
In particle a stationary wave is formed when a progressive wave and its reflected wave
are superposed. So a stationary wave can be produced only in a medium which has its
boundary. Two independent waves from opposite directions can not produce stationary
wave.
Two types of stationary waves :
(i) Transverse stationary wave : When two identical transverse wave
travelling in opposite directions superpose, a transverse stationary wave is formed.
Example: Waves in stretched string fixed between two supports.
(ii) Longitudinal stationary waves : When two identical longitudinal
waves travelling in opposite direction superpose, a longitudinal stationary wave
is formed Example : Waves in organ pipes, waves in Kundt’s tube etc.
Formation of stationary waves
Figure shows the two combing waves, one travelling to the right fig (a) the other to the
left fig (b) Figure (c) shows the resultant stationary waves, obtained by using the
superposition principle graphically.

Fig. 10.34
650 MECHANICS, HEAT, THERMODYNAMICS & WAVES
(i) At t = 0, the two waves superpose in same phase, so the amplitude of the
resultant wave (2A) is twice the amplitude of individual wave (A). All the particles
are at their positions of maximum displacement. These position are called
antinodes [fig (i)].

T
(ii) At t = , each wave has moved a distance of /4 in their respective direction.
4
The two waves now superpose in opposite phase. The amplitude of resultant
wave becomes zero. All the particles of the medium are now passing through
their mean position [fig (ii)].

T
(iii) At t = , each wave has moved a distance /2 in their respective direction. The
2
two waves are again in same phase, so the amplitude of the resultant wave is
(– 2A). All the particles at their positions of maximum displacement but in the
direction opposite to those at t = 0 [fig. (iii)].

3T 3
(iv) At t = , each wave has moved a distance in their respective directions.
4 4
The two waves now again in opposite phase, so that the amplitude of resultant
wave becomes zero. All the particles now again passing through their mean
T
positions, but their directions of motion are opposite to those at t = (fig. iv).
4
(v) At t = T, each wave has moved a distance in their respective directions. The two
waves are again in same phase, so the amplitude of resultant wave is 2A. This
complete one cycle [fig (v)].
The whole cycle continues to repeat again and again. In the superposition of two
identical waves from opposite directions, there are some particles are called
antinodes. The positions A1, A2, A3, ........ are antinodes. The positions N1, N2, N3
......... where the amplitude of oscillation is zero are called nodes. Clearly the
distance between two consecutive nodes or antinodes is /2. The distance
between node and the next antinode is /4.

Analytical Treatment of stationary waves


Reflection from fixed boundary:
Consider two identical waves travelling from opposite directions
y1 = A sin(kx t) … (1)
and y2 = A sin(kx t) … (2)
According to the principle of superposition, the resultant wave is given by
y = y1 + y2
= A sin(kx t ) A sin(kx t)
or y = [2 A sin kx]cos t … (3)
The equation does not represent a progressive wave because it does not contain the
combination like (kx t ) . Instead, it describes a stationary wave of frequency and
amplitude
R = 2A sin kx.
Clearly the amplitude of oscillation is not same for all the particles of the medium. It
varies harmonically with the location x of the particle.
WAVE - II 651
Change in R with x at any time t :
The amplitude of the stationary wave will be zero at points, where
sin kx = 0
or kx = n
2
or x = n

x = n , where n = 0, 1, 2, 3, ........
2

or x = 0,
, ,........
2
The positions of zero amplitude are called nodes the distance between two consecutive
nodes is /2.
The amplitude of the stationary wave will have a maximum value of 2A at points, where
sin kx = 1

or kx = (2n 1)
2
2
or x = (2n 1)
2

x = (2 n 1) , where n = 0, 1, 2, 3, .......
4
3 5
or x =, ,
, ............
4 4 4
These positions of maximum amplitude are called antinodes.
Clearly the distance between two consecutive antinodes is /2.
Change in R of any particle at x with time :
At the instant t = 0, T/2, 3T/2, ...........
2
cos t = cos
t = 1.
T
Thus at these instants the displacement y becomes alternately positive and negative.
That is, all the particles of the medium pass through their positions of maximum
displacements twice in each cycle.
At the instant t = T/4, 3T/4, 5T/4, ..........
2
cos t = cos
t=0
T
Thus at these instants the displacement y becomes zero at all the points. That is all the
particles of the medium pass through their mean positions simultaneously twice in each
cycle. Fig. 10.35
Characteristics of stationary waves
(i) In a stationary wave, the disturbance does not move in any direction. The
conditions of crests and troughs merely appear and disappear in fixed positions
to be followed by opposite conditions after every T/2.
(ii) All the particles of the medium, except those at nodes, execute simple harmonic
motions with the period of the wave about their mean position.
(iii) During the formation of a stationary wave, the medium is broken into loops
between equally spaced points called nodes which remain at rest and in between
them are points of maximum displacement called antinodes.
(iv) The amplitudes of the particles are different at different points. The amplitude
varies gradually from zero at the nodes to the maximum at the antinodes.
652 MECHANICS, HEAT, THERMODYNAMICS & WAVES
(v) The maximum velocity is different at different points. Its value is zero at the nodes
and gradually increases towards the antinode. All the particles attain their maximum
velocities simultaneously when they pass through their mean positions.
(vi) All the particles in a particular segment between two nodes vibrate in the same
phase but the particles in the neighbouring segments vibrate in opposite phases,
as shown in figure.
(vii) The energy becomes alternately wholly potential and wholly kinetic in twice in
each cycle. It is wholly potential when particles are at their positions of maximum
displacement and wholly kinetic when the particle pass through the mean
positions.
(viii) A stationary wave has the same wavelength and time period as the component
waves.
(ix) The distance between two consecutive nodes and antinodes is /2. The distance
between node and next antinode is /4.
Ex. 17 Consider the following wave functions :
(a) y = A sin( t kx), (b) y = A sin( kx t) , (c) y = A cos( t – kx) ,
(d) y = A cos(kx t) , (e) y = A sin( t kx ) , (f) y = A cos( t kx) .
Write the equations of reflected wave after reflection from a free and a fixed boundary. Also find the resulting stationary waves formed
by the superposition of its reflected wave.

Sol. Incident wave Reflected wave from Reflected wave from


2 2
= ,k= free boundary, =0 fixed boundary, =
T
(a) y = A sin( t kx) y = A sin( t kx ) y = A sin( t kx )
= A sin( t kx )
(b) y = A sin( kx t) y = A sin( kx t) y = A sin( kx t )
= A sin(kx t) = A sin( kx t)
(c) y = A cos( t kx ) y = A cos( t kx) y = A cos( t kx )
= A cos( t kx)
(d) y = A cos(kx t) y = A cos( kx t) y = A cos( kx t )
= A cos(kx t) = A cos(kx t)
(e) y = A sin( t kx ) y = A sin( t – kx ) y = A sin( t – kx )
= A sin( t – kx )
(f) y = A cos( t kx) y = A cos( t – kx) y = A cos( t – kx )
= A cos( t kx)

(a) Stationary wave by superposition of Ex. 18 The constituent waves of a stationary wave have
amplitude, frequency and velocity as 10 cm, 50 Hz and 200 cm/s
respectively. Write down the equation of the stationary wave for
x = 0, there is antinode.
Sol. For the given condition, the equation of stationary wave is
y = 2A cos kx sin t

Fig. 10.37 2 x
= 2 A cos .sin 2 ft.
y1 = A sin( t kx)
v 200
and y2 = A sin( t kx ) Here = f = = 4 cm and A = 10 cm
50
y = y1 y2
2 x
= A[sin( t – kx) sin( t kx )] y = 2 10cos sin 2 50t
4
= 2 A cos kx sin t
The similar treatment can be done for other combination of waves x
= 20cos sin100 t cm . Ans.
to get stationary wave. 2
WAVE - II 653
Ex. 19 The transverse displacement of string (clamped at its v 180
two ends) is given by Frequency f = = = 60 Hz.
3
x (c) The speed of the transverse wave in the string is given by
y( x, t ) = 0.06sin cos(120 t )
3
where x, y are in m and t in s. The length of the string is 1.5 m and F
v = v
its mass is 3.0 × 10–2 kg. Answer the following: µ
(a) Does the function represent a travelling or a stationary wave?
(b) Interpret the wave as a superposition of two waves travelling F = F v 2µ
in opposite directions. What are the wavelength, frequency
and speed of propagation of each wave? 2
(c) Determine the tension in the string. 3.0 10
Here µ =
Sol. 1.5
(a) The given equation
= 2 10 2 kg/m, v 180 m/s
2 x
y ( x, t ) = 0.06sin cos(120 t ) …(i) F = 1802 2 10 2 = 648 N Ans.
3
represents a stationary wave because it is the product of two
separate harmonic functions of x and t. Note:
(b) Suppose two waves
1. When two waves of same frequency but different amplitudes
2 travel from opposite directions, a stationary wave is formed.
y1 = A sin ( x vt )
In this case there is no nodes but there are position of
and other reflected from fixed boundary minimum amplitude.
2 Amax = A1 A2
y2 = A sin ( x vt )
Amin = A1 A2
2
= A sin ( x vt )
By principle of superposition,
2 2
y = y1 y2 = A sin ( x vt ) A sin ( x vt )
Fig. 10.38
2 x 2 2. All harmonics are overtones but all overtones are not
or y = 2 A sin cos (vt ) … (ii)
harmonics. Overtones which are non-integral multiples of
Comparing the equations (i) and (ii), we get the fundamental are not harmonics. For fundamental
= 3m frequency 200 Hz, 300 Hz is overtone but not harmonic.
2 v 3. The property of a device to reproduce the original sound
and = 120 or v = 60 = 60 3 = 180 m/s in all its details is called fidelity of an instrument.

Difference between progressive and stationary waves

1. The disturbance move forward with a definite speed. 1. The disturbance remain confined at its place.
2. Each particle of the medium executes SHM about its 2. Except nodes, all the particles of the medium execute
mean position with the same amplitude. SHM with varying amplitude.
3. No particle of the medium is permanently at rest. 3. The particles of the medium at nodes are at rest.
4. There is continuous phase change from 4. All the particles between two successive
particle to particle. nodes vibrate in same phase.
5. There is no instant when all the particles 5. All particles of the medium pass through mean
are at the mean position together. position twice during each cycle.
6. There is transfer of energy from one place to another. 6. Energy of the disturbance remain confined in that region.
7. The energy averaged over one cycle is half 7. The energy becomes alternately wholly kinetic and
kinetic and half potential. wholly potential twice in a cycle.
654 MECHANICS, HEAT, THERMODYNAMICS & WAVES
10.8 STATIONARY WAVES IN STRETCHED STRING FIXED AT THE
ENDS
Qualitative discussion : Consider a string under tension, fixed at both ends and
lying along the x-axis. If the string is plucked at any point, two identical waves start from
the point in opposite directions. After reflection from the fixed ends, these waves produce
stationary waves. By plucking the string at different suitable points, it can be set into
different modes of vibration.
Consider a string of length L under tension F. Let µ be the mass per unit length of the
string. The speed of the transverse wave on the string
F
v = .
µ
As the two ends of the string are fixed, so there is a node N at each end.
First mode of vibration : If the string is plucked in the middle, it vibrates in
one segment, giving its lowest or fundamental notes.
1
Here L = or 1 = 2L
2
Frequency of vibration
v 1 F
f1 = = =f
1 2L µ
This is the first harmonic.
Second mode of vibration : If the string is pressed in the middle and plucked
at the mid-point of either half, then the string vibrates in two segments.
2 2
Here L =
2 2
or 2 = L

v 1 F
Frequency of vibration f2 = =
2 L µ

2 F
=
2L µ
= 2f
This frequency is called first overtone or second harmonic.
Third mode of vibration : If the string is pressed at one-third of its length and
plucked at the middle of the smaller length, it vibrates in three segments.
3 3 3
Here L =
2 2 2
2L
or 3 =
3

v 3 F
Frequency of vibration f3 = =
3 2L µ
= 3f
This frequency is called second overtone or third harmonic.
In general, if the string vibrates in P segments (loops), then
P F
fp = = Pf
2L µ
WAVE - II 655
Analytical treatment : Consider a uniform string of length L under tension F
lying along the x-axis, with its ends fixed at x = 0 and x = L. Suppose a transverse wave
produced in the string travels along the string along positive x-direction and get reflected
at the fixed end x = L. The two waves can be represented as
y1 = A sin(kx t)
and y2 = A sin( kx t )
= A sin(kx t)
The resultant wave is given by
y = y1 + y2
= A sin(kx t ) A sin(kx t)
or y = 2 A sin( kx) cos( t ) … (1)
The ends x = 0 and x = L are fixed, so they must be nodes. The boundary conditions are:
x = 0, y = 0 for all t
and x = L, y = 0 for all t
The first boundary condition is satisfied automatically by the equation (1). The second
boundary conditions will be satisfied if
y = 2 A sin(kL) cos t = 0
This will true for all values of t only if
sin kL = 0
or kL = n , where n = 0, 1, 2, .......
2
or L = n

2L
= … (2)
n
th
The frequency of vibration of the string in its n mode

n F
fn = … (3)
2L µ
By putting x = 1, 2, 3, .........., the first, second, third, ..... harmonics will be obtained,
whose frequencies are in the ratio 1 : 2 : 3 :....... . The first harmonic is called fundamental
note. The higher harmonics are called overtones. Thus second harmonic is first overtone,
third harmonic is second overtones and so on.
Nodes : In the nth mode of vibration, there are (n + 1) nodes (see figure), their position
from x = 0 end are :
L 2L
x = 0, , , ......, L
n n

Antinodes : In the nth mode of vibration, there are n antinodes, their positions from
x = 0 end are :
L 3L 5L (2 n 1) L
x = , , ,......... .
2n 2 n 2n 2n

dy
Strain : It is defined by .
dx
dy d
Strain = = [2 A sin( kx) cos( t )]
dx dx
= 2 Ak cos(kx) cos( t )
656 MECHANICS, HEAT, THERMODYNAMICS & WAVES
The strain is maximum for all values of t, for which
cos kx = 1 or kx = n
2
or x = n

x = n
, x n , where n = 0, 1, 2, 3, ..........
2 2
x = 0, /2, , 3 /2, .........
These points are nodes. Thus strain is maximum at nodes and minimum at antinodes.
Melde’s experiment
The set up consists of a string which can be vibrated by an electrically maintained tuning
fork. The tension in the string is produced by hanging a load on the free end of the string.
There are two modes of vibration.
(i) Transverse mode of vibration: In this case tuning fork vibrates
perpendicular to the length of the string. The frequency of vibration of the string
is equal to the frequency of the tuning fork. If f is the frequency of the tuning fork,
then

P1 F
f = …(1)
2L µ
where P1 is the number of loops.
Fig. 10.40. Transverse mode of (ii) Longitudinal mode of vibration : In this case tuning fork vibrates
vibration of string. along the length of the string. In one complete vibration of the tuning fork, string
completes half vibration. Thus, if f is the frequency of tuning fork, then

f P2 F
= … (2)
2 2L µ
where P2 is the number of loops.
For the same tension in the string in both the modes of vibration and for equal length of
the string we have
P1
P2 = … (3)
2
Fig. 10.41. Longitudinal mode of
vibration of string. From equation (1), FP12 = 4 f 2 L2 µ = constant

From equation (2), FP2 2 = f 2 L2 µ = constant


Thus FP2 = Constant. … (4)

Ex. 20 Find the tension needed to produce stationary waves Ex. 21 In Melde’s experiment, when a string is stretched by a
with four loops in a string one metre long and 0.5 g in weight, fixed piece of glass it vibrate with 7 loops. When the glass piece is
to a tuning fork of frequency 200 Hz, when the prongs of the fork completely immersed in water the string vibrates in 9 loops what is
are vibrating perpendicular to the string. the specific gravity of glass?
Sol. If string vibrates in P loops, then Sol.
If F1 and F2 are the tensions in the string in the two cases, then
P F
f =
2L µ F1P12 = F2 P22

4L2 f 2 µ F1 P2 2 92
or F = or = =
P2
F2 P12 72

3 81
2 2 0.5 10 =
4 1 200 49
1
= = 5 N Ans. If Fb is the buoyant force on the glass piece, then
42 F b = F1 – F2
WAVE - II 657

Wt in air Ex. 23 The length of a wire between the two ends of a sonometer
Specific gravity of glass = Loss in weight in water is 105 cm. Where should the two bridges be placed so that the
fundamental frequencies of the three segments are in the ratio 1 :
F1 F1 3 : 15 ?
= =
Fb F1 F2 Sol.
If L1, L2 and L3 are the lengths of the three segments, then
1
= L1 L2 L3 = 105 … (i)
F
1 2
F1 1 1
As f or L f ,
L
1
= = 2.53 Ans. 1 1 1
49 L1 : L2 : L3 = : : = 15 : 5 :1 … (ii)
1 1 3 15
81
Sum of the ratio = 15 + 5 + 1 = 21
Ex. 22 The length of a sonometer wire is 0.75 m and density From equations (i) and (ii), we get
9 × 103 kg/m3. It can bear a stress of 8.1 × 108 N/m2 without exceeding
15
the elastic limit. What is the fundamental frequency that can be L1 = 105 = 75 cm
produced in the wire? 21
Sol. 5
105 = 25 cm
L2 =
The fundamental frequency in the wire is given by 21

1 F 1
f = L3 = 105 = 5 cm
2L µ 21
µ can be written as A × 1 × Hence the bridges should be placed at 75 cm and 75 + 25 = 100 cm from
one end.
1 F 1 stress
f = =
2L A 2L

1 8.1 108
= = 200 Hz Ans.
2 0.75 9 103

10.9 STATIONARY LONGITUDINAL WAVES IN ORGAN PIPES


When air is blown through the mouth, the sound waves move along the pipe and get
reflected at its end (open or close), producing stationary waves. If both the ends of the
pipe are open, it is called an open organ pipe. If one end of the pipe closed, it is called
closed organ pipe. Close end of the pipe behaves as rigid boundary and open end as the
free boundary for the displacement wave, so at the close end, displacement node or
pressure antinode forms and at the open end, displacement antinode or pressure node
forms.
Open organ pipe
Figure shows the various modes of vibrations of organ pipes in the form of displacement
and pressure waves.
658 MECHANICS, HEAT, THERMODYNAMICS & WAVES

(i) First mode of vibration : The lowest frequency note is called


fundamental note which is corresponding to the longest wavelength. Thus

1
L =
2
or 1 = 2L

v 1 P
Frequency of vibration f = = = f (say)
1 2L
This is the first harmonic.
(ii) Second mode of vibration :
2 2
Here L = = 2
2 2
or 2 = L

v 2 P
Frequency of vibration = =
2 2L
= 2f
This frequency is called first overtone or second harmonic.
(iii) Third mode of vibration :
3 3 3 3 3
Here L = =
2 2 2 2
v 3 P
Frequency of vibration f3 = =
3 2L
= 3f
This frequency is called second overtone or third harmonic.
Hence in open organ pipe, the harmonics of frequencies ratio 1 : 2 : 3 : ....... are
possible.

Analytical treatment : Consider an open organ pipe of length L lying along


the x-axis, with its ends at x = 0 and x = L. The sound wave travelling along the pipe can
be represented as
P = Pm sin(kx t)
The reflected sound wave from open end (rigid boundary) is represented by
P2 = Pm sin( kx t )
= Pm sin(kx t)
WAVE - II 659
The resultant stationary wave is given by
P = P1 P2
= Pm sin(kx t) Pm sin(kx t)
or P = 2 Pm sin(kx) cos( t )
For all values of t, the resultant pressure variation is zero, for which
sin kx = 0
or kx = n
2
or x = n

n
x = , where n = 0, 1, 2, 3, ........
2
3
x = 0, , ,, ..........
2 2
These points of zero pressure variation are called pressure nodes.
On the other hand, the pressure variation is maximum for all values of t, for which
sin kx = 1

or kx = (2 n 1)
2
2
or x = (2 n 1)
2

x = (2n 1) , where n = 0, 1, 2, 3, ................


4
3 5
x = ,
, ,........
4 4 4
These points of maximum pressure variation are called pressure antinodes.

Note:
1. If P0 is the normal pressure in the pipe, then at the positions of pressure nodes,
the pressure will be P0 and at the positions of pressure antinodes, it will be
P0 2 Pm or P0 2 ABk . Thus pressure at antinodes varies from P0 2 ABk to

P0 2 ABk .
2. The loud sound is heard at pressure antinode or displacement node.

Strain : We know that bulk modulus of medium (air) is

dP dP
B = =
dV ( dy / dx )
V

dy dP
strain = =
dx B

P
=
B
660 MECHANICS, HEAT, THERMODYNAMICS & WAVES

As P is maximum ( 2 Pm ) at the antinodes, so strain (positive or negative) at the


pressure antinodes and zero at pressure nodes.
At antinodes due to the compressions or the rarefactions of the oppositely travelling
waves, the strain becomes maximum. At nodes strain becomes zero due to the compression
of one wave coming across the rarefaction of the other, as shown in figure.
End corrections : Till now we have assumed that node/ antinode is formed just
at the open end. Lord Rayleigh showed that due to inertia the vibrating particles form
node/ antinode little above the open end of the pipe. So an end correction is applied
which is approximately e = 0.61 r, where r is the radius of the pipe. Thus,
for close organ pipe, the effective length Le = L + e,
for open organ pipe, the effective length Le = L + 2e.
Close organ pipe
Fig. 10.48, shows the various modes of vibration in the form of displacement and
pressure waves.

(i) First mode of vibration : In this mode of vibration


1
L = or 1 = 4L
4

v 1 P
Frequency, f1 = = = f (say)
1 4L
This frequency is called first harmonic or fundamental note.
WAVE - II 661
(ii) Second mode of vibration : In this mode of vibration

L = 2 2 = 3 2
2 4 4
4L
or 2 =
3
3 P v
Frequency f2
= 4L = = 3f
2
(iii) Third mode of vibration : In this mode of vibration
3 3 3 5 3
L = =
2 2 4 4
4L
or 3 =
5

v 5 P
Frequency , f3 = = = 5f
3 4L
This frequency is called second overtone or fifth harmonic.
Hence different frequencies produced in a closed organ pipe are in the ratio
1 : 3 : 5 : .......... i.e., only odd harmonics are present in a closed organ pipes.
Analytical treatment :
Consider a cylindrical pipe of length L lying along the x-axis with its closed end at x = 0
and open end at x = L.
The sound wave sent along the pipe can be represented as
P1 = Pm sin(kx t)
The reflected wave from the closed end is represented by (sound wave suffers no phase
change due to the reflection from closed end.).
P2 = Pm sin( kx t)
= Pm sin(kx t)
The resultant wave is given by
P = P1 P2
= Pm sin(kx t) Pm sin(kx t)
or P = –2 Pm cos(kx )sin( t )
For all values of t, the resultant pressure variation is zero, for which
cos(kx ) = 0

or kx = (2n 1)
2
2
x = (2 n 1)
2

x = (2n 1) , where n = 0, 1, 2, 3, ..........


4
3 5
x = ,
, ,........
4 4 4
These points of zero pressure variation are called pressure nodes. On the other hand, the
pressure variation is maximum for all values of t, for which
cos(kx ) = 1
or kx = n
662 MECHANICS, HEAT, THERMODYNAMICS & WAVES

2
or x = n

x = n , where n = 0, 1, 2, 3, ............
2
3
x = 0, , , ,......
2 2
These points of maximum pressure variation are called pressure antinodes (see Fig.10.49).
The pressure at these points varies from (P – 2 Pm) to (P0 + 2 Pm).

Resonance tube : It is used to determine the speed of sound in air with the help
of tuning fork of known frequency. It is a close pipe whose length can be changed by
changing level of liquid in the tube. When a vibrating tuning fork is brought over its
mouth, its air column vibrates longitudinally. If the length of the air column is varies until
its natural frequency becomes equal to the frequency of fork, then resonance will occur
and loud sound is heard.

For the first resonance L1 e = … (i)


4
3
and for second resonance L2 e = …(ii)
4
Here L1 and L2 are the length of resonance columns and e is the end correction.
After solving equations (i) and (ii), we get
= 2( L2 L1 )

L2 – 3L1
and e =
2
If f is the frequency of the fork, then speed of sound in air
v = f = 2 f ( L2 L1 )

Ex. 24 The first overtone of an organ pipe beats with the first Ex. 25 Determine the possible harmonics in the longitudinal
overtone of a close organ pipe with a beat frequency of 2.2 Hz. The vibration of a rod clamped in the middle.
fundamental frequency of the closed organ pipe is 110 Hz. Find the Sol. Consider a rod of length L clamped in the middle. It has one node
lengths of the pipes. Velocity of sound in air = 330 m/s.
in the middle and two antinodes at its free ends in the fundamental mode.
Sol.
Suppose Lo and Lc are the lengths of open and close pipes respectively.
Frequency of first overtone of open organ pipe,
2v v
fo = 2 Lo = Lo
Frequency of first overtone of close organ pipe
3v Fig. 10.52
fc = 4 Lc
1
Here L = 2 or 1 = 2L
Given fo fc = 2.2 Hz 4
Frequency of first harmonic
v 3v
Lo 4 Lc = 2.2 v v
f1 = =
1 2L
v
As 4 L = 110 Hz and v = 330 m/s
c

330
3 110 = 2.2
Lo
or L o = 0.99m Ans. Fig. 10.53
WAVE - II 663
In the second mode of vibration
v
3 (d) f =
L =
2 2 2 2
=
2 4L
4 2 2 4 2
v 400
2L L = 4 f = 4 25 = 4m Ans.
or 2 =
3
Ex. 27 Find the temperature T0 at which the fundamental
v 3v frequency of an organ pipe is independent of small variation in the
Frequency, f2 = = = 3f1 temperature in terms of the coefficient of linear expansion ( ) of
2 2L
the material of the tube.
This is called the third harmonic or first overtone .
Similarly for third mode
Sol.
f3 = 5f1. If L0 is the length of the pipe at T0, then its length at temperature T is
This is the fifth harmonic or second overtone. T = L0[1 (T T0 )]
Hence f1 : f 2 : f3 :....... = 1: 3 : 5: ........... RT
The speed of sound, v =
Ex. 26 Three successive frequencies for a string are 75, 125, M
175 Hz. We have to find the temperature T0 at which
(a) State whether the string is fixed at one end or at both ends. f (T0 ) = f (T ) for small (T – T0)
(b) What is the fundamental frequency?
(c) To which harmonics do these frequencies corresponds? RT0
RT
(d) Taking the speed of the transverse wave on the string as M
= M
400 m/s, determine the length of the string. 2 L0
2 L0 [1 (T T0 )]
Sol.
(a) The given harmonics are in the ratio 1 : 3 : 5, so the string is fixed T
or T0 = 1 (T T0 )
at one end.
(b) As the common maximum frequency in the harmonics is 25 Hz, so
1/ 2
fundamental frequency = 25 Hz. T T0
(c) The given harmonics are the third, fifth and seventh harmonics. or 1 = 1 (T T0 )
T0
For small (T – T0), we can write

1 T T0
1 = 1 (T T0 )
2 T0

1
or T0 = Ans.
2
Kundt’s tube : It is a long glass tube about 5 cm in diameter held horizontally. At
one end it carries a disc of cork or board connected with a metal rod which is clamped at
its middle. Other end of the tube is closed by a movable piston, so that its length can be
adjusted. Lycopodium power is spread on the box of the tube. The free end of the rod
rubbed along its length by resin cloth. The rod begins to vibrate longitudinally. These
vibrations forced air inside tube through disc. And so stationary longitudinal vibrations
are set-up in the tube. At resonance, frequency of vibration of rod becomes equal to
frequency of vibration of air column inside tube.
rod
For rod : = Lrod rod = 2Lrod
2
Fig. 10.55
air
For air : = Lair air = 2Lair
2
Since frod = fair
vrod vair vrod rod Lrod
= = =
rod air vair air Lair
By using Kundt’s tube one can compare the speed of sound in different mediums.
664 MECHANICS, HEAT, THERMODYNAMICS & WAVES

Ex. 28 A long wire PQR is made by joining two wires PQ and


Sol.
QR of equal radii. PQ has length 4.8 m and mass 0.06 kg. QR has
length 2.56 m and mass 0.2 kg. The wire PQR is under a tension of
80 N. A sinusodial wave-pulse of amplitude 3.5 cm is sent along the
wire PQ from the end P. No power is dissipated during the
propagation of the wave-pulse. Calculate
(a) the time taken by the wave-pulse to reach the other end R of
the wire and
(b) the amplitude of the reflected and transmitted wave-pulse
after the incident wave-pulse crosses the joint. Fig. 10.57
Length of aluminium wire = 0.6 m.
Sol. Mass per unit length of wire PQ Length of steel wire = 0.9 m
Mass per unit length of aluminium wire
0.06
µ1 = m
4.8 µ1 = = a = 2.6 103 1 10 6 ,

= 2.6 × 10–3 kg/m


Frequency of aluminium wire
Fig. 10.56
P1 F
1 f1 = 2 L1 µ1
= kg/m
80 For steel wire
0.2 20 µ2 = a = 1.04 104 1 10 6
and µ2 = = kg/m
2.56 256 2
= 1.04 10 kg/m

F 80 P2 F
Now v1 = f2 =
µ1 = 1/80
= 80 m/s 2 L2 µ2
For composite wire, f1 = f2
80
and v2 = F / µ2 = = 32 m/s Ans. P1 F P2 F
20 / 256 =
2 L1 µ1 2 L2 µ2
(a) Time taken by wave-pulse to reach the other end of wire
P1 L1 µ1
4.8 2.56 or =
t = = 0.14 s P2 L2 µ2
80 32

0.6 2.6 10 3 1
v2 v1 32 80 = =
(b) Ar = Ai v v = 3.5 0.9 1.04 10 2 3
1 2 32 80
The minimum number of loops of aluminium wire = 1
= – 1.5 cm Ans. and minimum number of loops of iron wire = 3
The total number of nodes in composite wire = 5
2v 2 2 32 excluding two of the ends, there are = 5 – 2 = 3.
and At = Ai = 3.5
v1 v2 80 32 P1 F
Lowest frequency f1 = 2 L1 µ1
= 2 cm Ans.
Ex. 29 An aluminium wire of cross-sectional area 1 × 10–6m2 1 10 9.8
is joined to a steel wire of the same cross-sectional area. This = = 16.2 Hz Ans.
2 0.6 2.6 10 3
compound wire is stretched on a sonometer, pulled by a weight of
10 kg. The total length of the compound wire between the bridges
Ex. 30 A 3.6 m long vertical pipe resonates with a source of
frequency 212.5 Hz. When water level is at certain height in the
is 1.5 m of which the aluminium wire is 0.6 m and the rest is steel
pipe. Find the heights of water level (from the bottom of the pipe) at
wire. Transverse vibrations are set up in the wire by using an which resonances occur. Neglect the correction. Now the pipe is
external source of variable frequency. Find the lowest frequency of
filled to a height of H ( 3.6m) . A small hole is drilled very close to
excitation for which standing waves are formed, such that the joint
its bottom and water is allowed to leak. Obtain an expression for
in the wire is a node. What is the total number of nodes observed at the rate of fall of water level in the pipe as a function of H. If the
this frequency, excluding the two at the ends of the wire? The radii of the pipe and hole are 2 × 10–2 m and 1 × 10–3 m respectively,
density of the aluminium is 2.6 × 10 3 kg/m3 and that of steel is calculate the time interval between the occurrence of first two
1.04 × 104 kg/m3. resonances. Speed of sound in air is 340 m/s and g = 10 m/s2.
WAVE - II 665
Sol. The wavelength of sound (a) What are the wavelength and frequency of the incident wave?
(b) Write the equation for the reflected wave.
v (c) In the resultant wave formed after reflection, find the
= f maximum and minimum values of the particle speeds in the
medium.
340 (d) Express the resultant wave as a superposition of a standing
= = 1.6 m
212.5 wave and a travelling wave. What are the positions of the
The first resonance length antinodes of the standing wave? What is the direction of
propagation of wave?
=
4 Sol.
(a) The given equation can be written in the form
1.6 yi = Asin (ax + bt + /2)
= = 0.4
4 Compare this equation with standard equation of travelling wave
Next resonance lengths are at 3 , 5 , 7 , 9 , y = A sin( kx t ) , we have
k = a and = b
= 1.2 m, 2.0 m , 2.8 m, 3.6 m, Frequency of wave
The water level in the pipe b
h1 = (3.6 – 0.4) = 3.2 m n = =
2 2
h2 = (3.6 – 1.2) = 2.4 m
h3 = (3.6 – 2) = 1.6 m 2 2
h4 = (3.6 – 2.8) = 0.8 m and wavelength = =
k a
h5 = (3.6 – 3.6) = 0 m
(b) The amplitude of reflected wave Ar = 0.80 Ai
At any height H
The equation of reflected wave
dH yr = 0.80 Ai cos( ax bt )
A = av
dt
yr = 0.80 Ai cos(bt ax) Ans.
= a 2 gH (c) Particle speed due to incident wave
dH a dyi
or = 2 g H 1/ 2 v1 = = Ab sin(ax bt )
dt A dt
Fig. 10.58 (v1)max = –Ab
r2 r2 Particle speed due to reflected wave
= 2 g H 1/ 2 = 2 g H 1/ 2
R 2
R2 dyr
v2 = = 0.8 Ab sin(at ax)
dt
3 2
1 10 (v2 )max = – 0.8Ab
= 2
2g H 1/ 2
2 10 The maximum value of particle speed in medium is
Rate of fall of level of water vmax = | v1 |max | v2 |max

dH = Ab 0.8 Ab = 1.8 Ab
= 5 5 10 3 H 1/ 2 Ans. The minimum value of particle speed in medium.
dt
vmin = Ab – 0.8 Ab = 0.2 Ab
The time interval between two occurrence of resonance : (d) The equation of resulting wave is
A dH y = yi yr
dt =
a 2 g H 1/ 2 = A cos( ax bt ) 0.8 A cos(bt ax )
2.4 = A cos( ax bt ) ( A 0.2 A) cos(bt ax )
R2 1/ 2
or t = H dH = A cos(ax bt ) – A cos(bt ax)
r 2 2 10 3.2
[0.2 A cos(bt ax)]
R2 2.4 3.2 ax bt bt ax
= 2 (1/ 2) = 2 A sin
r 20 2
= 43 s Ans.
bt ax ax bt
Ex. 31 The displacement of the medium in a sound wave is sin
2
0.2 A cos(bt ax)
given by the equation yi = A cos( ax bt ) where A, a, b are positive
= –2 A sin bt .sin ax 0.2 A cos(bt ax )
constants. The wave is reflected by an obstacle situated at x = 0.
The intensity of the reflected wave is 0.64 times that of the incident 2 A sin ax sin bt 0.2 A cos(bt ax )
wave. = standing wave travelling wave
666 MECHANICS, HEAT, THERMODYNAMICS & WAVES
The position of antinodes : (ii) As detector can detect the intensity > 2A2, so
I ( 2 A sin ax ) 2A2 = 4 A2 cos 2 ( f1 f 2 )t
Intensity is maximum, when sin ax = 1
1
or cos ( f1 f 2 )t =
or ax = (2n 1) , n = 0, 1, 2, ............ 2
2
3 5
or ( f1 f 2 )t = , , , .........
or x = (2n 1) 4 4 4
2a
1 3 5
3 5 t = 4( f , , ,....
= , ,
,.......... 1 f 2 ) 4( f1 f 2 ) 4( f1 f2 )
2a 2a 2 a
The time interval between two successive instants for which intensity
The travelling wave is moving along positive x-axis.
Ex. 32 Two radio stations broad-cast their programmes at the remains 2A2 is
same amplitude A and at slightly different frequencies f1 and f2 1
respectively. A detector receives the signals from the two stations t = 2( f1 f2 ) Ans.
simultaneously. It can only detect signals of intensity > 2A 2. Find
(i) time interval between successive maxima of the intensity of Ex. 33 A string under a tension of 129.6 N produces 10 beats/
the signal received by the detector. s when it is vibrated along with a tuning fork. When the tension in
(ii) Time for which the detector remains idle in each cycle of the the string is increased to 160 N, it sounds in unision with the same
intensity of signal. tuning fork. Calculate the fundamental frequency of the tuning
Sol. fork.
Assuming the detector is located at x =0, then the equations of the waves Sol.
broad-casting from the two stations Suppose f be the frequency of the tuning fork. The frequency of the
string will be either (f – 10) or (f + 10). With the increase in tension its
y1 = A sin 1t = A sin 2 f1t
frequency becomes f, so initial frequency of the string is (f – 10). Thus
and y2 = A sin 2t = A sin 2 f 2t
1 129.6
The resultant wave For F = 129.6 N, (f – 10) = … (i)
2L µ
y = y1 y2
= A sin 2 f1t A sin 2 f 2t 1 160
and for f = 160 N, f = … (ii)
2L µ
2 f1t 2 f 2t
= 2 A sin . After solving equations (i) and (ii), we get
2
f = 100 Hz Ans.
2 f1t 2 f 2t Ex. 34 A metal wire of diameter 1 mm is held on two knife
cos
2 edges separated by a distance of 50 cm. The tension in the wire is
100 N. The wire, vibrating with its fundamental frequency and a
f1 f2 vibrating tuning fork together produces 5 beats/s. The tension in
= 2 A sin 2 t cos f1 f2 t
the wire is then reduced to 81N. When the two are excited, beats
2
are heard at the same rate. Calculate
or y = R sin 2 f avt , (i) the frequency of the fork and
where R = 2 A cos ( f1 f 2 )t (ii) the density of the material of wire.
The intensity of resultant wave Sol.
Suppose the frequency of the tuning fork is f, then the frequency of wire
I R2 in first case be (f + 5). With the decreases in tension, the frequency of
4 A2 cos 2 ( f1 f 2 )t wire will decrease.So to give same number of beats/s, it should be (f – 5).
Thus
(i) Intensity of the resultant wave is maximum (4A2 > 2A2) for which
cos ( f1 f 2 )t = 1 1 F
f+5 =
2L µ
or ( f1 f 2 )t = n

n 1 100
t = or f+5 = … (i)
( f1 f2 ) 2L µ

1 2 1 81
or t = 0, , ,......... and f–5 = … (ii)
( f1 f 2 ) ( f1 f2 ) 2L µ
The time interval between successive maximas (i) After solving equations (i) and (ii), we get
1 f = 95 Hz
t = (f f2 ) Ans. (ii) From equation (i)
1
WAVE - II 667

1 100
Sol.
95 + 5 = Suppose f be the frequency of the fork, then frequency of air column of
2 0.50 µ 51°C be either (f + 4) or (f – 4). As the beat frequency decreases with
decrease in temperature, so it must be (f + 4). Thus
1
After solving µ = kg/m v51
100 (f 4) = … (i)
µ µ At 16°C, the frequency of the fork will be (f + 1),
Density = =
A r2 v16
(f 1) = … (ii)
1
= Dividing equation (i) by (ii),
100 (0.5 10 3 ) 2
(f 4) v51
= 12.73 × 103 kg/m3 Ans. =
(f 1) v16
Ex. 35 A tube of certain diameter and of length 48 cm is open at
both ends. Its fundamental frequency of resonance is found to be 273 51
320 Hz. The velocity of sound in air is 320 m/s. Estimate the = = 1.06
273 16
diameter of the tube. One end of the tube is now closed. Calculate
After solving, f = 49 Hz Ans.
the lowest frequency of resonance for the tube.
Ex. 37 AB is a cylinder of length 1.0 m filled with a thin flexible
Sol. diaphram C (see figure) at the middle and two other thin flexible
The tube with open ends is shown in figure. diaphrams A and B at the ends. The portions AC and BC contain
hydrogen and oxygen gases respectively. The diaphrams A and B
are set into vibrations of same frequency. What is the minimum
frequency of these vibrations for which the diaphram C is a node?
Under the conditions of the experiment, the velocity of sound in
hydrogen is 1100 m/s and in oxygen is 300 m/s.

Fig. 10.59
Suppose the radius of the tube is r. The effective length of the tube open Fig. 10.60
at both ends
Sol.
e = 0 2e = 0 2 0.6 r = 0 1.2 r As ends A and B are set in vibrations, so displacement antinodes are
formed at these ends. The fundamental
or = 48 + 1.2r
2

v 320
But = f = 320 = 1m = 100 cm

100 Fig. 10.61


= 48 + 1.2r
2 frequency of each pipe is corresponding to one node and one antinode. If
or r = 1.67 cm f1 and f2 be the fundamental frequencies of gases in AC and BC respectively,
Diameter d = 2r = 3.33 cm Ans. then
When one end of the tube is closed, then v1 1100
f1 = = = 550 Hz
4L 4 0.5
= 0 0.6r
4 v2 300
and f2 = = = 150 Hz
= 48 0.6 1.67 4L 4 0.5
= 196 cm As the two frequencies are different, so the two columns are not vibrating
in the fundamental mode. The close column of gas vibrates only in odd
v 320 100 harmonics with frequencies 1 : 3 : 5 : 7 : ........... . Thus we can write
Frequency, f = = = 163.3 Hz Ans.
196 f1 550 11 22 33
= = , ,
Ex. 36 A column of air at 51°C and a tuning fork produce 4 f2 150 3 6 9
beats/s when sounded together. As the temperature of air column is or 3f1 = 11f2
decreased, the number of beats per second tends to decrease and The common minimum frequency
when the temperature is 16°C then two produce one beat per second. = 3f1 = 3 × 550 = 1650 Hz
Find the frequency of tuning fork. also = 11f2 = 11 × 150 = 1650 Hz Ans.
668 MECHANICS, HEAT, THERMODYNAMICS & WAVES
Ex. 38 The first overtone of an open organ pipe beats with the L2 = 0.75 m Ans.
first overtone of a closed organ pipe with a beat frequency of 2.2 As the beats frequency is 2.2 Hz, so
Hz. The fundamental frequency of the closed organ pipe is 110 Hz.
Find the lengths of the pipes. 2v 3v
2 L1 4 L2 = 2.2
Sol.
Let L1 and L2 be the lengths of open and close pipes respectively.
2 330 3 330
The frequency of first overtone of an open organ pipe or – = 2.2
2 L1 4 0.75
2v
= 2L After solving
1
L 1 = 0.993 m Ans.
The frequency of first overtone of closed organ pipe
3v 3v 2v
= Also 4 L2 2 L1 = 2.2
4 L2
Given fundamental frequency of closed organ pipe
3 330 2 330
v or 4 0.75 2L1 = 2.2
4L 2 = 110
After solving L 1 = 1.006 m Ans.
330
or 4L2 = 110

Review of formulae & Important Points


16. Boundary effects : (a) Constructive interference :
(i) When wave is reflected from rigid boundary, the reflected = 2 n, n = 0, 1, 2, ....
wave will suffer a phase change of radian. Thus if or x = n
R max = (A 1 + A 2)
(b) Destructive interference :
yi A sin(kx t ), then
(2n 1) , n =1, 2, .....

yr A sin[ k ( x) t ] A sin(kx t) or x (2n 1)


2
(ii) When wave is reflected from free boundary, the reflected
wave suffers no phase change. R min = A 1 – A 2
Thus if The ratio of maximum and minimum intensities

I max ( A1 A2 ) 2
I min ( A1 A2 ) 2
yi A sin( kx t ), then
19. In Quinke's tube, each x cm slide of tube will cause a path
yr A sin( kx t) A sin(kx t) difference 2x. Thus for a mixima and next minima

17. Principle of superpositoin : If y 1 , y 2 , ..... y n are the x or = 4x.


displacements produced by waves acting separately, then the 2
resultant desplacement 20. Beats : In the superposition of two waves of slightly different
y = y1 + y2 + ...... + yn. frequencies
There are three types of superpositions. y1 A1 sin 2 f1t
18. Interference : When two or more waves of same frequency
travel simultaneously in the same direction or nearly along the and y2 A2 sin 2 f 2t
same direction in a medium, they superpose on each other and
number of maximum per second are f1 ~ f2, and number of
give rise new desturbance is called interference.
minimum per second are f1 ~ f2.
If A1 and A2 are the amplitudes of the interfering waves, then
Beats frequency, f = f 1 ~ f 2.
resultant amplitude

R A12 A22 2 A1 A2 cos


WAVE - II 669
21. Stationary waves : 22. Resonance tube : It is used to determine the speed of sound
(a) Consider two waves travelling from opposite directions in air.
y1 = A sin (kx – t) For the two consecutive resonances

and y 2 = A sin (kx + t) 3


L1 e and L2 e
The resultant wave 4 4
y = y 1 + y2 = [2A sin kx] cos t ( L2 3L1 )
= 2(L 2 – L 1) and e
2
Speed of sound v = f = 2f(L 2 – L 1)
23. Vibrations of stretched string :
(a) Frequency of fundamental note

1 F
f1
2L

Positions of nodes : x 0, , , .....


2

3
Positions of antinodes : x ,
, .....
4 4
(b) When wave is reflected from free boundary, then (b) If string vibrates in P loops, then

y1 A sin(kx t) P F
fp
2L
and y2 A sin( kx t) A sin(kx t)
24. Organ pipes
The resultant wave
(a) Open organ pipe : The frequency of fundamental note
y = [2A cos kx] sin t
v
f
3 2L
Positions of nodes : x , , .....
4 4 Harmonics of frequencies ratio 1 : 2 : 3 : ..... are possible.
(b) Close organ pipe : The frequency of fundamental note

v
f
4L
Harmonics of frequencies ratio 1 : 3 : 5 : ..... are possible.
670 MECHANICS, HEAT, THERMODYNAMICS & WAVES

MCQ Type 1 Exercise 10.1

Level -1
Only one option correct 5. A standing sound wave in a pipe has five displacement nodes and
1. Two waves with the same amplitude and wavelength interfere in five antinodes. The harmonic number for the standing wave is
three different situations to produce resultant waves with the (a) 3 (b) 4
following equations :
(c) 5 (d) 9
(1) y = 2 sin (3 x – 4t)
6. The tenth harmonic is set up in a pipe. The pipe is
(2) y = 2 sin (3x) cos (4t)
(a) open pipe (b) close pipe
(3) y = 2 sin (5x + 4t)
In which situation are the two combining waves travelling in (c) any of them (d) none
opposite directions ? 7. For a particular tube, there are four of the six harmonic frequencies
(a) 1 (b) 2 below 1000 Hz : 300, 600, 750 and 900 Hz. Two frequencies are
(c) 3 (d) None missing from the list are
2. In the sixth harmonic on a string fixed at both ends, the number of (a) 100, 200 Hz (b) 150, 400 Hz
nodes and antinodes are (c) 150, 450 Hz (d) 450, 800 Hz
(a) 5, 6 (b) 6, 6 8. Pipe A has length twice the pipe B. Pipe A has both ends open and
(c) 6, 7 (d) 7, 6 pipe B has one end open. Which harmonics of pipe A have a
3. Two standing waves are given by frequency that matches a resonance frequency of pipe B
y1 = (2 mm) sin (3x) cos (4t) (a) 1 (b) 2
and y2 = (2 mm) sin (3x + /6) cos (4 t). (c) 3 (d) 4
If waves are confined in the same distance, then the distance 9. A tuning fork arrangement (pair) produces 4 beat/s with one fork
between first node of wave y1 and first node of wave y2 is : of frequency 288 cps. A little wax is placed on the unknown fork
2 and it then produces 2 beat/s. The frequency of the unknown fork
(a) 0 (b) is
3
(a) 286 cps (b) 292 cps
(c) (d) (c) 294 cps (d) 288 cps
3 6
2 2
4. Figure shows sound waves of wavelength are emitted by a point 10. The equation f (x, t) = j sin ( vt)cos ( x) represents
source S and travel to a detector D directly along path 1 and via
reflection from a panel along path 2. Initially, the panel is almost (a) transverse progressive wave
along path 1 and the waves arriving at D along the two paths are
(b) longitudinal progressive wave
almost exactly in phase. Then the panel moved away from path 1
as shown until the waves arriving at D are exactly out of phase, the (c) longitudinal stationary wave
path difference x of the waves along the two paths (d) transverse stationary wave
11. A wave represented by the given equation y = a cos (kx – t) is
Panel superposed with another wave to form a stationary wave such
that the point x = 0 is a node. The equation for the other wave is
(a) y = a sin (kx + t) (b) y = –a cos (kx + t)
(c) y = –a cos (kx – t) (d) y = –a sin (kx – t)
S D 12. Standing waves can not be produced :
(a) on a string clamped at both the ends
(a) 0 (b) (b) on a string clamped at one end and free at the other
4
(c) when incident wave gets reflected from a wall
3 (d) when two identical waves with a phase difference of are
(c) (d)
2 2 moving in the same direction

Answer Key 1 (b) 3 (d) 5 (d) 7 (c) 9 (b) 11 (b)


Sol. from page 685 2 (d) 4 (c) 6 (a) 8 (a) 10 (d) 12 (d)
WAVE - II 671
13. A standing wave having 3 nodes and 2 antinodes is formed between (a) Zero
two atoms having a distance 1.21 Å between them. The wavelength (b) Purely kinetic
of the standing wave is : (c) Purely potential
(a) 1.21 Å (b) 2.42 Å (d) Partly kinetic and partly potential
(c) 6.05 Å (d) 3.63 Å 21. A man is standing on a railway platform listening to the whistle of
14. Two sinusoidal waves with same wavelengths and amplitudes an engine that passes the man at constant speed without stopping.
travel in opposite directions along a string with a speed 10 m/s. If If the engine passes the man at time t0. How does the frequency f
the minimum time interval between two instants when the string of the whistle as heard by the man changes with time :
is flat is 0.5 s, the wavelength of the wave is
(a) 25 m (b) 20 m
(c) 15 m (d) 10 m f f
15. Standing waves are produced in a 10 m long stretched string. If the
(a) (b)
string vibrates in 5 segments and the wave velocity is 20 m/s, the
frequency is
(a) 2 Hz (b) 4 Hz t0 t t0 t
(c) 5 Hz (d) 10 Hz
16. A string in musical instrument is 50 cm long and its fundamental
frequency is 800 Hz. If a frequency of 1000 Hz is to be produced, f
f
then required length of string is
(a) 62.5 cm (b) 50 cm (c) (d)
(c) 40 cm (d) 37.5 cm
17. A man is watching two trains, one leaving and the other coming in t0 t0 t
t
which equal speed of 4 m/s. If they sound their whistles, each of
frequency 240 Hz, the number of beats heard by the man (velocity 22. The figure shows four progressive waves A, B, C and D with their
of sound in air = 320 m/s) will be equal to phases expressed with respect to the wave A. It can be concluded
(a) 6 (b) 3 from the figure that
(c) 0 (d) 12
y
18. An open pipe is resonance in its 2nd harmonic with tuning fork of A C D
B
frequency f1. Now it is closed at one end. If the frequency of the
tuning fork is increased slowly from f1 then again a resonance is
obtained with a frequency f2. If in this case the pipe vibrates nth
harmonics, then O t
/2 /2 2

3 5
(a) n = 3, f2 = f (b) n = 3, f2 = f
4 1 4 1

5 3
(c) n = 5, f2 = f (d) n = 5, f2 = f
4 1 4 1
(a) The wave C is ahead by a phase angle of and the wave B
19. An organ pipe is closed at one end has fundamental frequency of 2
1500 Hz. The maximum number of overtones generated by this
pipe which a normal person can hear is lags behind by a phase angle of
2
(a) 14 (b) 13
(c) 6 (d) 9
(b) The wave C is behind by a phase angle of and the wave B
20. Two pulses in a stretched string whose centres are initially 8 cm 2
apart are moving towards each other as shown in the figure. The
speed of each pulse is 2 cm/s. After 2 s, the total energy of the lags ahead by a phase angle of
pulses will be 2
(c) The wave C is ahead by a phase angle of and the wave B
lags behind by a phase angle of
(d) The wave C is behind by a phase angle of and the wave B
lags ahead by a phase angle of
8 cm

Answer Key 13 (a ) 15 (c ) 17 (a ) 19 (c ) 21 (a )
Sol. from page 685 14 (d ) 16 (c ) 18 (c ) 20 (b ) 22 (b )
672 MECHANICS, HEAT, THERMODYNAMICS & WAVES
23. The diagram below shows the propagation of a wave. Which points 28. A standing wave is represented by y = A sin (100t) cos (0.01x)
are in same phase where y and A are in millimetre, t is in seconds and x is in metre.
The velocity of wave is
E
(a) 104 m/s
H (b) 1 m/s
(c) 10–4 m/s
A
D (d) Not derivable from above data
B 29. A stretched string of length l, fixed at both ends can sustain
F G
C stationary waves of wavelength , given by

(a) F and G (b) C and E f2 l2


(c) B and G (d) B and F (a) (b)
2l 2f
24. Two pulses travel in mutually opposite directions in a string with
a speed of 2.5 cm/s as shown in the figure. Initially the pulses are 2l
10 cm apart. What will be the state of the string after two seconds (c) (d) 2l f
f
10 cm 30. A string is rigidly tied at two ends and its equation of vibration is
given by y = cos 2 t sin 2 x. Then minimum length of string is

(a) 5
(a) 1m (b) m
2
(c) 5 m (d) 2 m
31. A source of sound placed at the open end of a resonance column
10 cm
sends an acoustic wave of pressure amplitude P0 inside the tube.
If the atmospheric pressure is PA, then the ratio of maximum and
(b) minimum pressure at the closed end of the tube will be
( PA P0 ) ( PA 2 P0 )
(a) ( PA P0 ) (b) ( PA 2 P0 )
(c)

1
PA PA P0
(c) (d) 2
(d) PA 1
PA P0
2

25. When two sound waves with a phase difference of /2, and each 32. A string fixed at both ends has consecutive standing wave modes
having amplitude A and frequency , are superimposed on each for which the distances between adjacent nodes are 18 cm and
other, then the maximum amplitude and frequency wave is 16 cm respectively. The minimum possible length of the string is
(a) 144 cm (b) 152 cm
A A (c) 176 cm (d) 200 cm
(a) : (b) :
2 2 2 33. A note has a frequency of 128 Hz. The frequency of a note which
is two octave higher than this is
(c) 2A: (d) 2A: (a) 256 Hz (b) 320 Hz
2
(c) 400 Hz (d) none of these
26. The displacement of the interfering sound waves are y1 = 4sin t 34. If the source is moving towards right, wave front of sound waves
get modified to
and y2 = 3 sin t . What is the amplitude of the resultant
2
wave
(a) 5 (b) 7
(c) 1 (d) 0 (a) (b)
27. Which two of the given transverse waves will give stationary
waves when get superimposed
z1 = a cos (kx – t) ...(A)
z2 = a cos (kx + t) ...(B)
z3 = a cos (ky – t) ...(C)
(a) A and B (b) A and C (c) (d) None of these
(c) B and C (d) any two

Answer Key 23 (d) 25 (d) 27 (a) 29 (c) 31 (a) 33 (d)


Sol. from page 685 24 (c) 26 (a) 28 (a) 30 (b) 32 (a) 34 (b)
WAVE - II 673

Level -2
Only one option correct 7. A tuning fork of frequency 392 Hz, resonates with 50 cm length of
1. Figure shows a stretched string of length L and four pipes of a string under tension (T). If length of the string is decreased by
length L, 2L, L/2 and L/2 respectively. The string’s tension is 2 %, keeping the tension constant, the number of beats heard
adjusted until the speed of waves on the string equals the speed of when the string and the tuning fork made to vibrate simultaneously
sound waves in the air. The fundamental mode of oscillation is is :
then set up on the string. In which pipe will the sound produced
(a) 4 (b) 6
by the string cause resonance
(c) 8 (d) 12
L
8. Two vibrating strings of the same material but lengths L and 2 L
have radii 2r and r respectively. They are stretched under the
(i) (ii) same tension. Both the strings vibrate in their fundamental modes,
(iii) (iv) the one of length L with frequency f1 and the other with frequency
f2. The ratio f1/f2 is given by
(a) (i) (b) (ii)
(c) (iii) (d) (iv) (a) 2 (b) 4
2. The superposing waves are represented by the following equations (c) 8 (d) 1
y1 = 5 sin 2 (10 t – 0.1 x), y2 = 10 sin 2 (20 t – 0.2 x). Ratio of 9. If in an experiment for determination of velocity of sound by
I max resonance tube method using a tuning fork of 512 Hz, first
intensities I will be resonance was observed at 30.7 cm and second was obtained at
min
63.2 cm, then maximum possible error in velocity of sound is
(a) 1 (b) 9 (consider actual speed of sound in air is 332 m/s)
(c) 4 (d) 16 (a) 204 cm/s (b) 110 cm/s
3. Equation of motion in the same direction is given by
(c) 58 cm/s (d) 80 cm/s
y1 = A sin ( t – kx), y2 = A sin ( t – kx – ). The amplitude of the
medium particle will be 10. The ends of a stretched wire of length L are fixed at x = 0 and
x = L. In one experiment , the displacement of the wire is y1 = A sin
(a) 2 Acos (b) 2 Acos x
2 sin t and energy is E 1, and in another experiment its
L
(c) 2 A cos 2 (d) 1.2 f, 1.2 2 x
displacement is y2 = A sin sin 2 t and energy is E2. Then :
4. A tuning fork of known frequency 256 Hz makes 5 beat/s with the L
vibrating string of a piano. The beat frequency decreases to (a) E2 = E1 (b) E2 = 2 E1
2 beat/s when the tension in the piano string is slightly increased.
(c) E2 = 4 E1 (d) E2 = 16 E1
The frequency of the piano string before increasing the tension
was 11. In a large room, a person receives direct sound waves from a
(a) 256 + 5 Hz (b) 256 + 2 Hz source 120 m away from him. He also receives waves from the
same source which reach him, being reflected from the 25 m high
(c) 256 – 2 Hz (d) 256 – 5 Hz
celling at a point halfway between them. The two waves interfere
5. The equation of stationary wave along a stretched string given by
constructively for wavelength of
x
y = 5 sin cos 40 t, where x and y are in cm and t in second. The 20 20
3
(a) 20, , etc (b) 10, 5, 2.5 etc
separation between two adjacent nodes is 3 5
(a) 1.5 cm (b) 3 cm (c) 10, 20, 30 etc (d) 15, 25, 35 etc
(c) 6 cm (d) 4 cm 12. In the experiment for the determination of the speed of sound in
6. A sonometer wire resonates with a given tuning fork forming air using the resonance column method. The length of the air
standing waves with five antinodes between the two bridges when column that resonates in the fundamental mode, with a tuning fork
a mass of 9 kg is suspended from the wire. When this mass is
is 0.1 m. When this length is changed to 0.35 m, the same tuning
replaced by a mass M, the wire resonates with the same tuning
fork resonates with the first overtone. The end correction is
fork forming three antinodes for the same positions of the bridges.
The value of M is (a) 0.012 m (b) 0.025 m
(a) 25 kg (b) 5 kg (c) 0.05 m (d) 0.024 m
(c) 12.5 kg (d) 1/25 kg

Answer Key 1 (d) 3 (a) 5 (b) 7 (c) 9 (d) 11 (a)


Sol. from page 686 2 (b) 4 (d) 6 (a) 8 (d) 10 (c) 12 (b)
674 MECHANICS, HEAT, THERMODYNAMICS & WAVES
13. A string of length 0.4 m and mass 10–2 kg is tightly clamped at its 19. A glass tube 1.0 m length is filled with water. The water can be
ends. The tension in the string is 1.6 N. Identical wave pulses are drained out slowly at the bottom of the tube. If a vibrating tunning
produced at one end at equal intervals of time t. The minimum fork of frequency 500 c/s is brought at the upper end of the tube
value of t which allows constructive interference between and the velocity of sound is 330 m/s, then the total number of
successive pulses is resonances obtained will be
(a) 0.05 s (b) 0.10 s (a) 4 (b) 3
(c) 0.20 s (d) 0.40 s (c) 2 (d) 1
20. A wave pulse on a string has the dimension shown in figure. The
14. A police car moving at 22 m/s, chases a motorcyclist. The policeman
wave speed is v = 1 cm/s. If point O is a free end, the shape of
sounds his horn at 176 Hz, while both of them move towards a
wave at time t = 3 s is
stationary siren of frequency 165 Hz. The speed of the motorcycle,
if it is given that he does not observe any beats is v = 1cm/s
1cm
Police car Motorcycle

1cm 1cm 2cm


Stationary siren
22 m/s v (165 Hz) O
(176 Hz) (a)

O
(a) 33 m/s (b) 22 m/s
(c) zero (d) 11 m/s 1cm
15. The equation of displacement of two waves are given as (b)
1cm
y1 = 10 sin (3 t + ); y2 = 5 (sin 3 t + 3 cos 3 t). Then what is
3 1cm
the ratio of their amplitudes (c)
(a) 1 : 2 (b) 2:1 1cm
(c) 1 : 1 (d) None of these
O
16. Two identical straight wires are stretched so as to produce 6 beats
per second when vibrating simultaneously. On changing the tension
O
in one of them, the beat frequency remains unchanged. Denoting
by T1, T2, the higher and the lower initial tension in the strings, 2cm
then it could be said that while making the above change in
tension (d)
1cm
(a) T2 was decreased (b) T2 was increased
(c) T1 was increased (d) T1 was kept constant 21. Two waves are propagating to the point P along a straight line
17. A metal wire of linear mass density of 9.8 g/m is stretched with a produce by two sources A and B of simple harmonic and of equal
frequency. The amplitude of every wave at P is ‘a’ and the phase
tension of 10 kg weight between two rigid supports 1 m apart.
of A is ahead by /3 than that of B and the distance AP is greater
The wire passes at its middle point between the poles of a
than BP by 50 cm. Then the resultant amplitude at the point P will
permanent magnet, and it vibrates in resonance when carrying an
be, if the wavelength is 1 meter
alternating current of frequency f. The frequency f of the alternating
source is (a) 2a (b) a 3
(a) 25 Hz (b) 50 Hz (c) (d) a
a 2
(c) 100 Hz (d) 200 Hz
22. The amplitude of a wave represented by displacement equation
18. An earthquake generates both transverse (S) and longitudinal (P)
sound waves in the earth. The speed of S waves in about 1 1
y sin t cos t will be
4.5 km/s and that of P waves is about 8.0 km/s. A seismograph a b
records P and S waves from an earthquake. The first P wave
arrives 4.0 min. before the first S wave. The epicenter of the a b a b
(a) (b)
earthquake is located at a distance about ab ab
(a) 25 km (b) 250 km
a b a b
(c) 2500 km (d) 5000 km (c) (d)
ab ab

Answer Key 13 (b) 15 (c) 17 (b) 19 (b) 21 (d)


Sol. from page 686 14 (b) 16 (b) 18 (c) 20 (d) 22 (d)
WAVE - II 675
23. Two travelling waves y1 = A sin [k (x – ct)] and y2 = A sin [k (x + ct)]
A
are superimposed on string. The distance between adjacent nodes
is
(a) ct / (b) ct / 2 B
(c) /2k (d) /k
24. A transverse sinusoidal wave moves along a string in the positive
1
x-direction at a speed at 10 cm/s. The wavelength of the wave is
0.5 m and its amplitude is 10 cm. At a particular time t, the snap- 1 2
shot of the wave is shown in figure. The velocity of point P when
h
its displacement is 0.05 m, is

y
3 3
P (a) ( 1 3 2 2 h) (b) ( 1 3 2 2 h)
2 2
x
2 2
(c) ( 1 3 2 2 h) (d) ( 1 3 2 2 h)
3 3
29. A massless rod is suspended by two identical strings AB and CD
of equal length. A block of mass m is suspended from point O such
that BO is equal to x .
3 ˆ 3 ˆ Further, it is observed that the frequency of 1st harmonic
(a) jm / s (b) jm / s
50 50 (fundamental frequency) in AB is equal to 2nd harmonic frequency
in CD. Then length of BO is
3 ˆ 3 ˆ
(c) i m/ s (d) i m/ s
50 50 A C
25. The stationary wave y = 2a sin kx cos t in a closed organ pipe is
the result of the superposition of y = a sin ( t – kx) and
(a) y = –a cos ( t + kx) (b) y = –a sin ( t + kx)
(c) y = a sin ( t + kx) (d) y = a cos ( t + kx)
26. While measuring the speed of sound by performing a resonance O
column experiment, a student gets the first resonance condition at B D
a column length of 18 cm during winter. Repeating the same
experiment during summer, she measures the column length to be
L
x m
x cm for the second resonance. Then
(a) x > 54 (b) 54 > x > 36
(c) 36 > x > 18 (d) 18 > x L 4L
(a) (b)
x 5 5
27. The equation of a plane progressive wave is y 0.9sin 4 t .
2 3L L
(c) (d)
2 4 4
When it is reflected at a rigid support, its amplitude becomes of 30. A vibrating string of certain length l under a tension T resonates
3
with a mode corresponding to the first overtone (third harmonic)
its previous value. The equation of the reflected wave is
of an air column of length 75 cm inside a tube closed at one end.
x x The string also generates 4 beats per second when excited along
(a) y 0.6sin 4 t (b) y 0.6sin 4 t
2 2 with a tuning fork of frequency f. Now when the tension of the
string is slightly increased the number of beats reduces to 2 per
x x second. Assuming the velocity of sound in air to be 340 m/s, the
(c) y 0.9sin 8 t (d) y 0.6sin 4 t
frequency f of the tuning fork in Hz is
2 2
(a) 344 (b) 336
28. A tube U–shaped has a uniform cross–section with arm lengths
(c) 117.3 (d) 109.3
1 and 2 ( 2 < 1). Tube has a liquid of density 1 filled to a height 31. Three waves of equal frequency having amplitudes 10 mm, 4 mm
1 and 7 mm arrive at a given point with successive phase difference
h. Another liquid of density 2 is poured in arm A. Both
2
liquids are immiscible. The length of the second liquid that should of . The amplitude of the resulting wave in mm is given by :
2
be poured in A so that first overtone of A is in unison with (a) 7 (b) 6
fundamental tone of B is (c) 5 (d) 4

Answer Key 23 (d) 25 (b) 27 (b) 29 (a) 31 (c)


Sol. from page 686 24 (a) 26 (a) 28 (d) 30 (a)
676 MECHANICS, HEAT, THERMODYNAMICS & WAVES

MCQ Type 2 Exercise 10.2


Multiple options correct 7. A student performed the experiment to measure the speed of sound
1. The displacement of a particle in string stretched in x direction is in air using resonance air-column method. Two resonances in the
represented by y. Among the following expressions for y, those air-column were obtained by lowering the water level. The resonance
describing wave motions are with the shorter air-column is the first resonance and that with the
(a) cos kx sin t (b) k2 x2 – 2 t2 longer air-column is the second resonance. Then,
(a) The intensity of the sound heard at the first resonance was
(c) cos (kx + t) (d) cos (k2 x2 – 2 t2)
more than that at the second resonance
2. Two point sources S1 and S2, which are out of phase, emit sound
(b) The prongs of the tuning fork were kept in a horizontal plane
waves of wavelength 2.0 m. The phase difference between the
above the resonance tube
waves arriving at point P if x1 = 18 m and x2 = 19 m
(c) The amplitude of vibration of the ends of the prongs is
S1 typically around 1 cm
P (d) The length of the air-column at the first resonance was
somewhat shorter than 1/4th of the wavelength of the sound
S2 in air
8. Standing waves are produced on a stretched string of length L with
fixed ends . When there is a node at a distance L/3 from one end,
(a) 0 (b) then
2
(a) minimum and next higher number of nodes excluding the
(c) (d) 2 ends are 2, 5 respectively
3. Coherent sources are characterized by the same (b) minimum and next higher number of nodes excluding the
(a) Phase and phase velocity ends are 2, 4 respectively
(b) Wavelength, amplitude and phase velocity (c) frequency produced may be v/3L
(c) Wavelength, amplitude and frequency (d) frequency produced may be 3v/2L
(d) Wavelength and phase 9. The equation of a stationary wave in a string is y = ( 4mm) sin
[(3.14 m–1) x] cos t
4. Standing waves can be produced
Select the correct alternative(s):
(a) On a string clamped at both the ends
(a) The amplitude of component waves is 2 mm
(b) On a string clamped at one end and free at the other (b) The amplitude of component waves is 4 mm
(c) When incident wave gets reflected from a wall (c) The smallest possible length of string is 0.5 m
(d) When two identical waves with a phase difference of are (d) The smallest possible length of string is 1.0 m
moving in the same direction 10. In a resonance tube experiment, a close organ pipe of length
5. It is desired to increase the fundamental resonance frequency in a 120 cm resonates when tune with a turning fork of frequency 340
tube which is closed at one end. This can be achieved by Hz. If water is poured in the pipe then (given vair = 340 m/sec)
(a) Replacing the air in the tube by hydrogen gas (a) minimum length of water column to have the resonance is
(b) Increasing the length of the tube 45 cm
(c) Decreasing the length of the tube (b) the distance between two successive nodes is 50 cm
(c) the maximum length of water column to create the resonance
(d) Opening the closed end of the tube
is 95 cm
6. A wave disturbance in a medium is described by y (x, t) = 0.02 cos
(d) none of these
11. A transverse sinusoidal wave of amplitude a, wavelength and
50 t cos (10 x), where x and y are in metres and t in frequency f is traveling on a stretched string. The maximum speed
2
of any point on the string is v/10 where v is the speed
seconds of propagation of the wave. If a = 10–3 m and v = 10 ms–1, then
(a) A displacement node occurs at x = 0.15 m and f are given by
(b) An antinode occurs at x = 0.3 m
(a) 2 10 2 m (b) 10 3 m
(c) The wavelength of the wave is 0.2 m
(d) The speed of the wave is 5.0 m/s (c) f 103 / 2 Hz (d) f 10 4 Hz

Answer Key 1 (a, c) 3 (a, d) 5 (a, c, d) 7 (a, d) 9 (a, d) 11 (a, c)


Sol. from page 688 2 (a, d) 4 (a, b, c) 6 (a, b, c, d) 8 (a, d) 10 (a, b, c)
WAVE - II 677
12. A closed organ pipe of length 1.2 m vibrates in its first overtone y
B
mode. The pressure variation is maximum at
C
(a) 0.8 m from the open end (b) 0.4 m from the open end A

(c) closed end (d) 1.0 m from the open end O


x
13. For a certain stretched string, three consecutive resonance
frequencies are observed as 105, 175, 245 Hz respectively. Then
select the correct alternatives(s) (a) The wave is travelling to the right.
(b) Displacement amplitude of the wave is equal to displacement
(a) The string is fixed at both ends of B at this instant.
(b) The string is fixed at one end only (c) At this instant velocity of C is also directed upwards.
(c) The fundamental frequency is 35 Hz
(d) Phase difference between A and C may be equal to .
(d) The fundamental frequency is 52.5 Hz 2
16. The (x, y) coordinates of the corners of a square plate are (0, 0),
14. An air column in a pipe, which is closed at one end will be in
(L, 0), (L, L) and (0, L). The edges of the plate are clamped and
resonance with a vibrating tuning fork of frequency 264 Hz, then transverse standing waves are setup in it. If u (x, y) denotes the
the length of the column in cm is displacement of the plate at the point (x, y) at some instant of
(a) 31.25 (b) 62.50 time, the possible expression (s) for u is (are) ( a = positive
constant)
(c) 93.75 (d) 125
x y x y
15. A wave is travelling along a string. At an instant, shape of the (a) a cos cos (b) a sin sin
2L 2L L L
string is as shown in figure. At this instant, point A is moving
upwards. Which of the following statements is / are correct : x 2 y 2 x y
(c) a sin sin (d) a cos cos
L L L L

12 (b, c) 14 (a, c) 16 (b, c)


Answer Key
Sol. from page 688 13 (b, c) 15 (b, d)
678 MECHANICS, HEAT, THERMODYNAMICS & WAVES

Statement Questions Exercise 10.3


Read the two statements carefully to mark the correct option out of the options given below:
(a) If both the statements are true and the statement - 2 is the correct explanation of statement - 1.
(b) If both the statements are true but statement - 2 is not the correct explanation of the statement - 1.
(c) If statement - 1 true but statement - 2 is false.
(d) If statement - 1 is false but statement - 2 is true.

1. Statement -1 7. Statement -1
Principle of superposition can be used for any physical quantity. Velocity of particles, while crossing mean position in case of
Statement -2 Principle of superposition can be used only when stationary waves varies from maximum at antinodes to zero at
amplitude of quantity is small. nodes.
2. Statement -1 Statement -2
It is not possible to have interference between the waves produced Amplitude of vibration at antinodes is maximum and at nodes, the
by two independent sources of same frequency. amplitude is zero, and all particles between two successive nodes
Statement -2 cross the mean position together.
For interference of two waves the phase difference between the 8. Statement -1
waves remain constant. Like sound, light can not propagate in vacuum.
3. Statement -1 Statement -2
To hear distinct beats, difference in frequencies of two sources Light waves are transverse in nature.
should not be greater than 10.
Wave Length
Statement -2 9. Statement -1 Speed of wave = time period .
Persistance of human ear is 10 per second.
4. Statement -1 Statement -2
Sound produced by an open organ pipe is richer than the sound Wavelength is the distance between two nearest particles in phase.
produced by a closed organ pipe. 10. Statement -1
Statement -2 The flash of lightening is seen before the sound of thunder is
Outside air can enter the pipe from both ends, in case of open heard.
organ pipe. Statement -2
5. Statement -1 Speed of sound is greater than speed of light.
In the case of a stationary wave, a person hear a loud sound at the 11. Statement -1
nodes as compared to the antinodes. When a beetle moves along the sand within a few tens of centimeters
Statement -2 of a sand the scorpion immediately turn towards the beetle and
In a stationary wave all the particles of the medium vibrate in dashes to it.
phase. Statement -2
6. Statement -1 When a beetle disturbs the sand, it sends pulses along the sands
The fundemental frequency of an open organ pipe increases as the surface one set of pulses is longitudinal while other set is transverse.
temperature is increased. 12. Statement -1
Statement -2 The reverberation time dependent on the shape of enclosure,
As the temperature increses, the velocity of sound increases more position of source and observer.
rapidly than length of the pipe. Statement -2
The unit of absorption coefficient in MKS system metric sabine.

Answer Key 1 (d) 3 (a) 5 (c) 7 (a) 9 (b) 11 (a)


Sol. from page 690 2 (a) 4 (b) 6 (a) 8 (d) 10 (c) 12 (d)
WAVE - II 679

Passage & Matrix Exercise 10.4

Passage for (Q. 1 - 3) : about its maximum which is located at a distance of 20 cm from the end
joined to the ring.
Waves y1 = A cos(0.5 x 100 t ) and y2 = A cos(0.46 x 92 t ) are
travelling along x-axis. (Here x is in m and t is in second)
1. The number of times intensity is maximum in time interval of 1 sec

(a) 4 (b) 6
(c) 8 (d) 10 7. Assuming that the wave is reflected from the ends without loss of
2. The wave velocity of louder sound is energy, the time taken by the string to regain its shape is
(a) 100 m/s (b) 192 m/s (a) 3 s (b) 6 s
(c) 200 m/s (d) 96 m/s (c) 2 s (d) 4 s
3. The number of times y1 + y2 = 0 at x = 0 in 1 sec is
8. The shape of the string changes periodically with time. The time
(a) 100 (b) 46
period is
(c) 192 (d) 96
(a) 3 s (b) 5 s
Passage for (Q. 4 - 6) : (c) 4 s (d) 2 s
A pulse is started at a time t = 0 along the + x direction on a long, taut 9. The tension in the string is
string. The shape of the pulse at t = 0 is given by function f (x) with (a) 1 × 10–3N (b) 1.5 × 10–3N
(c) 1.8 × 10–3N (d) 2 × 10–3N
x
1 for 4 x 0
4
Passage for (Q. 10 - 11) :
f ( x) x 1 for 0 x 1
0 otherwise

Here f and x are in centimeters. The linear mass density of the string is 50
g/m and it is under a tension of 5N.
4. The shape of the string is drawn at t = 0 and the area of the pulse
enclosed by the string and the x-axis is measured. It will be equal to
(a) 2 cm2 (b) 2.5 cm2
(c) 4 cm2 (d) 5 cm2
5. The vertical displacement of the particle of the string at 10. A sonometer wire 1 m long and weighing 1.25 g is in unison with
x = 7 cm and t = 0.01s will be a tuning fork of frequency 212 Hz when a wooden bridge is
(a) 0.75 cm (b) 0.5 cm placed at position A such that OA = 40 cm. If the weight is fully
immersed in water. The bridge A has to be moved towards O, 5 cm
(c) 0.25 cm (d) zero
to regain unison. The density of material of weight M is
6. The transverse of the particle at x = 13 cm and t = 0.015 s will be (a) 4267 kg/m3 (b) 3000 kg/m3
(a) – 250 cm/s (b) – 500 cm/s (c) 2800 kg/m 3 (d) None of these
(c) 500 cm/s (d) – 1000 cm/s 11. If two wires are stretched on a sonometer each wire with length
OA = 40 cm, one having mass in air and other in water and both
Passage for (Q. 7 - 9) :
vibrate simultaneously what will be the number of beats produced
A string of linear mass density 0.5 g/cm and a total length 30 cm is tied to
(a) 26.2 Hz (b) 20.2 Hz
a fixed wall at one end and to a frictionless ring at the other end. The ring
(c) 24.4 Hz (d) 30.6 Hz
can move on a vertical rod. A wave pulse is produced on the string which
moves towards the ring at a speed of 20 cm/s. The pulse is symmetric

Answer Key 1 (a) 3 (d) 5 (c) 7 (a) 9 (d) 11 (a)


Sol. from page 690 2 (c) 4 (b) 6 (a) 8 (b) 10 (a)
680 MECHANICS, HEAT, THERMODYNAMICS & WAVES

12. Two identical speakers emit sound waves of frequency 660 Hz uniformly in all directions. The audio output of each speaker is 1 mW and the
speed of sound in air 330 m/s. A point P is a distance 2m from one speaker and 3m from the other. Match the columns :

Column I Column II
A. Intensity of speaker S1 at P (p) 55.3 × 10–6 W/m2
B. Intensity of sound at P, if the speakers are driven coherently and in phase (q) 19.90 × 10–6 W/m2
C. Intensity of sound at P, if speakers are incoherent and out of phase by 180° (r) 28.7 × 10–6 W/m2
D. Intensity of sound at P, if speakers are incoherent. (s) 2.2 × 10–6 W/m2

13. The air column in a pipe closed at one end is made to vibrate in its second overtone by a tuning fork of frequency 440 Hz. The speed of sound
in air is 330 m/s. End correction may be neglected. Let P0 denotes the mean pressure at any point in the pipe and P0 the maximum amplitude
pressure variation. Then the columns :
Column I Column II
A. The length of the air column (p) 0.75 m
B. The amplitude of pressure variation at the middle (q) 0.94 m

C. Maximum pressure at open end (r) P0 / 2

D. Minimum pressure at closed end (s) P0


(t) (P0 – P0)

Answer Key
12 A (q); B (p); C (s); D (r) 13 A (q); B (r); C (s); D (t)
Sol. from page 686
WAVE - II 681

Subjective Integer Type Exercise 10.5


Solution from page 692
1. In an experiment it was found that the string vibrated in three 9. A source emitting sound of frequency 180 Hz is placed in front of
loops when 8 g were placed on the scale pan. What mass must be a wall at a distance of 2 m from it. A detector is also placed in front
placed on the pan to make the string vibrate in six loops ? Neglect
the mass of the string and the scale pan. of the wall at the same distance from
it. Find the minimum distance between
Ans. 2 g.
the source and the detector for which
2. A fork of unknown frequency when sounded with one of frequency the detector detects
288 Hz gives 4 beats/s and when loaded with a piece of wax again
gives 4 beats/s. How do you account for this and what was the a maximum of sound. Speed of
unknown frequency ? sound in air = 360 m/s.
Ans. 292 Hz.
3. Two sitar strings A and B playing the note ‘Ga’ are slightly out of Ans. x = 3.
tune and produce beats of frequency 6 Hz. The tension in the 10. A heavy string is tied at one end to a movable support and to a
string A is slightly reduced and the beat frequency is found to
light thread at the other end as shown in figure. The thread goes
reduce to 3 Hz. If the original frequency of A is 324 Hz, what is the
over a fixed pulley and supports a weight to produce a tension.
frequency of B ?
The lowest frequency with which the heavy string resonates is
Ans. 318 Hz. 120 Hz. If the movable support is pushed to the right by 10 cm so
4. At 16°C, two open end organ pipes, when sounded together produce that the joint is placed on the pulley, what will be the minimum
34 beats in 2 second. How many beats per second will be produced,
frequency at which the heavy string can resonate ?
if the temperature rises to 51°C ? Neglect the increase in length of
the pipes.
Ans. 18 s–1.
5. A column of air and a tuning fork produced 4 beats/s when sounded
together. The tuning fork gives the lower note. The temperature
of air is 15°C. When the temperature falls to 10°C, the two produce
3 beats/s. Find the frequency of the fork.
Ans. 110 Hz.
6. A guitar string is 90 cm long and has a fundamental frequency of
124 Hz. Where should it be pressed to produce a fundamental Ans. 240 Hz .
frequency of 186 Hz ?
11. A rotating disc contains two sets of holes in the form of equally
Ans. 60 cm.
spaced concentric circles.
7. A string,fixed at both ends, vibrates in a resonant mode with a
The area of innermost circle is equal to
separation of 2.0 cm between the consecutive nodes. For the next
higher resonant frequency, this separation is reduced to 1.6 cm. area between two circles.
Find the length of the string. A jet of air directed on the inner set of
Ans. 8.0 cm. holes produces a note, of frequency
100Hz. What will be the frequency of
8. Figure shows an aluminium wire of length 60 cm joined to a steel
note produced when a jet of of air is
wire of length 80 cm and stretched between two fixed supports.
directed on the outer set of holes ? The
The tension produced is 40 N. The cross–sectional area of the
spacingbetween inner and outer set of
steel wire is 1.0 mm2 and that of the aluminium wire is 3.0 mm2.
What could be the minimum frequency of a tuning fork which holesis same.
can produce standing waves in the system with the joint as a Ans. 1414Hz.
node ? The density of aluminium is 2.6 g/cm3 and that of steel is
12. A column of air at 51°C and a tuning fork produce 4 beats/s when
7.8 g/cm3.
sounded together. As the temperature of air column is decreased
the number of beats per second tends to decrease and when
temperature is 16°C the two produce 1 beat/s. Find the frequency
Ans. 180 Hz . of tuning fork. Ans. 50 Hz.
682 MECHANICS, HEAT, THERMODYNAMICS & WAVES

Subjective Exercise 10.6


Solution from page 693
1. Stationary waves are set up by the superposition of two waves resonance with the same source be observed if one end of the pipe
given by y1 = 0.05 sin (5 t – x) and y2 = 0.05sin (5 t + x) is closed ? Take the speed of sound in air as 330 m/s.
where x and y are in metre and t in second. Find the displacement Ans. Second harmonic of the open pipe, No resonance will be
of a particle situated at a distance x = 1m. observed with the source.
Ans. 0.054 m. 8. A metre–long tube open at one end, with a movable piston at the
2. The transverse displacement of a string (clamped at its two ends) other end, shows resonance with a fixed frequency source (a
is given by tuning fork of frequency 340 Hz), when the tube length is 25.5
cm or 79.3 cm. Estimate the speed of sound in air at the
y (x, t) = 0.06 sin (2 x/3) cos (120 t),
temperature of the experiment. Ignore edge effect.
where x, y are in m and t is in s. Ans. 346.8 m/s.
(i) Do all the points on the string oscillate with the same 9. A steel rod 100 cm long is clamped at its middle. The fundamental
(a) frequency, (b) phase and (c) amplitude ? frequency of longitudinal vibrations of the rod is given to be 2.53
Explain your answers. (ii) What is the amplitude of a point 0.375 kHz. What is the speed of sound in steel ?
m away from one end ? Ans. 5.06 km/s.
Ans. (i) (a) same (b) same (c) not same (ii) 0.042 m. 10. A set of 24 tuning forks is arranged in a series of increasing
3. (i) Write the equation of a wave identical to the wave represented frequencies. If each fork gives 4 beats/s with the preceding one
by the equation : y = 5 sin (4.0 t – 0.02 x) but moving in opposite and the last sounds the octave of the first, find the frequencies of
direction. (ii) Write the equation of stationary wave produced by the first and the last forks.
the composition of the above two waves and determine the distance Ans. 92 Hz, 184 Hz.
between two nearest nodes. All the distances in the equation are in
11. The two parts of sonometer wire divided by a movable knife
mm.
differ by 2 mm and produce one beat per second when sounded
Ans. (i) y = 5 sin (4.0 t + 0.02 x) together. Find their frequencies if the whole length of the wire is
(ii) y = 10 cos 0.02 x sin 4.0 t, 50 mm. one metre.
4. A wire stretched between two rigid supports vibrates in its Ans. 249.5 Hz, 250.5 Hz.
fundamental mode with a frequency of 45 Hz. The mass of the 12. Figure shows two wave pulses at t = 0 travelling on a string in
wire is 3.5 × 10–2 kg and its linear density is 4.0 × 10 –2 kg/m. What opposite directions with the same wave speed 50 cm/s. Sketch
is the shape of the string at t = 4 ms, 6 ms, 8 ms and 12 ms.
(i) the speed of a transverse wave on the string, and
(ii) the tension in the string ?
Ans. (i) 248 N (ii) 78.75 m/s.
5. A stone hangs in air from one end of a wire which is stretched over
a sonometer. The wire is in unison with a certain tuning fork when
the bridges of the sonometer are 45 cm apart. Now the stone
hangs immersed in water at 4 °C and the distance between the
bridges has to be altered by 9 cm to re–establish unision of the
wire with the same fork. Calculate the density of the stone.
13. A string of length L fixed at both ends vibrates in its fundamental
Ans. 2.778 g/cm3.
mode at a frequency f and a maximum amplitude A. (a) Find the
6. A wire having a linear mass density of 5.0 × 10–3 kg/m is wavelength and the wave number k. (b) Take the origin at one end
stretched between two rigid supports with a tension of 450 N. of the string and the x–axis along the string. Take the y–axis along
The wire resonates at a frequency of 420 Hz. The next higher the direction of the displacement. Take t = 0 at the instant when
frequency at which the same wire resonates is 490 Hz. Find the the middle point of the string passes through its mean position
length of the wire. and is going towards the positive y–direction. Write the equation
Ans. 2.14 m. describing the standing wave.
7. A pipe 30.0 cm long is open at both ends. Which harmonic mode Ans. (a) 2L, /L (b) y = A sin (x /L) sin (2 ft).
of the pipe is resonantly excited by a 1.1 kHz source ? Will
WAVE - II 683
14. In a Quinke’s experiment, the sound intensity has a minimum 20. A particular guitar wire is 30.0 cm long and vibrates at a frequency
value I at a particular position. As the sliding tube is pulled out of 196 Hz when no finger is placed on it. The next higher notes
by a distance of 16.5 mm, the intensity increases to a maximum of on the scale are 220 Hz, 247 Hz, 262 Hz and 294 Hz. How far
9I. Take the speed of sound in air to be 330 m/s (a) find the from the end of the string must the finger be placed to play these
frequency of the sound source (b) find the ratio of the amplitudes notes ?
of the two waves arrivingat the detector assuming that it does not Ans. 26.7 cm, 23.8 cm, 22.4 cm, and 20.0 cm.
change much between the positions of minimum intensity and
21. A wave is propagating along the length of a string taken as positive
maximum intensity.
Ans. (a) 5.0 kHz (b) 2. t x 2
x–axis. The wave equation is given by y = – – where
T
15. Two stereo speakers are separated by a distance of 2.40 m. A Ae
person stands at a distance of 3.20 m directly in front of one of the A = 5mm, T = 1.0 s and = 8.0 cm.
speakers as shown in figure. Find the frequencies in the audible (a) Find the velocity of the wave.
range (20–20000 Hz) for which the listener will hear a minimum
(b) Find the function f(t) representing the displacement of particle
sound intensity. Speed of sound in air = 320 m/s.
at x = 0.
(c) Find the function g(x) representing the shape of the string of
t = 0.
(d) Plot the function g(x) at the string at t = 0 and t = 5 s.

2 x 2
Ans. (a) 8 cm/s (b) Ae–(t/T) (c)
Ae

Ans. 200 (2n + 1) Hz, where n = 0, 1, 2,.......49.


16. Consider the situation shown in figure. The wire which has a mass
of 4.00 g oscillates in its second harmonic and sets the air column (d)
in the tube into vibrations in its fundamental mode. Assuming that
the speed of sound in air is 340 m/s, find the tension in the wire. 22. Two speakers connected to the same source of fixed frequency are
placed 2.0 m apart in a box. A sensitive microphone placed at a
distance of 4.0 m from their mid–point along the perpendicular
bisector shows maximum response. The box is slowly rotated till
the speakers are in a line with the microphone. The distance
between the midpoint of the speakers and the microphone remains
unchanged. Exactly 5 maximum responses are observed in the
microphone in doing this. Calculate the wavelength of sound
wave.
Ans. 0.4 m.
Ans. 11.6 N.
23. In a large room a person receives direct sound wave from a source
17. Show that if the room temperature changes by a small amount
120 m away from him. He also receives wave from the same
from T to T + T, the fundamental frequency of an organ pipe
source which reach him being reflected from the 25 m high ceilling
1 T at a point half way between them. For which wavelength will
changes from f to f + f, where f /f .
2 T these two sound waves interfere constructively ?
18. A tuning fork of frequency 256 Hz produces 4 beats per second
with a wire of length 25 cm vibrating in its fundamental mode. The
beat frequency decreases when the length is slightly shortened.
What could be the minimum length by which the wire be shortened
so that it produces no beats with the tuning fork?
Ans. 0.39 cm.
19. A violin player riding on a slow track plays a 440 Hz note. Another
violin player standing near the track plays the same note. When
the two are close by and the train approaches the person on the
ground, he hears 4.0 beats per second. The speed of sound in air
= 340 m/s. (a) Calculate the speed of the train. (b) What beat
frequency is heard by the player in the train? 10
Ans. 10, 5, , 2.5m, ............. .
Ans. (a) 11 km/h (b) a little less than 4 beats/s. 3
684 MECHANICS, HEAT, THERMODYNAMICS & WAVES
24. Two waves are represented by pressure changes angle of 45° with the positive x and positive y axes. In each case,
find the position at which the resultant intensity is always zero.
P1( x, t ) = p0 cos kx – t , Ans. (i) z1 = A cos (kx – t) and z2 = A cos (kx + t)
4 (ii) (x – y) = (2n + 1) ( /k) n = 0,1,2,3,..........
31. A movable bridge divides a sonometer wire into two parts , which
p0
P2 ( x, t ) = sin kx – t , differ in length by 1 cm and produce 4 beats /s when sounded
2 4 together. If the whole length is 100 cm, find the frequencies of the
parts.
2
with k = . What is the total pressure at (i) x = and t = 0 (ii) Ans. 202, 198 Hz.
32. Two narrow cylindrical pipes A and B have the same length. Pipe
x = and t = 2 / 3 . Also calculate the amplitude of total A is open at both ends and is filled with a monoatomic gas of molar
pressure. mass MA. Pipe B is open at one end and closed at the other end
3 5 and is filled with a diatomic gas of molar mass MB. Both gases are
Ans. p0 , Pressure amplitude = maximum pressure = p0 . at the same temperature.
2 2 2
(a) If the frequency to the second harmonic of the fundamental
25. The fundamental frequency of a sonometer wire increases by 6 Hz mode in pipe A is equal to the frequency of the third harmonic of
if its tension is increased by 44%, keeping the length constant. the fundamental mode in pipe B; determine the value of MA / MB
Find the change in the fundamental frequency of the sonometer .
wire when the length of the wire is increased by 20%, keeping the (b) Now the open end of pipe B is also closed (so that pipe B is
original tension in the wire. closed at both ends). Find the ratio of the fundamental frequency
Ans. 30Hz, 5Hz (decrease). in pipe A to that in pipe B.
26. A sonometer wire is stretched by a hanging a mass of 50.7 kg from Ans. (a) (400 / 189), (b) (3 / 4).
the open end of the wire. The volume of the hanging mass is 33. A tube is of shape shown in figure. The straight line portion
0.0075 m3 and the fundamental frequency of the wire is 260 Hz. If l1 = 100 cm and the side tube measures l2 = 185 cm. Find the
the hanging mass is completely immersed in water, what will be audible frequencies for which this device acts as a silencer (speed
the fundamental frequency of the wire ? of sound = 340 m/s).
Ans. 240 Hz.
27. A string of mass per unit length is clamped at both ends such
that one end of the string is at x = 0 and the other end at x = L.
When string vibrates in fundamental mode, amplitude of the

midpoint of string is a
Ans. 200, 600, 900, ........... 19800 Hz.
and tension in string is F. Find the total oscillation energy stored 34. In figure S is a sound source and O an observer at a horizontal
in the string. distance D, the direct wave from S and the wave reflected from
2 2 point A, at a horizontal level at altitude H are in same phase.
a F
Ans. . When the layer rises a distance h and the wave is reflected from
4L
point B, no signal is detected at O. Given that the incident and the
28. A metal wire of diameter 1 mm is held on two knife edges by a reflected rays make the same angle with the reflecting layer. Find
distance 50 cm. The tension in the wire is 100 N. The wire an expression for the wavelength of the waves in terms of D, H
vibrating with its fundamental frequency and a vibrating tuning and h.
fork together produce 5 beats/s. The tension in the wire is then
reduced to 81 N. When the two are excited, beats are heard at the
same rate. Calculate (i) frequency of a fork and (ii) the density of
material of wire.
Ans. (i) 95 cycles/s (ii) 12.7 × 103 kg/m3.
29. A metallic rod of length 1 m is rigidly clamped at its mid point.
Logitudinal stationary waves are set up in the rod in such a way
that there are two nodes on either sides of mid–point. The amplitude
of antinode is 2 × 10–6 m. Write the equation of motion at a point
2 cm from the mid–point and those of the constituents waves in
the rod. (Young’s modulus = 2 × 10 11 N/m2, density = 8000
kg/m3).
Ans. y = 2 × 10–6 cos 0.6 sin 25000 t, y1 = 1 × 10–6 sin
(25000 t – 5 x), y2 = 1 × 10–6 sin (25000 t + 5 x). Ans. 2 4( H h) 2 D 2 – 2 (4 H 2 D 2 ).
30. The following equations represent transverse waves : 35. A particle oscillates in a wave according to the law y = 4 cos2 0.5t
z1 = A cos (kx – t), z2 = A cos (kx + t) , z3 = A cos (ky – t) sin 1000t. How many harmonic components are there in the
Identify the combination(s) of waves which will produce (i) complex wave and what are their frequencies ?
standing wave(s) (ii) a wave travelling in the direction making an Ans. Three, 999, 1000, 1001Hz.
WAVE - II 685

Solutions Exercise10.1Level -1
1. (b) Equation y = 2 sin (3x) cos (4t) is the combination of two
320
wave equations. 17. (a) f1 = f = 240 320 4 = 243 Hz.
2. (d) In sixth harmonic, there are six loop and so, there are 7 nodes s
and 6 antinodes.
320
3. (d) and f2 = f = 240 = 237 Hz.
s 320 4
4. (c) For , x =
2 2 2 Beats frequency fb = f1~f2 = 6 Hz.
5. (d) Equal number of nodes and antinodes occur in close pipe. It
is corresponding to ninth harmonic. 18. (c) f1 = 2
2
6. (a) The even number of harmonics possible in open pipe.
7. (c) Six harmonics are; 150, 300, 450, 600, 750 and 900 Hz.
f2 = n
4
8. (a) fA = 2 2 and fB =
4
2 n
Clearly first harmonic of both the pipes f2 = f1 ; (where n is odd number.)
4
have equal frequency. A B
9. (b) f = 288 + 4 = 292 Hz or 284 Hz. As f 2 > f1, n = 5.
Suppose, f = 292 Hz; On waxing, let its frequency becomes 19. (c) A person can hear sound of frequency f 20000 Hz.
290, which on known tuning fork gives 2 beats/s. So f must
n ×1500 = 20000
be 292 Hz.
10. (d) The wave is going along x direction, while particles are 20000
oscillating along y direction, so it represents transverse or n = 1500 ; where n is an odd number
stationary wave.
= 13.33
11. (b) y1 = a cos kx t , if we take n = 13.
It is 13th harmonic or 6 overtones.
y2 = a cos kx t , then 20. (b) After 2 s, the each wave travels a distance = 2 × 2 = 4 m.
The wave shape is shown in figure.
y = y1 + y2 = 2 a sin kx cos t
Thus energy is purely kinetic.
= A cos t
For x = 0, A = 0, which represents node. 21. (a) f1 = f and f2 = f ; so the frequency of
vs vs
12. (d) For stationary wave, two waves must be from opposite
directions. whistle suddenly changes from f1 to f2.
13. (a) The wavelength 22. (b) For wave B, y = A and so /2 .
= 1.21 Å
For wave C, y = – A and so /2.
14. (d) T/2 = 0.5 s, T=1s
23. (d) The displacement of the points B and F are equal in magnitude
T = 10 × 1 = 10 m. and sign. So these points are in same phase.
f 24. (c) After 2 second, each wave travels a distance = 2.5 × 2 = 5 m
and so the shape in figure (c) is correct one.
15. (c) 5 = 10, = 4 m. 25. (d) The resultant amplitude
2
R = A12 A22 = A2 A2 = 2A ,
20
Now f = = = 5 Hz. but frequency remains the same.
4
1 26. (a) R = A12 A22 = 42 32 = 5
16. (c) As f = F/ ,
2 27. (a) Two identical waves from opposite directions will form
stationary wave.
f1 2
= 28. (a) On comparing with standard equation of stationary wave,
f2 1 y = A sin t cos kx, we get
= 100, k = 0.01
f1 800
or = 1 50 = 40 cm. 100
2 f2 1000 = = = 104 m/s
k 0.01
686 MECHANICS, HEAT, THERMODYNAMICS & WAVES
29. (c) For stationary waves in stretched string, 31. (a) The maximum pressure is = P0 + PA, at the closed end and
minimum pressure is PA – P0, at the open end.
2
= ; .
2 32. (a) = (18 – 16) = 2
2
30. (b) On comparing with standard equation, we get
= 2 and k = 2 = 2 × 2 = 4 cm.

2
or = 2 , = 1 m. The length of the string must be in integral multiple of .
2
The minimum of the given values is 144 cm.
Thus = = 0.5 m. 33. (d) f = 128 Hz.
2 The required frequency = 3 f = 3 × 128 = 384 Hz.
So the length of the string will be in integral multiple of 0.5, 34. (b) For a moving source, < (normal wavelength).
which is 2.5 m.

Solutions Exercise 10.1Level -2


1. (d) The fundamental frequency of the wire is 7. (c) The frequency of tuning fork, f = 392 Hz.
1
f = . Also 392 = F/ ..........(i)
2L 2 50
The frequency of pipes are : After decreasing the length by 2%, we have
1
f1 = ; f = ;f = ;f = f = F/ ..........(ii)
4L 2 4 2L 3 2 L / 2 4 4 L / 2 2 49
Thus the frequency of fourth pipe resonates with the From above equations,
frequency of wire. f = 400 Hz.
2. (b) A1 = 5 and A2 = 10.
Beats frequency = 8 Hz.
2 2
I max A1 A2 5 10 f1 l2r2 2L r
= 2 = 2 =9 8. (d) f2 = l1r1 = L =1
I min A1 A2 5 10 2r
3
3. (a) R = A2 A2 2 AA cos 9. (d) = 30.7 and = 63.2 cm
4 4
= 2 A cos / 2 = 65 cm.
4. (d) The frequency of piano wire may be The velocity of sound in air
f = 256 5 = 261 Hz or 251 Hz. v = f = 512 × 0.65 = 332.8 m/s.
Suppose f = 261 Hz; After increasing tension the frequency
As actual velocity of sound is 332 m/s, so the maximum
of wire may be 262 Hz, which will produce more number of
error in this can be 0.8 m/s.
beats with the fork. So the frequency of wire must be 251
10. (c) As I f 2; or I 2.
Hz.
x E1 2
5. (b) y = 5 sin cos 40 t . On comparing with standard equation 1
3 E2 = 2 = .
2 4
of standing wave, y = R sin kx cos t, we get
k = / 3 and = 40 11. (b) The path difference,
x = 2 × 65 – 2 × 60 = 60 m.
2
m

65

or = /3 , = 6 cm. For constructive interference


65

x = 10 = n 25 m
The separation between two adjacent nodes = / 2 = 3 cm. P
60m Q 60m
10
5 F 5 9g = ; For n = 1, 2, 3, ......; = 10, 5, 2.5....
n
6. (a) f= = .........(i)
2 2 Ceiling here acts as free boundary, so do not add phase
difference due to reflection.
5 /2
12. (b) 1+ e =
4
3 Mg
and f = ............(ii)
2
or 0.1 + e = .........(i)
From above equations, we get M = 25 kg. 4
WAVE - II 687

3
and 0.35 + e = .........(ii) 1 = = 0.165 m,
4 4
From above equations, we get
3
e = 0.025 m 2 = = 0.495 m,
4
m 10 2
13. (b) = = = 2.5 × 10–2 kg/m. 5
0.4 3 = = 0.825 m,
4
1 F 1 1.6 7
and f = = 2 and = = 1.155 m
2 2 0.4 2.5 10 4 4

1 4 As 4 is greater than 1 m, so allowed resonances are only


= 10 = 10 Hz.
0.8 5 three.
20. (d) At t = 3 s, the wave pulse will reach at the free end. At the
1 1 free end the displacement after superposition becomes
T = = 0.10 s.
f 10 R = 2 A = 2 × 1 = 2 cm.
21. (d) As = 1 m
176 0 0
14. (b) = 165 so, 0.5 m /2.
22
The total phase difference
Here v = 330 m/s, after simplifying , we get 3
v0 = 22 m/s = 2
3 3 = 120° A
0.5 m
B P
15. (c) y1 = 10 sin 3 t /3 , The resultant amplitude,

and y2 = 5sin 3 t 5 3 cos3 t . A= a 2 a 2 2aa cos120 = a.


Amplitude, A1 = 10 unit
2 2
2 1 1 1 1 a b
and A2 = 52 5 3 = 10 unit. 22. (d) A= =
a b
=
ab
.
a b
23. (d) y = A sin (kx – kct) + A sin (kx + kct)
I1 A12
= = 1. kx kct kx kct kx kct kx kct
I2 A22 cos
= 2 A sin 2 2
1 T1 = 2 A sin (kct). cos kx.
16. (b) f = ......(i)
2 2 2
Thus = k, =
k
1 T2
and f 6 = ......(ii)
2
The distance between adjacent nodes =
As T1 > T2, second equation must be 2 k
10 1
1 T2 24. (a) f = = = Hz.
f 6 = . 50 5
2
On increasing T2, the frequency of it may be (f + 6), which
again gives 6 beats/s. 2
= 2 f = .
5
1 F 1 10 g The particle velocity,
17. (b) f = = 3
2 2 1 9.8 10
2
= 50 Hz. vp = A2 y2 = 5 102 52
18. (c) If x be the distance of epicentre from the seismograph, then
x x 5 3 2
– = 4 × 60 = = 2 3 cm/s
vs vp 5

x x 3
or = 4 × 60 = m/s.
4.5 8 50
on simplifying, we get 25. (b) The superposition of y = a sin( t kx ) together with
x 2500 km
y = a sin( t kx ) will give
330
19. (b) = = = 0.66 m yR = a sin( t kx ) a sin( t kx)
f 500
The resonance lengths are : = 2a sin kx cos t .
688 MECHANICS, HEAT, THERMODYNAMICS & WAVES
26. (a) The second resonance length will be slightly greater than
y
three times the first in summer. Length of air column in arm B is B 2 h
4
Thus x > (3 × 18) = 54 cm
27. (b) The equation of reflected wave is Since first overtone of arm A is in unison with
fundamental tone of B.
2 x
y = 0.9 sin 4 t v v
3 2 3
4 A 4 B
= 0.6sin 4 (t x 2)
3y y
28. (d) Let y be the length of second liquid poured in A. Let the first =3 B 1 h 3 2 h
A 4 4
liquid come down by a level x in arm A and rises by x in arm
B.
2
y ( 1 3 2 2h )
3
A 29. (a) If F1 and F2 are the tension forces in strings AB and CD
respectively, then
B 1 F1 2 F2
A = ........(i)
2 2
B and F1x = F2 (L – x) ........(ii)
After solving above equations, we get
x = L/5.
2x
y v 340
30. (a) f = 3 = 3 4 0.75
C D 4
= 340 Hz.
The frequency of fork may be = 340 4 Hz.
Pressure at C = Pressure at D With the increase in tension in the string, beats frequency
decreases, so the frequency of fork must be = 340 + 4 = 344
2 y
2 gy 1 g (2 x) x y Hz.
2 1 4
31. (c) R= 32 42 = 5 mm
1
2 4 mm
2
Length of air column in arm A is 4 mm R
y 3y
A ( 1 h) ( y ) 1 h 7 mm 10mm 3mm
4 4

Solutions Exercise 10.2


1. (a,c) Explained in the theory of the chapter. 6. (a,b,c,d) On comparing the given wave with the standard equation
of wave, we have
2. (a,d) x = 19 – 18 = 1m
= 50 and k = 10
As = 2, 50
v = = = 5 m/s.
k 10
1m = 2 , or = rad
2
So = – = 0 or = + =2 . Also = 10 = 0.2 m.
3. (a,d) For coherent sources, waves must have same frequency and At node, R = 0
constant phase difference.
4. (a,b,c)Standing waves can be produced by the two identical waves or cos(10 x) = 0 10 x = 2 , 3 2 ,...
from opposite directions. It is possible in case (a), (b) and x = 0.05m, 0.15m, .....
(c). At antinode, R = maximum
v or cos(10 x) = ± 1
5. (a,c,d) f ; so the fundamental frequency can be increased or 10 x = 0, , 2 ,
2L
either by increasing wave velocity or by decreasing length x = 0, 0.1 m, 0.2 m, ....
7. (a,d) The intensity of second resonance will be smaller than
RT intensity of sound of first resonance. Consider end
of pipe. As v , so lighter gas will have large value of v.
M correction, the length of air column is slightly less than .
4
WAVE - II 689

L dy 2 v
8. (a,d) = a =
dt max 10
L/3 L/3 L/3

3 Amplitude a =
= L 2 10
2
2
= 2L or, = 20 a= 2 10 m
3

v 3v v 10 103
so f = = Frequency f = = 2 = Hz.
2L 2 10 2
9. (a,d) On comparing the given wave with the standard equation of
3
standing wave, we get 12. (b, c) l =
R = 2A = 4 mm, 4
A = 2mm
A
2
Also k = 3.14 or = 3.4 4 N l =1.2m

= 2m N
l
As l = 2 = = 1 m. m
2 4 3
Pressure variation will be maximum at displacement nodes
10. (a,b,c) i.e., at 0.4 m from the open end and at closed end.
25cm.

75cm. 13. (b, c) As f1: f2: f3 = 3 : 5 : 7 is fixed at one end. Its fundamental
120cm. frequency is
45cm. f1 l 05
f0 = = = 35 Hz
3 3
As v =
14. (a,c) For closed organ pipe,
v 340
= = = 1m v
340 f = n where n = 1, 3, 5...
4l
First resonance light
nv
1 l = 4f
R1 = = m = 25 cm.
4 4
nv (1)(330)
3 3 For n = 1, l1 = 4 f = = 100 cm = 31.25 cm
R2 = = m = 25 cm. 4 264
4 4
For n = 3, l3 = 3l1 = 93.75 cm
5 5
R3 = = m = 125 cm. For n = 5, l5 = 5l1 = 156.25 cm.
4 4
i.e.,third resonance does not establish. 15. (b,d) The separation between them is nearly
Now H2O is poured,
Minimum length of H2O column to have the resonance x 4 , 2
= 45 cm.
Distance between two successive nodes The amplitude of A or any other point will be equal to that
of B.
1
= = m = 50 cm. 16. (b,c) Since edges of the plate are clamped, so y
2 2
U(x,y) = 0 for (0, L) B
and maximum length of H2O column to create resonance OA; y = 0, 0 x L C (L, L)
i.e., 120 – 25 = 95 cm. AB; x = L; 0 y L
2 BC; x = L, 0 y L O A x
11. (a,c) y = a sin vt x (0,0) (L,0)
OC; x = 0, 0 0 y L
Above conditions are satisfied in (b) and (c).
dy 2 2
Particle velocity = a v cos vt x
dt
690 MECHANICS, HEAT, THERMODYNAMICS & WAVES

Solutions Exercise-10.3
1. (d) Principle of superposition can be used for vector quantity In stationary waves, particles in the same loop vibrate in
or tensor quantity. phase.
2. (a) In case of independent sources, the phase difference between v
6. (a) As f ; and so with increase in temperature v increases
them does not remain constant. 2l
more than l.
3. (a) Statement-2 is the answer of statement-1.
7. (a) Statement-2 is the answer of statement-1.
4. (b) Open pipe can produce more number of harmonics in
comparison to close pipe. 8. (d) Sound can not propagate in vacuum.

5. (c) At displacement nodes, pressure is maximum and so loud 9. (b) Both the statements are self explanatory.
sound is heard. 10. (c) Speed of light is very much greater than speed of sound.

Solutions Exercise-10.4
Passage (Q.1 – 3) : That is a triangular pulse
1. (a) 2. (c) 3. (d)
1 5
The equations are y 1 = A cos (0.5 x – 100 t) and Area of the pulse = [(4 1) (1 1)] = cm2
y2 = A cos (0.46 x – 92 t) represents two progressive wave 2 2
travelling in the same direction with slight difference in the
frequency. This will give the phenomenon of beats. T
5. (c) v = = 10m/s
Comparing it with the equation
y = A cos (kx – t), we get Solution of the wave equation that gives displacement of any
1 = 100 2 f1 = 100 f1 = 50 Hz and piece of the string at any time
2
k1 = 0.5 0.5 =4m ( x vt )
1 1 for vt 4 x vt
1
4
y f ( x, t ) ( x vt ) 1 for vt x vt 1
Wave velocity = 1f1 = 200 m/s [Alternatively use v = ]
k 0 otherwise
2 = 92 2 f2 = 92 f2 = 46 Hz
Therefore beat frequency = f1 – f2 = 4 Hz and
200 Using v = 1000 cm/s, t = 0.01 s vt = 10 cm.
2
k2 = 0.46 0.46 as (vt – 4) < (x = 7 cm) < vt
2= 46
2
1 1
200 y (7 10) 1 cm. 0.25 cm.
Wave velocity = ×46 = 200 m/s 4 4
46
Note : Wave velocity is same because it depends on the medium y
6. (a) Transverse velocity =
in which the wave is travelling. t
Now, at x = 0, at t = 0.015 s, vt = 15 cm.
y1 + y2 = (A cos 10 t) + (A cos 92 t) = 0 as for x = 13 cm., (vt – 4) < x < vt
y v
cos 100 t = – cos 92 t = cos (– 92 t) therefore, 250 cm/s
t 4
2n 1
= cos [(2n + 1) – 92 t t= Passage for (Q.7 – 9) :
192 7. (c) The situation is shown in figure. The distance travelled
1 x = 20 + 20
when t = 0, n = and when t = 1, = 40 cm
2
x 20 cm
191 The time required t =
n= 95.2 v
2
net amplitude is zero for n = 96 times (the nearest answer). 40
Passage (Q.4 – 6) : =
20
4. (b) Shape of the pulse at t = 0 = 2s.
y(cm) 8. (a) The distance travelled by the pulse to start from the same
position
x = 20 + 20 + 10 + 10
1 = 60 cm
x
x(cm) Thus time period T =
–4 0 1 v
60
= = 3s.
20
WAVE - II 691

F or I = ( I1 I 2 )2
9. (d) We have v =
= ( 19.9 8.85) 2 10 6
F = v2
= 55.3 × 10 W/m2
–6 .
= 0.202 × 0.5 × 10–1
= 2 × 10–3 N. (C) In the case 0 = rad
Passage for (Q.10 – 11) :
I = [ I1 I2 2 I1I 2 cos(4 )]
10. (a) The frequency of sonometer wire
= ( I1 I2 )2
1 F
f = 212 = =
( 19.9 8.85) 2 10 6
2 0.40
= 2.2 × 10–6 W/m2
1 Mg (D) For incoherent sources
= .
2 0.40 I = I1 I 2

m 1.25 10 3 = (19.9 8.85) 10 6


Here = =
1 = 28.7 × 10–6 W/m2
13. A (q); B (r); C (s); D (t)
= 1.25 10 3 kg / m. (A) The frequency of vibration
f = 440 Hz
1 Mg V g
Again 212 = ...(ii) v 330
2 0.35 Wavelength = f = 440 = 0.75 m
After solving above equations, we get
For the second overtone in closed pipe
M
= = 4267 kg/m3. 5 5
V L = = 0.75
11 (a) If f ' is the frequency when mass is in water, then 4 4
= 0.9375 m = 0.94 m
1 Mg V g (B) Equation of stationary wave with x from close end
f' = ...(iii)
2 0.4 = 2 Pm cos( kx)sin( t )
P
From equations (ii) and (iii), we get beats frequency Pressure amplitude
= 26.2 Hz. R = [2 Pmcos(kx)|
12. A (q); B (p); C (s); D (r)
(A) Intensity due to a source emitting sound uniformly in all Given 2 Pm = P0
directions is given by
R = | P0 cos(kx) |
P
I = 2 x
4 r2 P0 cos
=
3
1 10
So I1 = = 19.90 10 6 W/m2 At the middle of the pipe,
2
4 (2)
15
x = L/2 = m
1 10 3 32
I2 = = 8.85 × 10 –6 W/m2
4 (3)2
2 15
(B) The resultant intensity is given by R = P0 cos
(3/ 4) 32
I = I1 I2 2 I1I 2 cos
5
= P0 cos
2 4
where = x.
P0
v 330 1 =
Here = f = = and x = 3 – 2 = 1 m 2
660 2
(C) At the open end of the pipe, there is pressure node
2 Pressure there R = P 0
= 1 =4 (D) At the closed end of the pipe, pressure antinode is formed,
1/ 2
so
I = I1 I2 2 I1I 2 cos 4 Rmax = P0 + P0
Rmin = P0 – P0
= I1 I2 2 I1I 2
692 MECHANICS, HEAT, THERMODYNAMICS & WAVES

Solutions Exercise-10.5
1. For the given string, we have 7. If P and (P + 1) be the number of loops corresponding to two
consecutive resonances, then length of string
F1P12 = F2 P22 , L = P×2
2 = (P + 1) × 1.6
P1 P = 4
F2 = F1
P2 and L = 8.0 cm
8. If P1 and P2 are the number of loops in aluminium wire and steel
3
2 wire respectively then
= 8
6 P1 F
= 2g f = 2 L1 1
2. The unknown frequency
f ' = 288 ± 4 P2 F
= 292 or 284 Hz. = 2 L2 2
Suppose f ' = 292 Hz.
After waxing this frequency will decrease and so this may give 4 P1 L1 1
beat/s again when its frequency becomes 284 Hz. Thus unknown or = ...(i)
frequency will be 292 Hz. P2 L2 2
3. Given, fA = 324 Hz.
The frequency fB = 324 ± 6 1 2.6 3
Here =
= 330 Hz. 2 7.8 1
or = 318 Hz. = 1
When string A is waxed, its frequency will decrease and may give
P1 60
3 beats for the frequency 318 Hz. Thus = 1
fB = 318 Hz. P2 80
4. fbeat v 3
f beat1 = .
v1 4
fbeat2 = v2
3 40
Thus f = 3
273 16 2 0.6 2.6 3 10
= [ fbeat1 = 17] = 180 Hz.
273 51
fbeat2 = 18 beat/s 9. The wavelength of the sound wave S
5. If f be the frequency of the fork, then frequency of air column v
= f±4 = f
v
= 360 x
4L = 2m
As with decrease in temperature, beats frequency decreases, so 180
= 2m
v15
f+4 =
4L
D
v10 The path difference between reflected sound and direct sound
Also f+3 =
4L x
2
x = 2 22 x
f 4 v15 2
f 3 = v10 For maximum of sound,
x =
273 15
= 2
273 10 x
or 2 22 x =
f 110 Hz. 2
1 F = 2
6. We have, f1 = x = 3 m.
2L
10. In the first case, the joint is the antinode, and so
1 F v
or 124 = ...(i) 120 = .......... (i)
2 0.90 4L
In the second case, the joint becomes the node, and so
1 F
Also 186 = ...(ii) v
2L ' f = .......... (ii)
2L
From above equations, we get From equations (i) and (ii), we get
L' = 0.6 m f = 240 Hz.
WAVE - II 693
11. The number of holes in inner circle 12. If f be the frequency of tuning fork, then
2 R1 v51
n1 = , f
d = ±4
4L
2 R2 When temperature decreases, beats frequency also decreases, so
and in outer circle n2 = .
d v51
f = +4 ...(i)
Given R12 = ( R22 R12 ) 4L
v16
R2 = 2 R1. At 16°C, we have f = +1 ...(ii)
4L
f1 n1 R1 1 R (273 16)
Thus f2 = n2 = R2 = 2 Here v16 =
M
or f2 = 2f = 2f. R (273 51)
and v51 =
M
On solving above equations, we get
f = 50 Hz.

Solutions Exercise-10.6
1. The equation of stationary wave is 2
y = y1 + y2 3.5 10
4. The length of the wire, = 2
= 0.875 m
= 0. 05 sin (5 t – x) + 0.05 4.0 10
sin (5 t + x) Thus for fundamental mode of vibration
= 0.1 sin 5 t. cos x.
= (0.1 cos x) sin 5 t =
The amplitude of the wave is 2
R = 0.1 cos x. = 2 = 1.75 m.
The displacement of the particle situated at The speed of transverse wave
x = 1
v = f = 45 × 1.75 = 78.75 m/s.
R = 0.054 m.
2. (i) Frequency and phase of all the points on the string are same F
but amplitude depends on the distance of the particle. We have v =
(ii) Given,
F = v2
2 x
y = 0.06sin cos(120 t ). = 4 × 10–2 × (78. 75) 2
3
= 248 N.
2 x 5. Let V be the volume and be the density of the stone. In air
The amplitude, R = 0.06sin tension in wire
3
F = V g.
2 0.375 When hanged in water, then tension in the wire
= 0.06sin F' = V g–V g
3
For two lengths L and L', we have
= 0.042 m.
3. (i) Given, y1 = 5sin (4.0t 0.02 x). 1 V g 1 V( )g
=
The equation of the wave moving in opposite direction will 2L 2L '
be
L'
y2 = 5sin (4.0t 0.02 x). or =
L
(ii) The equation of stationary wave is,
y = y1 + y2
36 1
= 5sin (4.0t 0.02 x ) or =
45
+ 5sin (4.0t 0.02 x)
= 2.778 g/cm3.
= 10cos 0.02 x sin 4.0 t. 6. Suppose Pth
and (P + 1)th
be the number of loops corresponding
On comparing with, to the resonance frequencies 420 Hz and 490 Hz respectively.
2 x 2 t Then
y = R cos .sin ,
T P F
420 = ...(i)
we have = 100 mm. 2L
The distance between two nearest nodes
P 1 F
= = 50 mm. and 490 = ....(ii)
2 2L
694 MECHANICS, HEAT, THERMODYNAMICS & WAVES
= f + 92
490 P 1
= Given f' = 2f
420 P
or f + 92 = 2 f
or P = 6. f = 92 Hz
Substituting this value in equation (i), we get and f ' = 184 Hz.
11. If L1 and L2 be the lengths of two points of the wire, then
6 450
420 = 3
L1 + L2 = 1000 mm
2 L 5 10 and L1 – L2 = 2 mm
L = 2.14 m L 1 = 499 mm
7. The fundamental frequency and L 2 = 501 mm
They produce the frequency
v
f1 = v
4L
f1 =
2 L1
330
= = 550 Hz.
4 0.30 v
Second harmonic, f2 = 2 × 550 = 1100 Hz. and f2 =
2 L2
and f3 = 3 × 550 = 1650 Hz .
Clearly, a source of frequency 1.1 k Hz will resonantly excite the 1 1
or 1 = v
second harmonic of the open pipe. 2 0.499 2 0.501
If one end of the pipe is closed, then fundemental frequency
v = 250 m/s.
v Thus f1 = 250.5 Hz
f1' =
4L and f2 = 249.5 Hz
12. The distance travelled by the pulses in 4 ms
330 = 500 × 4 × 10–3 = 2 mm.
= = 275 Hz.
4 0.30 The shape of the string is shown in figure
Third harmonic f3' = 3 f1' = 825 Hz.
2
and f5' = 5 f1' = 1375 Hz.
–2 2 4 6 8 10 12 14
No resonance will be observed.
8. We know that the frequency of n th mode of vibration of closed end
Do the other part of the problem accordingly.
pipe is
(2n 1)v 13. (a) In this case = L
f = 2
4L
or = 2L
where n = 1, 2, .........
For two resonance lengths, L1 and L2, we have y

(2n1 1)v
f = O x
4 L1
L
(2 n2 1)v
= 2p
4 L2 Wave number, k =
l
2n1 1 25.5 1 2
or = =
2n2 1 79.3 3 2L
If n1 = 1,
then n2 = 2 = .
L
Thus f = 340
(2´1 -1)v 2 x
= (b) We have y = A sin .cos( t )
4 ´ 25.5
v = 346.8 m/s. Given, At t = 0
9. The wavelength = 2L L
= 2×1 x =
2
= 2 m. y = 0
The speed of sound v = f
= 2.53 × 103 × 2 L
2
= 5.06 × 103 m/s. A sin 2 cos(0 )
0 =
10. If f be the frequency of first fork, then frequency of last fork will 2L
be or cos = 0
f ' = f + (24 – 1) × 4
WAVE - II 695
The frequency of fundamental mode of vibration of the air column
or = .
2 v 340
= = = 85 Hz
4L ' 4 1
2 x
Now y = A sin cos(2 f t / 2)
2L 2 F 1 F
Given, 85 = =
x 2L 0.40 10 2
= sin (2 f t )
A sin
L F = 11.6 N
14. (a) The distance between minimum to next maximum 17. The fundamental frequency of organ pipe is given by
= 2 × 16.5 = 33 mm. v 1 RT
f = =
. M
This distance must be equal to
2
1/ 2
1 R
= T 1/ 2
Thus = 33 M
2
or = 2 × 33 f 1 T
= 66 mm. Thus we can write, = .
f 2 T
Frequency of sound
v 330 1 F
f = = = 5 kHz. 18. The frequency of fundamental mode of wire = .
3 2L
66 10
According to the given condition,
(a1 a2 )2 1
(b) Given, =
( a1 a2 ) 2 9 1 F
= 256 + 4
2L
a1 a2 1
= With the decrease in length of the wire its frequency increases and
a1 a2 3 beats frequency decreases, so
a1 1 F
= 2 = 256 – 4 = 252 ......... (i)
a2 2L
15. The path difference between two sounds For L' length of the wire, we have
x = 4 – 3.20
= 0.80 m. 1 F
=
For minimum sound intensity 2L '

3.20 m = 256 ......... (ii)


Dividing equation (i) by (ii), we have

2.40 m L' 252


= = 0.984
4m L 256
L' = 0.984 × 25 = 24.61 cm
Thus (L – L') = 25 – 24.61 = 0.39 cm.
19. The frequency heard by the person on the ground will be
x = (2n 1) f ' = 440 + 4
2 = 444 Hz.
If vs be the speed of the train, then
or 0.80 = (2n 1) ;
2 v
n = 0, 1, 2,.... 444 = 440 v vs
1.60 After putting, v = 340 m/s,
=
(2 n 1) and solving, we get vs 3 m/s 11 km/h

v
320 1 F
The frequency, f = = 20. Given, 196 = ......... (i)
1.60(2n 1) 2 30
= 200 (2n + 1) If x is the required distance for 220 Hz, then
16. The frequency of second harmonic of the wire
1 F
220 = ......... (ii)
2 F 2x
f =
2L Dividing equation (i) by (ii), we get
3
x = 26.7 cm
m 4 10 Do the other parts similarly.
Here, = = = 10–2 kg/m
L 0.40
696 MECHANICS, HEAT, THERMODYNAMICS & WAVES
21. Given, T = 1s
5 P0
1 1 P = sin 2
and so f = = = 1 Hz. 2 4
T 1
= 8.0 cm. 5 P0
= P0 sin
(a) The velocity of wave, 2 4
v = f = 1 × 8.0 = 8 cm/s
(b) For x = 0, 5 P0
= sin cos cos sin
2 2 4 4
t
y = T
Ae 5 P0
= cos sin
(c) For t = 0, 2 2
x
2 As tan = 2,
y = Ae 2
sin =
22. The path difference, x = 2 m 5
According to the given condition
1
2m and cos =
S1 S2 5
4m
5 P0 1 2 3
x = 5 P = = P0 Ans.
or 2 = 5 2 2 5 5 2 2
= 0.4 m 2
(ii) At x = ,t =
23. The path length SAP = 2 60 25 2 2
= 130 m 3
The path difference x = 130 – 120 5 P0 2
= 10 m P = sin 2
2 3 4
For constructive interference
x = n 5 P0 5
or 10 = n = P0 sin
2 12
10
= ; = – 0.224 P0
n 25. For L length of the sonometer wire, its fundamental frequency
n = 1, 2, ................
24. The resultant pressure 1 F
f = .............. (i)
2L
P = P0 cos kx t When tension is incresed by 44%, it becomes
4 F + 0.44 F = 1.44 F
P0 1 1.44 F
sin kx t Thus, f+6 = .............. (ii)
2 4 2L
Substituting, P0 = A sin From equations (i) and (ii), we get
P0 f = 30 Hz.
and = A cos When length of the sonometer wire is increased to L + 0.2 L = 1.2
2 L, its frequency
1 F
f' = .............. (ii)
Thus P = A sin cos kx t A cos sin kx t 2(1.2L)
4 4
From equations (i) and (iii), we get
f ' = 25 Hz.
or P = A sin kx t . Decrease in frequency
4 = 5 Hz.
26. The fundamental frequency is given by
2
P0 5 P0
where A = P02 = , 1 F1 Mg
2 2 f = = .
2L 2L
and tan = 2 When mass is immersed in water, the tension in the wire becomes
or = tan –1(2)
= ( Mg V g ),
5 P0 1
P = sin kx t tan 2 .......... (i) 1 Mg V g
2 4 and freguency, f' =
2L
(i) At x = , t = 0
After substituting the given values, we get
f ' = 240 Hz
WAVE - II 697
27. For fundamental mode of vibration The equation of stationary wave can be written as:
= 2 2 x
The equation of stationary wave is y = 2a cos .sin 2 ft
y = a sin kx cos t,
6 2 x
2 2 = 2 10 cos sin(2 12500 t )
where k = = = . 0.4
2
6
The amplitude of wave is given by = 2 10 cos5 x sin(25000 t )
A = a sin kx. At a point 2 cm from the mid point to the right,
The mass of dx length of string, x = 50 + 2 = 52 cm
dm = dx
The total energy stored in the string can be calculated as : y = 2 10 6 cos (5 0.52)sin(25000 t )
1 1 6
dE = (dm)v 2 = ( dx) 2 2
A = 2 10 cos (0.6 )sin(25000 t ).
2 2 The equations of constitutent waves are
2
y1 = 1 10 6 sin (25000 t 5 x)
= ( a sin kx )2 dx
2
and y2 = 1 10 6 sin (25000 t 5 x).
2 30. (i) For standing wave the two waves must travel from opposite
E = a 2 sin 2 kxdx directions. Thus these waves are;
2
0 z 1 = A cos (kx – t),
and z 2 = A cos (kx – t).
1 2 2 The resulting wave is given by
= a
4 z = z1 + z2
= A cos (kx – t) + A cos (kx + t)
v F/ = 2 A cos kx cos t. = R cos t.
Also, = 2 f =2 2
l 2 where R = 2 A cos kx.
The intensity of resulting wave
2
F I = R2
2 = .
2 = 4A2 cos2kx
The intensity is zero, when
2 2 2
cos2 kx = 0
1 2 F a F or cos kx = 0
Now E = a 2 =
4 4
or kx = (2n 1) ,
28. If f is the fundamental frequency of the wire, then 2
n = 0, 1, 2,.....
1 F 1 F
f+5 = =
2L 2L r2 or x = (2n 1)
2k
1 100 n = 0, 1, 2,.....
= (ii) For a wave propagating in a direction making an angle 45°
2 0.5 r2
with positive x and positive y-axis, the component waves must be
of equal amplitude. These are;
1 F' 1 81 z 1 = A cos (kx – t),
and f–5 = 2 = .
2L r 2 0.5 r2 and z 3 = A cos (ky – t).
Here f = 95 cycles/s, The sesultant wave is z = z1 + z3
and = 12.7 × 103 kg/m3. = A cos( kx t) A cos( ky t)
29. The situations is shown in figure.
k ( x y) k
y = 2 A cos t ) .cos ( x y)
2 2

k (x y) k ( x y)
x = 2 A cos cos t)
2 2
mid point
k (x y)
5 = R cos t)
Clearly = = 1m 2
2
= 0.4 m k ( x y)
where R = 2 A cos
2
5000
Speed of wave, v = = = 12500 Hz.. 2 2 k ( x y)
0.4 Intensity I = R2 = 4 A cos
2
698 MECHANICS, HEAT, THERMODYNAMICS & WAVES
For zero intensity 33. The path difference,
k (x y) x = 2 1 = 185 – 100 = 85 cm
cos = 0
2 For silence (zero intensity of sound)
k ( x y)
or = (2n 1) , x = (2n 1) ,
2 2 2
n = 0, 1, 2,......
n = 1, 2, .......
31. If 1, 2 be the lengths of two parts of the wire, then
= 100 or 0.85 = (2n 1)
1 2 2
and 1 2 = 1,
1.70
= 50.5 cm =
1 2n 1
and 2 = 42.5 cm.
v 340
1 F The frequency, f = =
1.70/(2n 1)
The frequency f1 =
2 1 = 200 (2n – 1)
Thus for n = 1, 2,.....
1 F
= ...(i) f = 200, 600, 900, Hz. .....
2 50.5 34. The length of the path SBO,
1 F 2
and f2 = ...(ii) D
2 49.5 x1 = 2 (H h)2 ,
2
Given f2 – f1 = 4
After solving above equations, we get and the length of the path SAO,
f1 = 198 Hz and 202 Hz.
2
32. (a) The frequency of second harmonic of pipe A D
x2 = 2 H2 .
2v A 2
fA = ,
2L
Thus path difference,
and frequency of third harmonic of pipe B x = x2 – x1
3vB
fB = . 2 2
4L D D
= 2 (H h) 2 2 H2
According to given condition 2 2
fA = fB
For no signal detected (destructive interference)
2v A 3vB
or =
2L 4L
x =
2
vA 3
vB =
4 2 2
D D
or = 2 (H h )2 2 H2
A RT 2 2 2
MA 3
= or = 2 4( H h) 2 D2 2 4H 2 D2
B RT 4
MB
35. Given, y = 4cos 2 0.5t / sin1000t
5
where A = = 2[2cos 2 0.5t ]sin1000t
3
= 2(cos t 1)sin1000 t
7
and B =
5 = 2cos t sin1000t 2sin100 t
5/ 3M B 3 = sin (100 t) + sin (999 t)
= + 2 sin 1000 t
7 / 5M B 4
Clearly, there are three waves of frequencies 999, 1000 and 1001
MA 400 Hz.
or MB =
189
(b) When pipe B is closed at both ends, its frequency
vB
f 'B =
2L
fA vA 3
Thus = .
f 'B vB = 4

Vous aimerez peut-être aussi